KEMBAR78
Calculus II Module | PDF | Integral | Function (Mathematics)
0% found this document useful (0 votes)
10 views177 pages

Calculus II Module

Copyright
© © All Rights Reserved
We take content rights seriously. If you suspect this is your content, claim it here.
Available Formats
Download as PDF, TXT or read online on Scribd
0% found this document useful (0 votes)
10 views177 pages

Calculus II Module

Copyright
© © All Rights Reserved
We take content rights seriously. If you suspect this is your content, claim it here.
Available Formats
Download as PDF, TXT or read online on Scribd
You are on page 1/ 177

CALCULUS II

CHAPTER-ONE:
INVERSE FUNCTIONS

Objectives:
By the end of this chapter, students will be able to:
 Find inverse of functions
 Find derivatives of inverse functions
 Find Inverses of trigonometric functions and their derivatives
 Find Inverse of hyperbolic functions and their derivatives

1.1 Properties of inverse functions


The idea of solving an equation 𝑦 = 𝑓(𝑥) for 𝑥 as a function of 𝑦, say 𝑥 = 𝑔(𝑦), is one of the
most important ideas in mathematics. Sometimes, solving an equation is a simple process; for
example, using basic algebra the equation

𝑦 = 𝑥3 + 1 𝑦 = 𝑓(𝑥)
Can be solved for 𝑥 as a function of 𝑦:
𝑥 = 3√𝑦 − 1 𝑥 = 𝑔(𝑥)
The first equation is better for computing 𝑦 if 𝑥 is known, and the second is better for computing
𝑥 if 𝑦 is known.

Definition: If the functions 𝑓 and 𝑔 satisfy the two conditions


𝑔(𝑓(𝑥)) = 𝑥 for every 𝑥 in the dornain of 𝑓.
𝑓(𝑔(𝑥)) for every 𝑦 in the domain of 𝑔.
then we say that 𝑓 and 𝑔 are inverse functions. Moreover, we call 𝑓 an inverse of 𝑔 and
𝑔 an inverse of 𝑓.

It can be shown that a function cannot have two different inverses. Thus, if a function 𝑓 has an
inverse, then the inverse is unique, and we are entitled to talk about the inverse of 𝑓. The inverse
of a function 𝑓 is commonly denoted by 𝑓 −1 (read"𝑓 inverse").

Note: The symbol 𝑓 −1 should always be interpreted as the inverse of 𝑓 and never as the
reciprocal 1⁄𝑓 .
If we use the notation 𝑓 −1 (rather than 𝑔) in the above Definition, and if we use 𝑥 as the
independent variable in the formulas for both 𝑓 and 𝑓 −1 I, then the defining equations relating
these functions are

Tesfaye G and Wanaw B Page 1


CALCULUS II

𝑓 −1 (𝑓(𝑥)) = 𝑥 for every 𝑥 in the domain of 𝑓


𝑓(𝑓 −1 (𝑥)) = 𝑥 for every 𝑥 in the domain of 𝑓 −1
EXAMPLE 1: show that
1
a) The inverse of 𝑓(𝑥) = 2𝑥 is 𝑓 −1 (𝑥) = 2 𝑥
1
b) The inverse of 𝑓(𝑥) = 𝑥 3 is 𝑓 −1 (𝑥) = 𝑥 3

1
Solution (a) 𝑓 −1 (𝑓(𝑥)) = 𝑓 −1 (2𝑥) = 2 (2𝑥) = 𝑥
1 1
𝑓(𝑓 −1 (𝑥)) = 𝑓 ( 𝑥) = 2( 𝑥) = 𝑥
2 2
1
−1 −1 (𝑥 3 )
Solution (b) 𝑓 (𝑓(𝑥)) = 𝑓 = (𝑥 3 )3 = 𝑥
1 1
𝑓(𝑓 −1 (𝑥)) = 𝑓 (𝑥 3 ) = (𝑥 3 )3 = 𝑥

Theorem: If an equation 𝑦 = 𝑓(𝑥) can be solved for 𝑥 as a function of 𝑦 ,then 𝑓 has an inverse
and the resulting equation is 𝑥 = 𝑓 −1 (𝑦).

EXAMPLE 2: Find the inverse of 𝑓(𝑥) = √3𝑥 − 2


Solution: From the above theorem we can find a formula for 𝑓 −1 (𝑦) by solving the equation
𝑦 = √3𝑥 − 2
For 𝑥 as a function of 𝑦.The computations are
𝑦 2 = 3𝑥 − 2

1
𝑥 = 3 (𝑦 2 + 2)

from which it follow that


1
𝑓(𝑦) = 3 (𝑦 2 + 2)

Theorem:(The Horizontal line test). A function 𝑓 has an inverse if and only if its graph is cut
at most once by any horizontal line.

EXAMPLE 3: We observed above that the function 𝑓(𝑥) = 𝑥 2 does not have an inverse. This is
confirmed by the horizontal line test. Since the graph of 𝑦 = 𝑥 2 is cut more than once by certain
horizontal lines.

Tesfaye G and Wanaw B Page 2


CALCULUS II

1
EXAMPLE 4: The function 𝑓(𝑥) = 𝑥 3 has an inverse [namely, 𝑓 −1 (𝑥) = 𝑥 3 ]. The existence of
an inverse is confirmed by the horizontal line test, since the graph of 𝑦 = 𝑥 3 is cut at most once
by any horizontal line.

Theorem: If the domain of f is an interval on which 𝑓′(𝑥) > 0 or on which 𝑓 ′ (𝑥) < 0 then the
function 𝑓 has an inverse.
EXAMPLE 3: The graph of 𝑓(𝑥) = 𝑥 2 + 𝑥 + 1 is always increasing on (∞, −∞), since
𝑓 ′ (𝑥) = 5𝑥 4 + 1 > 0
For all 𝑥 However, there is no easy way to solve the equation 𝑦 = 𝑥 5 + 𝑥 +l for 𝑥 in terms of 𝑦
(try it), so even though we know that 𝑓 has an inverse, we cannot produce a formula for it.

Sometimes a function that is not one-to-one can be made one-to-one by restricting its domain.
For example, although the function 𝑓(𝑥) = 𝑥 2 is not one to-one, the functions
𝑔(𝑥) = 𝑥 2 𝑥≥0
2
ℎ(𝑥) = 𝑥 𝑥≤0
Tesfaye G and Wanaw B Page 3
CALCULUS II

Which result from restricting the domain of 𝑓 are one to one since their graphs pass the
horizontal line test (the graph of 𝑔 is the right half of the parabola 𝑦 = 𝑥 2 and the graph
of ℎ is the left half. The inverses of 𝑔 and ℎ can be found by solving each of the equations
𝑦 = 𝑔(𝑥) and 𝑦 = ℎ(𝑥) for 𝑥 as a function of 𝑦. For example, to find the inverse of 𝑔 we solve
𝑦 = 𝑥2, 𝑥≥0
for 𝑥, which yields 𝑥 = √𝑦 hence, 𝑔−1 (𝑦) = √𝑦. Similarly, ℎ−1 (𝑦) = −√𝑦. Geometrically,
the graphs of 𝑔(𝑥) = 𝑥 2 , 𝑥 ≥ 0 and 𝑔−1 (𝑥) = √𝑥 are reflections of one another about the line
𝑦 = 𝑥. Which reveals that the graph of 𝑦 = √𝑥 is a portion of a reflected parabola.

Exercise 1.1

1. For each of the following determine whether 𝑓 and 𝑔 are inverse functions.
1
a) 𝑓(𝑥) = 4𝑥 , 𝑔(𝑥) = 4 𝑥
b) 𝑓(𝑥) = 3𝑥 + 1 , 𝑔(𝑥) = 3𝑥 − 1
3
c) 𝑓(𝑥) = √𝑥 − 2, 𝑔(𝑥) = 𝑥 3 + 2
4
d) 𝑓(𝑥) = 𝑥 4 , 𝑔(𝑥) = √𝑥
2. For each of the following functions find the formula of 𝑓 −1
a) 𝑓(𝑥) = (𝑥 + 2)4 , 𝑥 ≥ 0
b) 𝑓(𝑥) = √𝑥 + 3
c) 𝑓(𝑥) = 3𝑥 2 + 5𝑥 − 2 , 𝑥 ≥ 0
d) 𝑓(𝑥) = 𝑥 − 5𝑥 2 , 𝑥 ≥ 1

1.2 Derivative of inverse functions


In this section we are going to look how to find the derivative of the inverse of the function.
Theorem: (The Derivative Rule for Inverses). If ƒ has an interval I as domain 𝑓 ′ (𝑥) and exists
and is never zero on I, then 𝑓 −1 is differentiable at every point in its domain. The value
of (𝑓 −1 )′ at a point b in the domain of is the reciprocal of the value of 𝑓′ at the point
𝑎 = 𝑓 −1 (𝑏):
1
(𝑓 −1 )′(𝑏) = ′ −1
𝑓 (𝑓 (𝑏))
or

Tesfaye G and Wanaw B Page 4


CALCULUS II

EXAMPLE 1: Finding a Value of the Inverse Derivative


𝑑𝑓 −1⁄
Let𝑓(𝑥) = 𝑥 3 − 2. Find the value of 𝑑𝑥 at 𝑥 = 6 = 𝑓(2) without finding a formula
For 𝑓 −1 (𝑥).

𝑑𝑓
Solution: | = 3𝑥 2 |𝑥=2 = 2
𝑑𝑥 𝑥=2
𝑑𝑓 −1 1 1
| = 𝑑𝑓 = 12
𝑑𝑥 𝑥=𝑓(𝑥) |
𝑑𝑥 𝑥=2

−1
Exercise 1.2 Find f −1 (x) and df ⁄dx at x = f(a) for the following functions

1. 𝑓(𝑥) = 2𝑥 + 3 , 𝑎 = −1
1
2. 𝑓(𝑥) = 5 − 4𝑥 , 𝑎 = 2
𝑥
3. 𝑓(𝑥) = 5 + 7 , 𝑎 = −1
4. 𝑓(𝑥) = 2𝑥 , 𝑥 ≥ 0 , 𝑎 = 5

Tesfaye G and Wanaw B Page 5


CALCULUS II

1.3 Inverse of Trigonometric Function and Their Derivatives

Tesfaye G and Wanaw B Page 6


CALCULUS II

Tesfaye G and Wanaw B Page 7


CALCULUS II

Tesfaye G and Wanaw B Page 8


CALCULUS II

Tesfaye G and Wanaw B Page 9


CALCULUS II

Tesfaye G and Wanaw B Page 10


CALCULUS II

(b) by using product rule we have the following

Exercise 1.3 Find the derivative of the following functions

1. 𝑦 = 𝑡𝑎𝑛−1 √𝑥 6. ℎ(𝑡) = √1 − 𝑡 2 arcsin 𝑡


2. 𝑦 = 𝑠𝑖𝑛−1 (2𝑥 + 1) 7. 𝑔(𝑥) = 𝑥𝑠𝑖𝑛−1 𝑥 − √1 − 𝑥 2
3. 𝑓(𝑥) = (1 + 𝑥 2 ) arctan 𝑥 8. 𝑔(𝑥) = 𝑠𝑒𝑐 −1 (𝑒 2𝑥 )
4. 𝑓(𝑥) = √𝑡𝑎𝑛−1 𝑥 9. 𝑓(𝑥) = 𝑡𝑎𝑛−1 (𝑥 −
√1 + 𝑥 2 )
1 10. 𝑓(𝜃) = arctan(cos 𝜃)
5. 𝑓(𝑥) = 𝑐𝑜𝑡 −1 𝑥 + 𝑐𝑜𝑡 −1 ( )
𝑥

Tesfaye G and Wanaw B Page 11


CALCULUS II

1.4 Exponential and logarithmic functions


In this section we are going to look definition and properties of Exponential and logarithmic
functions. And also we will see the relation of this to functions.

Exponential functions: are the functions of the form 𝑓(𝑥) = 𝑏 𝑥 where the base 𝑏 is a
positive constant and b ≠ 1.Some examples are

1 𝑥
𝑓(𝑥) = 2𝑥 , 𝑓(𝑥) = (2) and 𝑓(𝑥) = 𝜋 𝑥

Note that an exponential functions has a constant base and variable exponent. Thus, functions
such as 𝑓(𝑥) = 𝑥 2 and 𝑓(𝑥) = 𝜋 2 would not be classified as exponential functions. Since they
have a variable base and a constant exponent. Functions of this type, which are called power
functions.

Remark: If 𝑏 = 1 , then the function 𝑏 𝑥 is constant, since 𝑏 𝑥 = 1𝑥 = 1 . This case is of


no interest to us here. So we have excluded it from the family of exponential functions.

Logarithmic functions: If 𝑏 > 0 and 𝑏 ≠ 1 ,then for positive values of 𝑥 the logarithm to the
base 𝑏 of 𝑥 is denoted by log 𝑏 𝑥 , and is defined to be that exponent to
which must be raised to produce .
If 𝑎 > 0 and 𝑎 ≠ 1, the exponential function𝑓(𝑥) = 𝑎 𝑥 is either increasing or decreasing and
so it is one-to-one by the Horizontal Line Test. It therefore has an inverse function, which is
called the logarithmic function with base a and is denoted by log 𝑎 . If we use the formulation
of an inverse function 𝑓 −1 (𝑥) = 𝑦 ⇔ 𝑓(𝑦) ,then we have log 𝑎 𝑥 = 𝑦 ⇔ 𝑎 𝑦 = 𝑥
Thus, if 𝑥 > 0 , then log 𝑎 𝑥 is the exponent to which the base must be raised to give . For
example, log10 0.001 because 10−3 = 0.001 .

The logarithmic function has log 𝑎 domain (0, ∞) and range 𝑅. Its graph is the reflection of
the graph of 𝑦 = 𝑎 𝑥 about the line 𝑦 = 𝑥 .
The following properties of logarithmic functions follow from the corresponding properties of
exponential functions.

LAWS OF LOGARITHMS: If x and y are positive numbers, then


1. log 𝑎 𝑥𝑦 = log 𝑎 𝑦 + log 𝑎 𝑦
𝑥
2. log 𝑎 (𝑦) = log 𝑎 𝑥 − log 𝑎 𝑦
3. log 𝑎 (𝑥 𝑟 ) = 𝑟log 𝑎 𝑥 (where 𝑟 is any real number )

Example 1 Use the laws of logarithms to evaluate log 2 80 − log 2 5


SOLUTION: Using Law 2, we have

Tesfaye G and Wanaw B Page 12


CALCULUS II

80
log 2 80 − log 2 5 = log2 ( ) = log2 16 = 4
5
Because 24 = 16

The logarithm with base 𝑒 is called the natural logarithm and has a special notation:
log 𝑒 𝑥 = 𝑙𝑛𝑥

If we put and 𝑎 = 𝑒 replace log 𝑒 with “𝑙𝑛” in the equation log 𝑎 𝑥 = 𝑦 ⇔ 𝑎 𝑦 = 𝑥 then the
defining properties of the natural logarithm function become 𝑙𝑛𝑥 = 𝑦 ⇔ 𝑒 𝑦 = 𝑥

Example 2 Find 𝑥 if 𝑙𝑛𝑥 = 5

SOLUTION: From the property 𝑙𝑛𝑥 = 𝑦 ⇔ 𝑒 𝑦 = 𝑥 we have that


𝑙𝑛𝑥 = 5 means 𝑒 5 = 𝑥
Therefore 𝑥 = 𝑒 5 .

Example 3 Solve the equation 𝑒 5−3𝑥 = 10

SOLUTION: We take natural logarithms of both sides of the equation


𝑙𝑛(𝑒 5−3𝑥 ) = 𝑙𝑛10
5 − 3𝑥 = 𝑙𝑛10
3𝑥 = 5 − 𝑙𝑛10
1
𝑥= (5 − 𝑙𝑛10)
3

1
Example 4 Express 𝑙𝑛𝑎 + 2 𝑙𝑛𝑏 as a single logarithm.
1
1
SOLUTION: 𝑙𝑛𝑎 + 2 𝑙𝑛𝑏 = 𝑙𝑛𝑎 + 𝑙𝑛𝑏 2
= 𝑙𝑛𝑎 + 𝑙𝑛√𝑏
= 𝑙𝑛(𝑎√𝑏)
Theorem:( Change of base formula) For any positive number 𝑎(𝑎 ≠ 1), we have
𝑙𝑛𝑥
log 𝑎 𝑥 =
𝑙𝑛𝑎

Proof: Lety = log 𝑎 𝑥 . Then, we have 𝑎 𝑦 = 𝑥 . Taking natural logarithms of both sides of this
equation, we get 𝑦𝑙𝑛𝑎 = 𝑙𝑛𝑎. Therefore
𝑙𝑛𝑥
𝑦=
𝑙𝑛𝑎

Tesfaye G and Wanaw B Page 13


CALCULUS II

Example 5 Evaluate correct to six decimal places.


SOLUTION: From the above theorem we have
𝑙𝑛5
log 8 5 = ≈ 0.773976
𝑙𝑛8
Exercise 1.4

1. Solve each equation for 𝑥


a) 2𝑙𝑛𝑥 = 1
b) 𝑒 −𝑥 = 5
c) ln(5 − 2𝑥) = −3
d) 2𝑥−5 = 3
e) 𝑙𝑛𝑥 + ln(𝑥 − 1) = 1
2. Express the given quality as single logarithm.
a) 𝑙𝑛5 + 5𝑙𝑛3
b) ln(𝑎 + 𝑏) + ln(𝑎 − 𝑏) − 2𝑙𝑛𝑐
1
c) ln(1 + 𝑥 2 ) + 2 𝑙𝑛𝑥 − ln(𝑠𝑖𝑛𝑥)

Tesfaye G and Wanaw B Page 14


CALCULUS II

1.5 Exponential growth and decay

In many natural phenomena, quantities grow or decay at a rate proportional to their size.
For instance, if 𝑦 = 𝑓(𝑡) is the number of individuals in a population of animals or bacteria at
time t, then it seems reasonable to expect that the rate of growth 𝑓 ′ (𝑡) is proportional to the
population 𝑓(𝑡); that is, 𝑓 ′ (𝑡) = 𝑘𝑓(𝑡) for some constant k . Indeed, under ideal conditions
(unlimited environment, adequate nutrition, and immunity to disease) the mathematical model
given by the equation 𝑓 ′ (𝑡) = 𝑘𝑓(𝑡) predicts what actually happens fairly accurately. Another
example occurs in nuclear physics where the mass of a radioactive substance decays at a rate
proportional to the mass. In chemistry, the rate of a unimolecular first-order reaction is
proportional to the concentration of the substance.
In general, if 𝑦(𝑡) is the value of a quantity at time and if the rate of change of with respect to
is proportional to its size 𝑦(𝑡) at any time, then
𝑑𝑦
= 𝑘𝑦 Where is k a constant.
𝑑𝑡
The above equation is sometimes called the law of natural growth ( 𝑖𝑓 𝑘 > 0 ) or the law of
natural decay (𝑖𝑓 𝑘 < 0 ). It is called a differential equation because it involves an unknown
𝑑𝑦
function 𝑦 and its derivative .
𝑑𝑡
Any exponential function of the form 𝑦(𝑡) = 𝐶𝑒 𝑘𝑡 , where is C a constant, satisfies

𝑦 , (𝑡) = 𝐶(𝐾𝑒 𝑘𝑡 ) = 𝐾(𝐶𝑒 𝑘𝑡 ) = 𝐾𝑌(𝑡)

𝑑𝑦
Any function that satisfies = 𝑘𝑦 must be of the form 𝑦 = 𝐶𝑒 𝑘𝑡 .To see the significance of
𝑑𝑡
the constant C, we observe that

𝑦(0) = 𝐶𝑒 𝑘0 = 𝐶

Therefore C is the initial value of the function

𝑑𝑦
THEOREM: The only solutions of the differential equation = 𝑘𝑦 are the
𝑑𝑡
exponential functions 𝑦(𝑡) = 𝑦(0)𝑒 𝑘𝑡

Tesfaye G and Wanaw B Page 15


CALCULUS II

Population growth
What is the significance of the proportionality constant k? In the context of population
growth, where 𝑝(𝑡) is the size of a population at time, we can write
𝑑𝑝 1 𝑑𝑝 1 𝑑𝑝
= 𝑘𝑝 Or = 𝑘 ; the quantity is the growth rate
𝑑𝑡 𝑝 𝑑𝑡 𝑝 𝑑𝑡
divided by the population size; it is called the relative growth rate.

EXAMPLE 1 Use the fact that the world population was 2560 million in 1950 and 3040 million
in 1960 to model the population of the world in the second half of the 20th century. (Assume that
the growth rate is proportional to the population size.) What is the relative growth rate? Use the
model to estimate the world population in 1993 and to predict the population in the year 2020.

𝑑𝑝
are assuming that = 𝑘𝑝, the theorem gives
𝑑𝑡

The graph in the following Figure shows that the model is fairly accurate to the
end of the 20th century (the dots represent the actual population), so the
estimate for 1993 is quite reliable. But the prediction for 2020 is riskier.

Tesfaye G and Wanaw B Page 16


CALCULUS II

Radioactive decay
Radioactive substances decay by spontaneously emitting radiation. If 𝑚(𝑡) is the mass
remaining from an initial mass 𝑚0 of the substance after time t, then the relative decay rate
1 𝑑𝑚 𝑑𝑚
− has been found experimentally to be constant. (Since is negative, the relative
𝑚 𝑑𝑡 𝑑𝑡
𝑑𝑚
decay rate is positive.) It follows that = 𝑘𝑚 , where k is a negative constant. In other
𝑑𝑡
words, radioactive substances decay at a rate proportional to the remaining mass. This means that
we can use the above theorem to show that the mass decays exponentially:
𝑚(𝑡) = 𝑚 𝑒 𝑘𝑡
0 Physicists express the rate of
decay in terms of half-life, the time required for half of any given quantity to decay.

EXAMPLE 2 The half-life of radium-226 is 1590 years.


(a) A sample of radium-226 has a mass of 100 mg. Find a formula for the mass of the
sample that remains after years.
(b) Find the mass after 1000 years correct to the nearest milligram.
(c) When will the mass be reduced to 30 mg?

SOLUTION (a) let 𝑚(𝑡) be the mass of radium-226 (in milligrams) that remains
𝑑𝑚
after 𝑡 years. Then 𝑑𝑡 = 𝑘𝑚 and 𝑦(0) = 100,so the theorem gives

Tesfaye G and Wanaw B Page 17


CALCULUS II

Newton’s low of cooling

Newton’s Law of Cooling states that the rate of cooling of an object is proportional to the
temperature difference between the object and its surroundings, provided that this difference is
not too large. (This law also applies to warming.) If we let 𝑇(𝑡) be the temperature of the object
at time 𝑡 and 𝑇𝑠 be the temperature of the surroundings, then we can formulate Newton’s Law of
Cooling as a differential equation:
𝑑𝑇
= 𝑘(𝑇 − 𝑇𝑠 )
𝑑𝑡
𝑑𝑦
Where 𝑘 is a constant. This equation is not quite the same as the equation = 𝑘𝑦 above so we
𝑑𝑡
make the change of variable 𝑦(𝑡) = 𝑇(𝑡) − 𝑇𝑠 . Because 𝑇𝑠 is constant, we have 𝑦 , (𝑡) = 𝑇 ′ (𝑡)
and so the equation becomes
𝑑𝑦
= 𝑘𝑦
𝑑𝑡
We can then use the theorem to find an expression for , from which we can find 𝑇 .

EXAMPLE 3 A bottle of soda pop at room temperature (72℉) is placed in a refrigerator


where the temperature is 44℉. After half an hour the soda pop has cooled to 61℉.
(a) What is the temperature of the soda pop after another half hour?
(b) How long does it take for the soda pop to cool to 50℉?

By the theorem we have

Tesfaye G and Wanaw B Page 18


CALCULUS II

Tesfaye G and Wanaw B Page 19


CALCULUS II

Exercise 1.5

1. A population of protozoa develops with a constant relative growth rate of


0.7944 per member per day. On day zero the population consists of two
members. Find the population size after six days.
2. The table gives estimates of the world population, in millions, from 1750
to 2000:

(a) Use the exponential model for1750 and 1800 to predict the world
population in 1900and 1950. Compare with the actual figures.
(b) Use the exponential model for1850 and 1900 to predict the world
population in 1950.Compare with the actual population.
3. The half-life of cesium-137 is 30 years. Suppose we have a100-mg
sample.
a) Find the mass that remains after years.
b) How much of the sample remains after 100 years?
c) After how long will only 1 mg remain?
4. A freshly brewed cup of coffee has temperature 95℃ in a 20℃ room.
When its temperature is 70℃, it is cooling at a rate of 1℃ per minute.
When does this occur?

Tesfaye G and Wanaw B Page 20


CALCULUS II

1.6 Inverse of Hyperbolic functions and their Derivatives

Tesfaye G and Wanaw B Page 21


CALCULUS II

Tesfaye G and Wanaw B Page 22


CALCULUS II

Inverse of hyperbolic function:


You can see from the graph of sinh and tanh, that they are one-to-one functions
and so they have inverse functions denoted by 𝑠𝑖𝑛ℎ−1 𝑎𝑛𝑑 𝑡𝑎𝑛ℎ−1 . The graph of
cosh shows that cosh is not one-to-one, but when restricted to the domain [0, ∞) it
becomes one-to-one. The inverse hyperbolic cosine function is defined as the
inverse of this restricted function.

We can sketch the graph of 𝑠𝑖𝑛ℎ−1 , 𝑐𝑜𝑠ℎ−1 𝑎𝑛𝑑 𝑡𝑎𝑛ℎ−1 as follow

The hyperbolic functions are defined in terms of exponential functions, so that


the inverse hyperbolic functions can be expressed in terms of logarithms as we can
see the theorem below.

Theorem:

Tesfaye G and Wanaw B Page 23


CALCULUS II

The proofs of the formulas 2-4 are left as exercise.

The inverse hyperbolic functions are all differentiable because the hyperbolic functions are
differentiable. The formulas in theorem below can be proved by the method for inverse functions
or by differentiating the formulas in theorem.

Theorem:

Observe that we could have done the proof (1) by using formula (1) of the above Theorem as
below.

Tesfaye G and Wanaw B Page 24


CALCULUS II

Example 2

Solution: by using the theorem and chain rule, we have

Example 3

Solution: by using the theorem and chain rule, we have

Exercise 1.6 Find the derivative of the following functions

1. 𝑦 = 𝑠𝑖𝑛ℎ−1 5𝑥
2. 𝑦 = 𝑥𝑡𝑎𝑛ℎ−1 𝑥 + 𝑙𝑛√1 − 𝑥 2
3. 𝑓(𝑥) = √𝑐𝑜𝑠ℎ−1 𝑥
4. 𝑓(𝑥) = 𝑠𝑒𝑐ℎ−1 √1 − 𝑥 2 ,𝑥 > 0
5. 𝑔(𝑥) = 𝑥𝑡𝑎𝑛ℎ−1 𝑥 + 𝑙𝑛√1 − 𝑥 2
𝑥
6. 𝑓(𝑥) = 𝑥𝑠𝑖𝑛ℎ−1 ( )
3

7. 𝑦 = 𝑠𝑒𝑐ℎ−1 √1 − 𝑥 2 , 𝑥 > 0
8. 𝑔(𝑥) = 𝑐𝑜𝑡ℎ−1 √𝑥 2 + 1

Tesfaye G and Wanaw B Page 25


CALCULUS II

CHAPTER-TWO:
TECHNIQES OF INTEGRATION

Objectives:
By the end of this chapter, students will be able to:
 Evaluate indefinite integrals using integration by substitution
 Find indefinite integrals using integration by parts & by partial fraction
 Determine the value of trigonometric integrals
 Apply integration by trigonometric substitution to calculate integrals
 Apply Trapezoidal and Simpson’s rule to calculate integrals
 Apply integration to find area
 Apply integration to find volume

Over view: In this chapter, we are going to discuss Integration by substitution, by parts and by
partial fraction, Trigonometric integrals, Integration by trigonometric substitution. That is we
develop techniques for using the basic integration formulas
To obtain indefinite integrals of more complicated functions. Integration is not as
Straightforward as differentiation; there are no rules that absolutely guarantee obtaining an
indefinite integral of a function. Therefore we discuss a strategy for integration. And also we will
see some applications of integration such as: Area of a region bounded by curves of continuous

defined on a closed interval [a, b] and volume of a solid of revolutions.

2.1 Elementary Integration Formulas


To help us in the search for finding indefinite integrals, it is useful to build up a table of integral
formulas by inverting formulas for derivatives, as we have done in previous chapters. Then we
try to match any integral that confronts us against one of the standard types. This usually
involves a certain amount of algebraic manipulation as well as use of the Substitution Rule.
Recall the Substitution Rule ∫ 𝑔(𝑓(𝑥)) ∙ 𝑓 ′ (𝑥) 𝑑𝑥 = ∫ 𝑔(𝑢)𝑑𝑢

Where 𝑢 = 𝑓(𝑥) is a differentiable function whose range is an interval I and ƒ is continuous


on I. Success in integration often hinges on the ability to spot what part of the integrand should
be called u in order that one will also have du, so that a known formula can be applied. This
means that the first requirement for skill in integration is a thorough mastery of the formulas for
differentiation.

Tesfaye G and Wanaw B Page 26


CALCULUS II

The following table is shows the basic forms of integrals we have evaluated so far. In this section
we present several algebraic or substitution methods to help us use this table.

1. ∫ 𝑑𝑢 = 𝑢 + 𝑐 11.∫ tan 𝑢 𝑑𝑢 = − ln|𝑢| + 𝑐


= ln | sec 𝑢| + 𝑐
2. ∫ k du = ku + c (any number k) 12. ∫ cot 𝑢 𝑑𝑢 = ln | sin 𝑢| + 𝑐
= −ln | csc 𝑢| + 𝑐
3. ∫ 𝑢𝑛 𝑑𝑢 =
𝑢𝑛+1
+𝑐 (𝑛 ≠ −1) 13.∫ eu du = eu + c
𝑛+1

𝑑𝑢 𝑎𝑢
4. ∫ 𝑢
= ln|𝑢| + 𝑐 14.∫ 𝑎u du = ln 𝑢 + 𝑐

5. ∫ 𝑠𝑖𝑛 𝑢𝑑𝑢 = −𝑐𝑜𝑠 𝑢 + 𝑐 15.∫ sinh u du = cosh u + c

6. ∫ 𝑐𝑜𝑠 𝑢𝑑𝑢 = 𝑠𝑖𝑛 𝑢 + 𝑐 16.∫ cosh u du = sinh u + c

7. ∫ 𝑠𝑒𝑐 2 𝑢𝑑𝑢 = 𝑡𝑎𝑛 𝑢 + 𝑐 du


17. ∫ √𝑎2
u
= sin−1 (𝑎) + c
−u2

8. ∫ 𝑐𝑠𝑐 2 𝑐 𝑑𝑢 = − cot 𝑢 + 𝑐 du 1 𝑢
18. ∫ 𝑎2 +u2 = 𝑎 𝑡𝑎𝑛−1 (𝑎) + 𝑐

9. ∫ sec 𝑢 𝑡𝑎𝑛𝑢 𝑑𝑢 = sec 𝑢 + 𝑐

10.∫ csc 𝑢 cot 𝑢 𝑑𝑢 = − csc 𝑢 + 𝑐

2.2- Integration by parts

Every differentiation rule has a corresponding integration rule. For instance, the
Substitution Rule for integration corresponds to the Chain Rule for differentiation. The
rule that corresponds to the Product Rule for differentiation is called the rule for
integration by parts.
Recall that the Product Rule states that if 𝑓 and 𝑔 are differentiable functions, then
𝑑
[𝑓(𝑥)𝑔(𝑥)] = 𝑓(𝑥)𝑔′ (𝑥) + 𝑔(𝑥)𝑓 ′ (𝑥)
𝑑𝑥

In the notation for indefinite integrals this equation becomes


∫(𝑓(𝑥)𝑔′ (𝑥) + 𝑔(𝑥)𝑓 ′ (𝑥))𝑑𝑥 = 𝑓(𝑥)𝑔(𝑥)

Tesfaye G and Wanaw B Page 27


CALCULUS II

or equivalently,
∫(𝑓(𝑥)𝑔′ (𝑥)) 𝑑𝑥 + ∫(𝑓′(𝑥)𝑔(𝑥)) 𝑑𝑥 = 𝑓(𝑥)𝑔(𝑥)
Rearranging the latter equation we get
∫(𝑓(𝑥)𝑔′ (𝑥)) 𝑑𝑥 = 𝑓(𝑥)𝑔(𝑥) − ∫(𝑓′(𝑥)𝑔(𝑥)) 𝑑𝑥 (1)
Formula (1) is called the formula for integration by parts.
Letting 𝑢 = 𝑓(𝑥) and 𝑣 = 𝑔(𝑥) then the differentials become 𝑑𝑢 = 𝑓 ′ (𝑥)𝑑𝑥 and
𝑑𝑣 = 𝑔′ (𝑥)𝑑𝑥 and formula (1) becomes
∫ 𝑢 𝑑𝑣 = 𝑢𝑣 − ∫ 𝑣 𝑑𝑢

Example 1: Find ∫ 𝑥 𝑐𝑜𝑠 𝑥 𝑑𝑥


Solution: Let
𝑢 = 𝑥 and 𝑑𝑣 = cos 𝑥 𝑑𝑥
Then 𝑑𝑢 = 𝑑𝑥 and 𝑣 = sin 𝑥
Therefore, integration by parts yields
∫ 𝑥 cos 𝑥 𝑑𝑥 = 𝑥 sin 𝑥 − ∫ 𝑠𝑖𝑛 𝑥𝑑𝑥 = 𝑥 sin 𝑥 + cos 𝑥 + 𝐶

Example 2: Find ∫ ln 𝑥 𝑑𝑥
Solution: Here the integrand can be written as ln 𝑥 = (ln 𝑥) ∙ 1.
Now let 𝑢 = ln 𝑥 and 𝑑𝑣 = 1 𝑑𝑥
1
Then 𝑑𝑢 = 𝑑𝑥 and 𝑣=𝑥
𝑥

Therefore
1
∫ ln 𝑥 𝑑𝑥 = 𝑥 ln 𝑥 − ∫ 𝑥 ∙ 𝑥 𝑑𝑥 = 𝑥 ln 𝑥 − ∫ 𝑑𝑥 = 𝑥 ln 𝑥 − 𝑥 + 𝐶 ∎

Example 3: Find ∫ 𝑥 2 𝑒 𝑥 𝑑𝑥
Solution: Notice that 𝑥 2 becomes simpler when differentiated (whereas 𝑒 𝑥 remains
unchanged when differentiated or integrated), so we choose
𝑢 = 𝑥2 and 𝑑𝑣 = 𝑒 𝑥 𝑑𝑥
Then 𝑑𝑢 = 2𝑥 𝑑𝑥 and 𝑣 = 𝑒𝑥
Therefore, integration by parts gives
∫ 𝑥 2 𝑒 𝑥 𝑑𝑥 = 𝑥 2 𝑒 𝑥 − ∫ 𝑒 𝑥 (2𝑥) 𝑑𝑥 = 𝑥 2 𝑒 𝑥 − 2 ∫ 𝑥𝑒 𝑥 𝑑𝑥 (i)

Tesfaye G and Wanaw B Page 28


CALCULUS II

In the last integral of (1), let


𝑢=𝑥 and 𝑑𝑣 = 𝑒 𝑥 𝑑𝑥
Then 𝑑𝑢 = 𝑑𝑥 and 𝑣 = 𝑒 𝑥
Thus applying integration by parts for the second time, we get
∫ 𝑥𝑒 𝑥 𝑑𝑥 = 𝑥𝑒 𝑥 − ∫ 𝑒 𝑥 𝑑𝑥 = 𝑥𝑒 𝑥 − 𝑒 𝑥 + 𝐾 where K is any arbitrary
integration constant.
So, 2 ∫ 𝑥𝑒 𝑥 𝑑𝑥 = 2𝑥𝑒 𝑥 − 2 ∫ 𝑒 𝑥 𝑑𝑥 = 2𝑥𝑒 𝑥 − 2𝑒 𝑥 + 𝐶, (C = 2K) (ii)
Therefore, substituting (ii) in (i) we get
∫ 𝑥 2 𝑒 𝑥 𝑑𝑥 = 𝑥 2 𝑒 𝑥 − ∫ 𝑒 𝑥 (2𝑥) 𝑑𝑥 = 𝑥 2 𝑒 𝑥 − 2 ∫ 𝑥𝑒 𝑥 𝑑𝑥
= 𝑥 2 𝑒 𝑥 − (2𝑥𝑒 𝑥 − 2 ∫ 𝑒 𝑥 𝑑𝑥) = 𝑥 2 𝑒 𝑥 − 2𝑥𝑒 𝑥 + 2𝑒 𝑥 − 𝐶

Example 4: Find ∫ 𝑒 𝑥 cos 𝑥 𝑑𝑥


Solution: Let 𝑢 = 𝑒 𝑥 and 𝑑𝑣 = cos 𝑥 𝑑𝑥
Then 𝑑𝑢 = 𝑒 𝑥 𝑑𝑥 and 𝑣 = sin 𝑥
Thus applying integration by parts we get
∫ 𝑒 𝑥 cos 𝑥 𝑑𝑥 = 𝑒 𝑥 sin 𝑥 − ∫ 𝑒 𝑥 sin 𝑥 𝑑𝑥 (*)
In the second integral of (*) let
𝑢 = 𝑒𝑥 and 𝑑𝑣 = sin 𝑥 𝑑𝑥
Then 𝑑𝑢 = 𝑒 𝑥 𝑑𝑥 and 𝑣 = − cos 𝑥
Now,
∫ 𝑒 𝑥 sin 𝑥 𝑑𝑥 = −𝑒 𝑥 cos 𝑥 − ∫ 𝑒 𝑥 (− cos 𝑥)𝑑𝑥
= −𝑒 𝑥 cos 𝑥 + ∫ 𝑒 𝑥 cos 𝑥 𝑑𝑥 (**)
Therefore, substituting (**) in (*) we get
∫ 𝑒 𝑥 cos 𝑥 𝑑𝑥 = 𝑒 𝑥 sin 𝑥 − ∫ 𝑒 𝑥 sin 𝑥 𝑑𝑥
= 𝑒 𝑥 sin 𝑥 − (−𝑒 𝑥 cos 𝑥 + ∫ 𝑒 𝑥 cos 𝑥 𝑑𝑥)
= 𝑒 𝑥 sin 𝑥 + 𝑒 𝑥 cos 𝑥 − ∫ 𝑒 𝑥 cos 𝑥 𝑑𝑥 + 𝐶 (***)
which is still incomplete and if we continuing to apply integration by parts will be less important.
But rearranging the terms in (***) we have
2 ∫ 𝑒 𝑥 cos 𝑥 𝑑𝑥 = 𝑒 𝑥 sin 𝑥 + 𝑒 𝑥 cos 𝑥 + 𝐶
which implies

Tesfaye G and Wanaw B Page 29


CALCULUS II

1
∫ 𝑒 𝑥 cos 𝑥 𝑑𝑥 = 2 (𝑒 𝑥 sin 𝑥 + 𝑒 𝑥 cos 𝑥 + 𝐶)

Example 6 : A Reduction Formula


Obtain a reduction formula that expresses the integral ∫ 𝑐𝑜𝑠 𝑛 𝑥𝑑𝑥 interms of an integral of
alower power of 𝑐𝑜𝑠𝑥.
SOLUTION: Notice that 𝑐𝑜𝑠 𝑛 𝑥 = (𝑐𝑜𝑠 𝑛−1 𝑥)(cos 𝑥), and let
𝑢 = 𝑐𝑜𝑠 𝑛−1 𝑥 and 𝑑𝑣 = cos 𝑥 𝑑𝑥
Then 𝑑𝑢 = (𝑛 − 1)𝑐𝑜𝑠 𝑛−2 𝑥 (−sin 𝑥) 𝑑𝑥 and 𝑣 = sin 𝑥
Now,
∫ 𝑐𝑜𝑠 𝑛 𝑥 𝑑𝑥 = 𝑐𝑜𝑠 𝑛−1 𝑥 sin 𝑥 − ∫(sin 𝑥) (𝑛 − 1)𝑐𝑜𝑠 𝑛−2 𝑥 (−sin 𝑥) 𝑑𝑥
= 𝑐𝑜𝑠 𝑛−1 𝑥 sin 𝑥 + ∫ 𝑠𝑖𝑛2 𝑥(𝑛 − 1)𝑐𝑜𝑠 𝑛−2 𝑥 𝑑𝑥
= 𝑐𝑜𝑠 𝑛−1 𝑥 sin 𝑥 + (𝑛 − 1) ∫ 𝑐𝑜𝑠 𝑛−2 𝑥 (1 − 𝑐𝑜𝑠 2 𝑥) 𝑑𝑥
= 𝑐𝑜𝑠 𝑛−1 𝑥 sin 𝑥 + (𝑛 − 1)[∫ 𝑐𝑜𝑠 𝑛−2 𝑥 𝑑𝑥 − ∫ 𝑐𝑜𝑠 𝑛 𝑥 𝑑𝑥]
= 𝑐𝑜𝑠 𝑛−1 𝑥 sin 𝑥 + (𝑛 − 1) ∫ 𝑐𝑜𝑠 𝑛−2 𝑥 𝑑𝑥 − (𝑛 − 1) ∫ 𝑐𝑜𝑠 𝑛 𝑥 𝑑𝑥
Combining the terms that contain ∫ 𝑐𝑜𝑠 𝑛 𝑥 𝑑𝑥, we have
∫ 𝑐𝑜𝑠 𝑛 𝑥 𝑑𝑥 + (𝑛 − 1) ∫ 𝑐𝑜𝑠 𝑛 𝑥 𝑑𝑥 = 𝑛 ∫ 𝑐𝑜𝑠 𝑛 𝑥 𝑑𝑥
= 𝑐𝑜𝑠 𝑛−1 𝑥 sin 𝑥 + (𝑛 − 1) ∫ 𝑐𝑜𝑠 𝑛−2 𝑥 𝑑𝑥

Example 7 : Using a reduction formula evaluate

Solution: From the result in example 6

Tesfaye G and Wanaw B Page 30


CALCULUS II

Exercise 2.2 Evalualte the following integral using integration by parts

Tesfaye G and Wanaw B Page 31


CALCULUS II

2.3- Integration by trigonometric substitution

In this section we will discuss a method for evaluating integrals containing radicals by making
substitution involving quadratic polynomials can sometimes be evaluated by completing the
square.

The functions in these substitutions have inverses only for selected values of  . For
reversibility,

Tesfaye G and Wanaw B Page 32


CALCULUS II

TABELE OF TRIGONOMETRIC SUBSTITUTION

Expression in integrand Substitution


−𝜋 𝜋
√𝑎2 − 𝑥 2 𝑥 = 𝑎 sin 𝑢, 𝑤𝑖𝑡ℎ ≤𝑢≤
2 2

𝑑𝑥 = 𝑎 cos 𝑢 𝑑𝑢
−𝜋 𝜋
√𝑎2 + 𝑥 2 𝑥 = 𝑎 tan 𝑢, 𝑤𝑖𝑡ℎ 2
<𝑢< 2
2
𝑑𝑥 = 𝑎 sec 𝑢 𝑑𝑢
𝜋 3𝜋
√𝑥 2 − 𝑎2 𝑥 = 𝑎 sec 𝑢, 𝑤𝑖𝑡ℎ 0 ≤ 𝑢 < or 𝜋 ≤ 𝑢 <
2 2

𝑑𝑥 = 𝑎 sec 𝑢 tan 𝑢 𝑑𝑢

1
Example 1: Find ∫ 𝑑𝑥
𝑥 2 √9−𝑥 2

Solution: Since √9 − 𝑥 2 = √32 − 𝑥 2 , we substitute


𝑥 = 3 sin 𝑢, so that 𝑑𝑥 = 3 cos 𝑢 𝑑𝑢
Thus,
1 3 cos 𝑢 𝑑𝑢
∫ 𝑥 2 √9−𝑥 2 𝑑𝑥 = ∫ 9𝑠𝑖𝑛2 𝑢√9−9𝑠𝑖𝑛2
3 cos 𝑢 𝑑𝑢
=∫
(9𝑠𝑖𝑛2 𝑢)3√1−𝑠𝑖𝑛2
1 cos 𝑢 𝑑𝑢
= 9∫
(𝑠𝑖𝑛2 𝑢)√1−𝑠𝑖𝑛2
1 cos 𝑢 𝑑𝑢
= 9 ∫ (𝑠𝑖𝑛2 𝑢) cos 𝑢
1 𝑑𝑢 1
= 9 ∫ 𝑠𝑖𝑛2 𝑢 = 9 ∫ 𝑐𝑠𝑐 2 𝑢 𝑑𝑢
1
= − 9 cot 𝑢 + 𝐶

In order to write the answer in terms of the original variable x, we draw the triangle in the
following figure From this we see that
√16−𝑥 2
cot 𝑢 = 𝑥

Therefore,
1 1 √16−𝑥2
𝑥 𝑥 3 ∫ 𝑥 2 √9−𝑥2 𝑑𝑥 = − 16 +𝐶 ∎
𝑥

√9 − 𝑥 2

Tesfaye G and Wanaw B Page 33


CALCULUS II

1
Example 2: Evaluate the integral ∫02 √1 − 4𝑥 2 𝑑𝑥

Solution: Since √1 − 4𝑥 2 = √12 − (2𝑥)2 , we are led to substitute


cos 𝑢
2𝑥 = sin 𝑢, so that 𝑑𝑥 = 𝑑𝑢
2

For the limits of integration we notice that


1 𝜋
if 𝑥 = 0, then 𝑢 = 0, and if 𝑥 = 2, then 𝑢 = 2

Therefore,
1 𝜋
cos 𝑢
∫0 √1 − 4𝑥 2 𝑑𝑥 = ∫02 √1 − 𝑠𝑖𝑛2 𝑢 (
2
2
) 𝑑𝑢
𝜋
cos 𝑢
= ∫02 cos 𝑢 ( ) 𝑑𝑢
2
𝜋
1
= 2 ∫02 𝑐𝑜𝑠 2 𝑢 𝑑𝑢
1 1 𝜋 1 𝜋 1 1 1
= 2 (4 sin 2 (2 ) + 2 ∙ 2 ) − 2 (4 sin 2(0) + 2 ∙ 0)
𝜋
= ∎
8

Example 3: Find ∫ √𝑥 2 − 9 𝑑𝑥
Solution: Since √𝑥 2 − 9 = √𝑥 2 − 32 , we let
𝑥 = 3 sec 𝑢 then 𝑑𝑥 = 3 sec 𝑢 tan 𝑢 𝑑𝑢
Thus,
∫ √𝑥 2 − 9 𝑑𝑥 = ∫ √9𝑠𝑒𝑐 2 𝑢 − 9 (3 sec 𝑢 tan 𝑢)𝑑𝑢
= 9 ∫ √𝑠𝑒𝑐 2 𝑢 − 1 sec 𝑢 tan 𝑢 𝑑𝑢
= ∫ 𝑡𝑎𝑛2 𝑢 sec 𝑢 𝑑𝑢
= ∫(𝑠𝑒𝑐 2 𝑢 − 1) sec 𝑢 𝑑𝑢
= ∫ 𝑠𝑒𝑐 3 𝑢 𝑑𝑢 − ∫ sec 𝑢 𝑑𝑢
1
= 2 (sec 𝑢 𝑡𝑎𝑛 𝑢 + ln|sec 𝑢 + tan 𝑢|) − ln|sec 𝑢 + tan 𝑢| + 𝐶
1 1
= 2 (sec 𝑢 𝑡𝑎𝑛 𝑢) − 2 (ln|sec 𝑢 + tan 𝑢|) + 𝐶

𝑥
√𝑥 2 − 9

Tesfaye G and Wanaw B Page 34


CALCULUS II

Integrals Containing √𝒃𝒙𝟐 + 𝒄𝒙 + 𝒅

By completing the square in 𝑏𝑥 2 + 𝑐𝑥 + 𝑑, we can express √𝑏𝑥 2 + 𝑐𝑥 + 𝑑 in terms of


√𝑎2 − 𝑥 2 , √𝑎2 + 𝑥 2 , 𝑜𝑟√𝑥 2 − 𝑎2 for suitable 𝑎 > 0. Then the trigonometric substitution
eliminates the square root as before.
𝑥 𝑑𝑥
Example 4: Find ∫ √3−2𝑥−𝑥 2

Solution: We can transform the integrand in to a form for which trigonometric substitutions
are appropriate. In this case we first complete the square under the square root.
3 − 2𝑥 − 𝑥 2 = 3 − (𝑥 2 + 2𝑥) = 3 + 1 − (𝑥 2 + 2𝑥 + 1)
= 4 − (𝑥 + 1)2
This suggests that we make the substitution 𝑢 = 𝑥 + 1. Then 𝑑𝑢 = 𝑑𝑥 and 𝑥 = 𝑢 − 1, so
𝑥 𝑢−1
∫ √3−2𝑥−𝑥 2 𝑑𝑥 = ∫ √4−𝑢2 𝑑𝑢

Now let 𝑢 = 2 sin 𝜃 which gives 𝑑𝑢 = 2 cos 𝜃 𝑑𝜃 and √4 − 𝑢2 = 2 cos 𝜃, and so


𝑥 2 sin 𝜃−1
∫ √3−2𝑥−𝑥 2 𝑑𝑥 = ∫ 2 cos 𝜃
2 cos 𝜃 𝑑𝜃

= ∫(2 sin 𝜃 − 1)𝑑𝜃


= −2 cos 𝜃 − 𝜃 + 𝐶
We can devise the relation between u and 𝜃 from the following Figure

𝑢
2
𝜃

√4 − 𝑢2

√4−𝑢2 𝑢
From this, we have cos 𝜃 = and 𝜃 = 𝑠𝑖𝑛−1 ( 2) and substituting these values we get
2
𝑥
∫ √3−2𝑥−𝑥 2 𝑑𝑥 = −2 cos 𝜃 − 𝜃 + 𝐶
√4−𝑢2 𝑢
= −2 − 𝑠𝑖𝑛−1 (2) + 𝐶
2
𝑥+1
= −√4 − (𝑥 + 1)2 − 𝑠𝑖𝑛−1 ( )+𝐶
2

Tesfaye G and Wanaw B Page 35


CALCULUS II

Exercise 2.3: Evaluate the following Integrals

Tesfaye G and Wanaw B Page 36


CALCULUS II

2.4- Integration by Partial fraction

Tesfaye G and Wanaw B Page 37


CALCULUS II

Tesfaye G and Wanaw B Page 38


CALCULUS II

Tesfaye G and Wanaw B Page 39


CALCULUS II

Tesfaye G and Wanaw B Page 40


CALCULUS II

Tesfaye G and Wanaw B Page 41


CALCULUS II

Tesfaye G and Wanaw B Page 42


CALCULUS II

Note: we can determine the constants that appear in partial fractions using different methods
such as differentiation method and assigning numerical Values to x
Example 6 : Find A, B, and C in the equation using differentiation method

Tesfaye G and Wanaw B Page 43


CALCULUS II

Example 7: Find A, B, and C in the equation using assigning numerical value for x

Tesfaye G and Wanaw B Page 44


CALCULUS II

Exercise 2.4: Evaluate the following Integrals using integration by partial fraction

Tesfaye G and Wanaw B Page 45


CALCULUS II

2.5- Trigonometric Integrals

In this subsection we use trigonometric identities to integrate certain combinations of


trigonometric functions. We start with powers of sine and cosine. Trigonometric integrals
involve algebraic combinations of the six basic trigonometric functions. In principle, we can
always express such integrals in terms of sines and cosines, but it is often simpler to work with

other functions, as in the integral


The general idea is to use identities to transform the integrals we have to find into integrals that
are easier to work with.

Tesfaye G and Wanaw B Page 46


CALCULUS II

Tesfaye G and Wanaw B Page 47


CALCULUS II

Tesfaye G and Wanaw B Page 48


CALCULUS II

Tesfaye G and Wanaw B Page 49


CALCULUS II

Tesfaye G and Wanaw B Page 50


CALCULUS II

EXAMPLE 4 Evalute ∫ 𝑡𝑎𝑛4 𝑥 𝑑𝑥

EXAMPLE 5 Evaluate ∫ sec 3 x dx

Tesfaye G and Wanaw B Page 51


CALCULUS II

6 4
EXAMPLE 6 Evaluate  tan x sec x dx

EXAMPLE 7 Evaluate  tan 5  sec7  d

Tesfaye G and Wanaw B Page 52


CALCULUS II

EXAMPLE 8 Evaluate

Exercise 2.5: Evaluate the following Integrals

Tesfaye G and Wanaw B Page 53


CALCULUS II

2.6 Trapezoidal and Simpson’s rule


The Fundamental Theorem of Calculus provided us with a highly efficient method for
calculating definite integrals. Still, for some functions we have no good expression for its anti-
derivative. In such cases we may have to rely on numerical methods for integrating. Let us take
such a function, and show some methods for finding an approximate value for the integral. We
describe different ways to find, by numerical means, approximate values for the integral of a
function on 𝑓(𝑥) over the interval [𝑎 , 𝑏]:

𝑏
∫𝑎 𝑓(𝑥)𝑑𝑥

In all of the different approaches we partition the interval into smaller ones:
𝑎 = 𝑥0 < 𝑥1 < ⋯ < 𝑥𝑛−1 < 𝑥𝑛 = 𝑏
Left and Right Endpoint Method: In the left endpoint method we find the value of the function
at each left endpoint of the intervals of the partition. We multiply it with the length of the
associated interval, and then add up the terms. Explicitly, we calculate

𝐼𝐿 = 𝑓(𝑥0 )(𝑥1 − 𝑥0 ) + 𝑓(𝑥1 )(𝑥2 − 𝑥1 ) + ⋯ + 𝑓(𝑥𝑛−1 )(𝑥𝑛 − 𝑥𝑛−1 )

In the right endpoint method we proceed as we did on the left endpoint method, only we use the
value of the function at the right endpoint instead of the left endpoint:

𝐼𝑅 = 𝑓(𝑥1 )(𝑥1 − 𝑥0 ) + 𝑓(𝑥2 )(𝑥2 − 𝑥1 ) + ⋯ + 𝑓(𝑥𝑛 )(𝑥𝑛 − 𝑥𝑛−1 )

EXAMPLE 1 Use the left and right endpoint method to find approximate values for

2 2
∫0 𝑒 −𝑥 𝑑𝑥
2 2
Solution: Set (𝑥) = ∫0 𝑒 −𝑥 𝑑𝑥. We partition the interval [0, 2] into 4 smaller intervals by
choosing three intermediate points:
1 3
𝑥0 = 0 < 𝑥1 = < 𝑥2 = 1 < 𝑥3 = < 𝑥4 = 2
2 2
1
Then 𝑥𝑘 − 𝑥𝑘−1 = 2 𝑓𝑜𝑟 𝑘 = 1, 2, 3, 𝑎𝑛𝑑 4
1 3
𝑓(0)+𝑓( )+𝑓(1)+𝑓( )
2 2
Therefore 𝐼𝐿 = = 1.126039724 and
2
1 3
𝑓( )+𝑓(1)+𝑓( )+𝑓(2)
2 2
𝐼𝑅 = = 0.6351975438
2

Tesfaye G and Wanaw B Page 54


CALCULUS II

Fig(a) Use left endpoints Fig(b) Use right endpoints

Apparently, 𝐼𝐿 and 𝐼𝑅 are calculated by combining the areas of certain rectangles. In our case
the values of 𝑓(𝑥) are all positive and all of the rectangles are above the 𝑥 axis, so the areas of
the rectangles are all added. Note also, that our specific function 𝑓(𝑥) is decreasing on the
interval [0 , 2], so that 𝐼𝐿 is an upper sum for the function 𝑓(𝑥) over the interval [0 , 2], and 𝐼𝑅 is
a lower sum. In this sense, we have
2 2
𝐼𝑅 = 0.6351975438 ≤ ∫0 𝑒 −𝑥 𝑑𝑥 ≤ 𝐼𝐿 = 1.126039724
The function and the rectangles whose areas are added to give us 𝐼𝐿 and 𝐼𝑅 are shown in
Figure(a) and Figure(b) above.

Midpoint and Trapezoid Method: We may try and improve on the endpoint methods. In the
midpoint methods, we use the value of the function at the midpoints of the intervals of the
partition. That should be less bias. We use the same partition and notation as above. Then the
formula for the midpoint method is:

𝑥0 + 𝑥1 𝑥𝑛 + 𝑥𝑛−1
𝐼𝑀 = 𝑓 ( ) (𝑥1 − 𝑥0 ) + ⋯ + 𝑓 ( ) (𝑥𝑛 − 𝑥𝑛−1 )
2 2
In the trapezoid method we do not take the function at the average (i.e. midpoint) of the
endpoints of the intervals in the partition, but we average the values of the function at the
endpoints. Specifically, the formula is

𝑓(𝑥0 ) + 𝑓(𝑥1 ) 𝑓(𝑥𝑛−1 ) + 𝑓(𝑥𝑛 )


𝐼𝑇 = (𝑥1 − 𝑥0 ) + ⋯ + (𝑥𝑛 − 𝑥𝑛−1 )
2 2

It is quite easy to see that


𝐼𝐿 + 𝐼𝑅
𝐼𝑇 =
2

Tesfaye G and Wanaw B Page 55


CALCULUS II

Let us explain the reference to the word trapezoid. For simplicity, suppose that 𝑓(𝑥) is non-
negative on the interval [𝑎 , 𝑏]. Consider the trapezoid of width (𝑥1 − 𝑥0) which has height 𝑓(𝑥0 )
𝑓(𝑥0 )+𝑓(𝑥1 )
at its left and 𝑓(𝑥1 ) at its right edge. The area of this trapezoid is . This is the first
2
summand in the formula for 𝐼𝑇 . We have such a trapezoid over each of the intervals in the
partition, and their areas are added to give 𝐼𝑇 .

Expressed differently, we can draw a secant line through the points (𝑥0 , 𝑓(𝑥0 ))and
(𝑥1 , 𝑓(𝑥1 )).This gives us the graph of a function 𝑇(𝑥) over the interval [𝑥0 , 𝑥1 ].Over the interval
[𝑥1 , 𝑥2 ] the graph of 𝑇(𝑥) is the secant line through the points (𝑥1 , 𝑓(𝑥1 )) and
(𝑥2 , 𝑓(𝑥2 )).Proceeding in the fashion, we use appropriate secant lines above all of the intervals
in the partition to define the function 𝑇(𝑥) over the entire interval [𝑎, 𝑏]. Then

𝑏
𝐼𝑇 = ∫ 𝑇(𝑥) 𝑑𝑥
𝑎
This integral is easily computed by the formula
𝑓(𝑥0 ) + 𝑓(𝑥1 ) 𝑓(𝑥𝑛−1 ) + 𝑓(𝑥𝑛 )
𝐼𝑇 = (𝑥1 − 𝑥0 ) + ⋯ + (𝑥𝑛 − 𝑥𝑛−1 )
2 2

EXAMPLE 2 Use the midpoint and trapezoid method to find approximate values for

2
2
∫ 𝑒−𝑥 𝑑𝑥
0

Solution: We use the same partition of [0, 2] and accuracy as in Example 1. For 𝐼𝑀 ,

𝑥0 +𝑥1 𝑥1 +𝑥2 𝑥2 +𝑥3 𝑥3 +𝑥4


𝐼𝑀 = 𝑓 ( ) (𝑥1 − 𝑥0 ) + 𝑓 ( ) (𝑥2 − 𝑥1 ) + 𝑓 ( ) (𝑥3 − 𝑥2 ) + 𝑓 ( ) (𝑥4 − 𝑥3 )
2 2 2 2

𝑓(0.25) + 𝑓(0.75) + 𝑓(1.25) + 𝑓(1.75)


𝐼𝑀 = = 0.8827889485
2

As for the endpoint methods, 𝐼𝑀 is the combined area of certain rectangles. Their heights are
the values 𝑓(𝑥𝑖 ) at the midpoints of the intervals of the partition. Their widths are the lengths of
the intervals of the partition. The areas of the rectangles are added or subtracted, depending on
whether 𝑓(𝑥𝑖 ) is positive or negative. In our specific case, the areas of the rectangles are all
added. You see this calculation illustrated in the following Figure.

Tesfaye G and Wanaw B Page 56


CALCULUS II

Fig(a) Use midpoints Fig(b) Trapezoid Method


𝐼𝐿 +𝐼𝑅
Based on our previous calculations and Formula 𝐼𝑇 = we find 𝐼𝑇 = 0.8806186341
2

2
We illustrated this calculation in Figure (b) above. There you see the function 𝑓(𝑥) = 𝑒 −𝑥
for which we like to find the integral over the interval [0, 2]. you also see five dots on the graph,
and they are connected by straight line segments. These line segments form the graph of a
function 𝑇(𝑥), and 𝐼𝑇 is the area of the region under this graph. So
2
𝐼𝑇 = ∫ 𝑇(𝑥) 𝑑𝑥
0

Simpson's Method: In Simpson's method we combine the endpoint and midpoint methods in a
weighted fashion. Again, we use the same notation for the function and the partition as above.
The specific formula for an approximate value of the integral of 𝑓(𝑥) over [𝑎, 𝑏] is

1 𝑥0 + 𝑥1
𝐼𝑆 = [𝑓(𝑥0 ) + 4𝑓 ( ) + 𝑓(𝑥1 )] (𝑥1 −𝑥0 ) + ⋯
6 2
1 𝑥𝑛−1 + 𝑥𝑛
+ [𝑓(𝑥𝑛−1 ) + 4𝑓 ( ) + 𝑓(𝑥𝑛 )] (𝑥𝑛 −𝑥𝑛−1 )
6 2

It is quite easy to see that


𝐼𝐿 + 4𝐼𝑀 + 𝐼𝑅 𝐼𝑇 + 2𝐼𝑀
𝐼𝑆 = =
6 3

Let us explain the background to Simpson's method. We define a function 𝑃(𝑥) over the
interval [𝑎, 𝑏] by defining a degree 2 polynomial on each of the intervals of the partition. The
polynomial over the interval [𝑥𝑘−1 , 𝑥𝑘 ] is chosen so that it agrees with 𝑓(𝑥) at the endpoints and
at the midpoint of this interval. In other words, the polynomial goes through the points
𝑥𝑘−1 +𝑥𝑘 𝑥𝑘−1 +𝑥𝑘
(𝑥𝑘−1 , 𝑓(𝑥𝑘−1 )) , ( ,𝑓( ))) and (𝑥𝑘 , 𝑓(𝑥𝑘 )). With some work one can then
2 2
𝑏
show that .𝐼𝑆 = ∫𝑎 𝑝(𝑥)𝑑𝑥

Tesfaye G and Wanaw B Page 57


CALCULUS II

In this sense, Simpson's method is a refinement of the Trapezoid method. In one method we
use two points on the graph and connect them by a straight line segment. In the other one we use
three points on the graph and construct a parabola through them.

EXAMPLE 3 Use Simpson's method to find an approximate value for


2
2
∫ 𝑒 −𝑥 𝑑𝑥
0
Solution: We use the same partition of [0, 2] and accuracy as in Example 1. The formula for 𝐼𝑆
specializes to
𝐼𝐿 + 4𝐼𝑀 + 𝐼𝑅
𝐼𝑆 =
6
Where 𝐼𝐿 , 𝐼𝑀 and 𝐼𝑅 are as above. Using the values from Examples 1and 2, we come up with a
numerical value for 𝐼𝑆 :
𝐼𝑆 = 0.88206555104

Fig (Simpson's Method)

You see the method illustrated in the above Figure. There you see the graphs of two
2
functions, the function 𝑓(𝑥) = 𝑒 −𝑥 and the function 𝑃(𝑥) from the discussion of Simpson's
method. Only the thickness of the line suggests that there are two graphs of almost identical
functions.
EXAMPLE 4 Compare the accuracy of the various approximate values of
2 2
∫0 𝑒 −𝑥 𝑑𝑥

Tesfaye G and Wanaw B Page 58


CALCULUS II

Solution: We compare the approximate values for the integral obtained by the different
formulas. In addition, we vary the number n of smaller intervals into which [0, 2] is subdivided.
We tabulate the results. They should be compared with an approximate value for the integral of
0.882081390762421

𝑛=1 𝑛 = 10 𝑛 = 100 𝑛 = 1000


𝐼𝐿 2.0000000 0.9800072469 0.891895792451 0.883063050702697
𝐼𝑅 0.0366313 0.7836703747 0.872262105229 0.881095681980474
𝐼𝑀 0.7357589 0.8822020700 0.882082611663 0.882081402972833
𝐼𝑇 1.0183156 0.8818388108 0.882078948840 0.882081366341586
𝐼𝑆 0.8299445 0.8820809836 0.882081390722 0.882081390762417

It should be apparent that Simpson's method is far superior to all of the other ones.
E.g., Simpson's method with n = 4 gives a result which is better than the left and right endpoint
method with n = 1000. Even if you use the Midpoint and trapezoid method with n = 1000, then
the result is far less accurate that Simpson's method with n = 100.

Exercise 2.6: Proceed as in Example 4 and compare the different methods applied to
the calculation of
𝝅
𝟐
∫ 𝒔𝒊𝒏𝒙𝒅𝒙
𝟎

Tesfaye G and Wanaw B Page 59


CALCULUS II

2.7- Application of integration (area, volume, arc length, surface area)

Area
Let f be non-negative continuous functions on [a, b], then the area of the region R bounded by f
and the x-axis between x =a & x = b (as shown in the fig. below) is given by:
𝑏
𝐴(𝑅) = ∫𝑎 𝑓(𝑥)𝑑𝑥 …………………………….(*)

Example:1)Find the area of the region bounded by the graph of the function 𝑓(𝑥) = 𝑥 2 − 3𝑥 + 2
and the x-axis between x=0 & x=3.
Solution: The region R is the shaded region as shown below.

Now let R1, R2 and R3 be the sub-regions b/n x=0 and x=1, x=1 and x=2, and x=2 and x=3,
respectively. Then, using (*):
𝐴(𝑅) = 𝐴(𝑅1 ) + 𝐴(𝑅2 ) + 𝐴(𝑅3 )
1 1 𝑥3 𝑥2 1 5
Where, 𝐴(𝑅1 ) = ∫0 𝑓(𝑥)𝑑𝑥 = ∫0 (𝑥 2 − 3𝑥 + 2)𝑑𝑥 = ( 3 − 3 + 2𝑥) | = 6
2 0
2 𝑥3 𝑥2 2 1
𝐴(𝑅2 ) = − ∫1 (𝑥 2 − 3𝑥 + 2)𝑑𝑥 = −( 3 − 3 + 2𝑥) | = 6
2 1
Since the sub-region R2 is below the x- axis, the negative sign is important. Similarly,

Tesfaye G and Wanaw B Page 60


CALCULUS II

3 𝑥3 𝑥2 3 5
𝐴(𝑅3 ) = ∫2 (𝑥 2 − 3𝑥 + 2)𝑑𝑥 = ( 3 − 3 + 2𝑥) | = 6. Thus, area of the region is:
2 2
5 1 5 11
𝐴(𝑅) = 6 + 6 + 6 = .
6

We can see that the area of a region bounded by a continuous curves y = f(x), y = g(x) and the
lines x = a, x = b such that 𝑓(𝑥) ≥ 𝑔(𝑥) for all x in [a, b] is:
𝑏
𝐴(𝑅) = ∫𝑎 [𝑓(𝑥) − 𝑔(𝑥)]𝑑𝑥 ……………………(*)

Example 2: Find area of the region enclosed by the parabolas 𝑦 = 𝑥 2 and 𝑦 = 2𝑥 − 𝑥 2 .


Solution: The sketch of the region is shown below. So, using (**):

1 1 1
2) 2 ]𝑑𝑥
𝐴(𝑅) = ∫ [𝑓(𝑥) − 𝑔(𝑥)]𝑑𝑥 = ∫ [(2𝑥 − 𝑥 −𝑥 = ∫ (2𝑥 − 2𝑥 2 )𝑑𝑥
0 0 0
1 1
𝑥3
= (𝑥 2 − 2 3 ) | = 3.
0
Note that the intersection points can be obtained as: 𝑦 = 2𝑥 − 𝑥 2 = 𝑥 2 ⇒ 2𝑥 2 − 2𝑥 = 0.
Solving this equation for x, we get 𝑥 = 0 𝑜𝑟 𝑥 = 1, then 𝑦 = 0 𝑜𝑟 𝑦 = 1, as indicated in the
graph above.
Example 3: Find the area of the region bounded by the curves y = sin x and y = cos x, between
𝜋
x = 0 & x = 2.

Solution: The region R with its parts is as shown below:

Tesfaye G and Wanaw B Page 61


CALCULUS II

𝜋 𝜋
The points of intersection occur when sin x = cos x, that is, when 𝑥 = 4 (since 0 ≤ 𝑥 ≤ 2 ).
𝜋 𝜋 𝜋
Observe that cos x ≥ sin x when 0 ≤ 𝑥 ≤ but cos x ≤ sin x when 4 ≤ 𝑥 ≤ 2 . Therefore, area of
4

the region is:


𝜋 𝜋
𝐴(𝑅) = 𝐴1 + 𝐴2 = ∫04 [𝑐𝑜𝑠𝑥 − 𝑠𝑖𝑛𝑥]𝑑𝑥 + ∫𝜋2 [𝑠𝑖𝑛𝑥 − 𝑐𝑜𝑠𝑥]𝑑𝑥
4
𝜋
𝜋
= [𝑠𝑖𝑛𝑥 + 𝑐𝑜𝑠𝑥] | 4 + [−𝑐𝑜𝑠𝑥 − 𝑠𝑖𝑛𝑥] |𝜋2 = 2√2.
0 4

Exercise:
1. Find the area of the region enclosed by the graph of f(x) = sin x and the x-axis between
𝜋
𝑥 = − 2 and 𝑥 = 2𝜋.

2. Find the area of the region in the first quadrant which is enclosed by the y-axis and the
curves of f(x) = cos x and g(x) = sinx.
3. Determine the area of the region enclosed by the graphs of 𝑥 = −𝑦 2 and 𝑥 = 9 − 2𝑦 2
4. Find the area of the region enclosed by the graphs 𝑦 = 𝑥 2 + 1 and the line 𝑦 = 5.

VOLUME
In section, we will apply integral calculus to determine the volume of a solid region by
considering cross sections.
Suppose a region rotates about a straight line, then a solid figure called a solid of revolution,
is formed. The volume of such a solid is said to be a volume of revolution and the line about
which the region rotates is an axis of symmetry.
Now consider the solid of revolution generated by revolving the region between the curve
𝑦 = 𝑓(𝑥) and the x-axis from 𝑥 = 𝑎 to 𝑥 = 𝑏 as shown below.

Tesfaye G and Wanaw B Page 62


CALCULUS II

Every cross section which is perpendicular to the x-axis at x a circular region with radius, r = f(x).
Thus, the area of the cross section A(x) is 𝐴(𝑥) = 𝜋𝑟 2 = 𝜋(𝑓(𝑥))2 .
Thus, we define the volume of revolution V as by:
𝑏 𝑏
𝑉 = ∫𝑎 𝐴(𝑥) 𝑑𝑥 = ∫𝑎 𝜋(𝑓(𝑥))2 𝑑𝑥 ……………………..(∆)
Example 1: Find the volume of the solid obtained by rotating about the x-axis the region under the
curve 𝑦 = √𝑥 from 0 to 1.
Solution: The region and the solid figure rotated about the x-axis from 0 to 1 are shown in fig.(a)
and fig.(b) below.

Now, the area of the cross section is 𝐴(𝑥) = 𝜋(𝑓(𝑥))2 = 𝜋(√𝑥)2 = 𝜋𝑥. So, using (∆), the volume
of revolution V is given by:
1 1 𝑥2 1 𝜋
𝑉 = ∫0 𝐴(𝑥) 𝑑𝑥 = ∫0 𝜋𝑥 𝑑𝑥 = 𝜋 | = 2.
2 0
Note that if f and g are continuous, non-negative on [a, b] such that 𝑓(𝑥) ≥ 𝑔(𝑥) for all x in [a, b].
let R be a region bounded by f(x) and g(x), on [a, b] as in the following figure. Then, the volume of
the solid of revolution generated by revolving the region R about the x-axis is given by:
𝑏 2 2
𝑉 = ∫𝑎 𝐴(𝑥) 𝑑𝑥, where 𝐴(𝑥) = 𝜋[(𝑓(𝑥)) − (𝑔(𝑥)) ].

Tesfaye G and Wanaw B Page 63


CALCULUS II

Example 2: The region R enclosed by the curves 𝑦 = 𝑥 and 𝑦 = 𝑥 2 is rotated about the x-axis.
Find the volume of the resulting solid.
Solution:

The curves y = x and y = x2 intersect at the points (0,0) and (1, 1). The region and solid of rotation
are shown in the fig. above. The cross-section in the plane has the shape of annular ring with inner
radius x2 and outer radius x. So, 𝐴(𝑥) = 𝜋[𝑥 2 − (𝑥 2 )2 ] = 𝜋(𝑥 2 − 𝑥 4 ). Therefore, we have
1 1 𝑥3 𝑥5 1 2𝜋
𝑉 = ∫0 𝐴(𝑥) 𝑑𝑥 = ∫0 𝜋(𝑥 2 − 𝑥 4 ) 𝑑𝑥 = 𝜋[ 3 − ] | = 15
5 0

Consider again the case where a region R is rotated about the y-axis: where, either R is a region
bounded by the curve x = u(y) and the y-axis, between the lines y = c & y = d or R is a region
between two curves v(y) and w(y) on [c, d] as in following figures:

Tesfaye G and Wanaw B Page 64


CALCULUS II

Then the corresponding volume of revolution is given by:


𝑑 𝑑
𝑉 = ∫𝑐 𝜋[𝑢(𝑦)]2 𝑑𝑦 and 𝑉 = ∫𝑐 𝜋([𝑤(𝑦)]2 − [𝑣(𝑦)]2 ) 𝑑𝑦 respectively.
Example 3: Find the volume of the solid obtained by rotating the region bounded by y = x3, y = 8,
and x = 0 about the y-axis.
Solution: The rotation of the region is shown as:

2
Here, u(y) = x = 3√𝑦 . So, the area of the cross-section is 𝐴(𝑦) = 𝜋[𝑢(𝑦)]2 = 𝜋𝑦 3 . Thus, the
volume V of revolution is:
2
𝑑 8 96𝜋
𝑉 = ∫𝑐 𝜋[𝑢(𝑦)]2 𝑑𝑦 = ∫0 𝜋𝑦 3 𝑑𝑦 = .
5

Exercises:
1. Find the volume of solid of revolution about the x-axis generated by revolving the area
between the lines y = x and y = 4 from x = 1 to x = 3.
2. Find the volume of the solid of revolution about the y-axis generated by revolving the
region enclosed by the curve x = √𝑦 and the y-axis from y = 0 to y = 4.

Tesfaye G and Wanaw B Page 65


CALCULUS II

3. The area bounded by the graph of y = x2+1 and the line y = 4 rotates about the y – axis,
find the volume of the solid generated.
4. Find the volume of the solid obtained by rotating the region bounded by the curves y2 = x
and x = 2y about y-axis.

Arc length
In this section we are going to look at computing the arc length of a function. Because it’s
easy enough to derive the formulas that we’ll use in this section we will derive one of them and
leave the other to you to derive.
We want to determine the length of the continuous function 𝑦 = 𝑓 (𝑥) on the interval
[𝑎, 𝑏].Initially we’ll need to estimate the length of the curve. We’ll do this by dividing the
interval up into n equal subintervals each of width 𝛥𝑥 and we’ll denote the point on the curve at
each point by 𝑃𝑖 . We can then approximate the curve by a series of straight lines connecting the
points. Here is a sketch of this situation for 𝑛 = 9 .
Now

Now denote the length of each of these line segments by |𝑃𝑖−1 𝑃𝑖 | and the length of the curve will then be
approximately,
𝐿 = ∑𝑛𝑖−1 |𝑃𝑖−1 𝑃𝑖 |
and we can get the exact length by taking n larger and larger. In other words, the exact length will be,

𝐿 = lim𝑛→∞ ∑𝑛𝑖−1 |𝑃𝑖−1 𝑃𝑖 |


Now, let’s get a better grasp on the length of each of these line segments. First, on each segment
let’s define. We can then compute directly the length of the line segments as follows.

Tesfaye G and Wanaw B Page 66


CALCULUS II

2
|𝑃𝑖−1 𝑃𝑖 | = √(xi − xi−1 )2 + (yi − yi−1 ) = √∆x2 + ∆yi 2

By the Mean Value Theorem we know that on the interval [𝑥𝑖−1 , 𝑥𝑖 ] there is a point 𝑥𝑖∗ so that,

However, using the definition of the definite integral, this is nothing more than,

Before we work any examples we need to make a small change in notation. Instead of having
two formulas for the arc length of a function we are going to reduce it, in part, to a single
formula.

Tesfaye G and Wanaw B Page 67


CALCULUS II

Arc Length Formula(s)

Tesfaye G and Wanaw B Page 68


CALCULUS II

1 1
Example 3 Determine the length of 𝑥 = 𝑦 2 for 0 ≤ 𝑥 ≤ . Assume that y is positive.
2 2

Tesfaye G and Wanaw B Page 69


CALCULUS II

Tesfaye G and Wanaw B Page 70


CALCULUS II

Exercise :Find the length of the curve

3
1. 𝑦 = 1 + 6𝑥 2 , 0≤ 𝑥 ≤ 1
2. 𝑦 2 = 4(𝑥 + 4)3 , 0≤ 𝑥 ≤ 2 𝑦>0
𝑥5 1
3. 𝑦 = + 10𝑥 3 , 1 ≤ 𝑥 ≤ 2
6
𝑥2 𝑙𝑛𝑥
4. 𝑦 = + , 2≤ 𝑥 ≤ 4
2 4
1
5. 𝑥 = 3 √𝑦(𝑦 − 3) , 1 ≤ 𝑦 ≤ 9
𝜋
6. 𝑦 = ln(𝑐𝑜𝑠𝑥) , 0≤ 𝑥 ≤ 3
2
7. 𝑦 = 4𝑥 ,0 ≤ 𝑦 ≤ 2

Surface area
In this section we are going to look once again at solids of revolution. We first looked at them
when we found the volume of the solid of revolution. In this section we want to find the surface
area of this region.

So, for the purposes of the derivation of the formula, let’s look at rotating the continuous
function 𝑦 = 𝑓 (𝑥) in the interval [𝑎, 𝑏] about the x-axis. Below is a sketch of a function and
the solid of revolution we get by rotating the function about the x-axis.

We can derive a formula for the surface area much as we derived the formula for arc length.
We’ll start by dividing the integral into n equal subintervals of width 𝛥𝑥 . On each subinterval
we will approximate the function with a straight line that agrees with the function at the
endpoints of the each interval. Here is a sketch of that for our representative function using n = 4.

Tesfaye G and Wanaw B Page 71


CALCULUS II

Now, rotate the approximations about the x-axis and we get the following solid.

The approximation on each interval gives a distinct portion of the solid. Each of these portions
are called frustums and we know how to find the surface area of frustums.
The surface area of a frustum is given by,
𝐴 = 2𝜋𝑟𝑙
Where,
1
𝑟 = (𝑟1 + 𝑟2 )
2
𝑟1 = Radius of right end
𝑟1 = Radius of left end
And 𝑙 is the length of the slant of the frustum.

Tesfaye G and Wanaw B Page 72


CALCULUS II

Tesfaye G and Wanaw B Page 73


CALCULUS II

Tesfaye G and Wanaw B Page 74


CALCULUS II

Tesfaye G and Wanaw B Page 75


CALCULUS II

Tesfaye G and Wanaw B Page 76


CALCULUS II

Exercise :Find the area of the surface obtained by rotating the curve about x-axis

1. 𝑦 = 𝑥3 , 0 ≤ 𝑥 ≤ 2
2. 9𝑥 = 𝑦 2 + 18 , 2 ≤ 𝑥 ≤ 6
3. 𝑦 = √𝑥 , 4 ≤ 𝑥 ≤ 9
𝜋
4. 𝑦 = 𝑐𝑜𝑠2𝑥 , 0 ≤ 𝑥 ≤ 6
5. 𝑦 = 𝑐𝑜𝑠ℎ𝑥 , 0 ≤ 𝑥 ≤ 1
𝑥3 1 1
6. 𝑦 = + 2𝑥 , ≤𝑥≤1
6 2
3
1
7. 𝑥 = 3 (𝑦 2 + 2) , 1 ≤ 𝑦 ≤ 2
2

8. 𝑥 = 1 + 2𝑦 2 , 1 ≤ 𝑦2

CHAPTER 3
Tesfaye G and Wanaw B Page 77
CALCULUS II

Indeterminate forms, improper integrals and Taylor’s polynomial


3.1) Cauchy’s formula
Objectives:
At the end of this topic, students be able to
 State and proof Cauchy formula
 Do examples and exercises related with this formula

Theorem: If 𝑓 𝑎𝑛𝑑 𝑔 are continuous on [a, b] and differentiable on (a, b) and if 𝑔′ (𝑥) ≠ 0 for
every x in (a, b), then there is a number c in (a, b) such that
𝑓(𝑏)−𝑓(𝑎) 𝑓 ′ (𝑐)
= 𝑔′ (𝑐)
𝑔(𝑏)−𝑔(𝑎)
Proof: we first not that𝑔(𝑏) − 𝑔(𝑎) ≠ 0, because otherwise g(a)=g(b) and, by Rolle’s theorem,
there is a number c is (a, b) such that 𝑔′ (𝑐)=0, contrary to our assumption about 𝑔′ .
Let us introduce a new function h as follows:
ℎ(𝑥) = [𝑓(𝑏) − 𝑓(𝑎)]𝑔(𝑥) − [𝑔(𝑏) − 𝑔(𝑎)]𝑓(𝑥)
for every x in [a, b]. It follows that h is continuous on [a, b] and differentiable on (a, b) and that
h(a)=h(b). By Rolle’s Theorem, there is a number c in (a, b) such that ℎ′ (𝑐) = 0; that is,
[𝑓(𝑏) − 𝑓(𝑎)] 𝑔′ (𝑐) −[ 𝑔(𝑏) − 𝑔(𝑎)] 𝑓 ′ (𝑐) = 0.
This is equivalent to Cauchy’s formula.
3.2 Indeterminate forms (L’Hopital’s Rule)
Objectives:
At the end of this topic, students be able to
 Distinguish indeterminate forms of type 0/0
 Apply L’Hopital’s rule for form 0/0
 distinguish indeterminate form of type ∞/∞
 Apply L’Hopital’s rule for form ∞/∞
 Distinguish indeterminate forms of type 0.∞, ∞-∞,00 ∞0 𝑎𝑛𝑑 1∞

Indeterminate forms of type 𝟎⁄𝟎


𝑠𝑖𝑛𝑥
lim𝑥→0 𝑥 = 1 …………………………………………………………….(1)
What makes the limit in (1) bothersome is the fact that the numerator and denominator both
approach 0 as 𝑥 → 0. Such limits are called indeterminate forms of type 0/0. In limits of this
type there are two tendencies working against each other: as the numerator approaches 0 it tends
to drive the ratio toward 0, and as the denominator approaches 0 it tends to drive the ratio toward
0 it tends to drive the ratio toward +∞ 𝑜𝑟 − ∞. What happens in (1) is that these conflicting
tendencies offset each other in such a way that the limit is 1.

L’Hopital’s Rule
The idea of using local linear approximation to evaluate indeterminate forms of type 0/0 can be
used to motivate a more general procedure for finding such limits. For this purpose, suppose that

Tesfaye G and Wanaw B Page 78


CALCULUS II

𝑓(𝑥)
lim
𝑥→𝑥0 𝑔(𝑥)
Is an indeterminate form of type 0/0, that is,
lim𝑥→𝑥0 𝑓(𝑥) = 0 𝑎𝑛𝑑 lim𝑥→𝑥0 𝑔(𝑥) = 0…………………………………………….(2)
For simplicity, let us also assume that 𝑓 𝑎𝑛𝑑 𝑔 are differentiable at 𝑥 = 𝑥0 and 𝑓’ and 𝑔’ are
continuous at 𝑥 = 𝑥0 .The differentiability of 𝑓 𝑎𝑛𝑑 𝑔 at 𝑥 = 𝑥0 implies that 𝑓 𝑎𝑛𝑑 𝑔 are
continuous at 𝑥 = 𝑥0 , and hence from (2)
𝑓(𝑥0 )= lim𝑥→𝑥0 𝑓(𝑥) = 0 𝑎𝑛𝑑 𝑔(𝑥0 ) = lim 𝑔(𝑥0 ) = 0…………………………..(3)
𝑥→𝑥0
Moreover, the continuity of 𝑓’ 𝑎𝑛𝑑 𝑔’ at 𝑥 = 𝑥0 implies that
lim 𝑓 ′ (𝑥) = 𝑓 ′ (𝑥0 ) 𝑎𝑛𝑑 lim 𝑔′ (𝑥) = 𝑔′ (𝑥0 )…………………………………………..(4)
𝑥→𝑥0 𝑥→𝑥0
Thus from (3) and (4) and the local linear approximations of f and g at 𝑥 → 𝑥0 , we have
𝑓(𝑥) f(x0 ) + f ′ (x0 )(x − x0 )
lim = lim
𝑥→𝑥0 𝑔(𝑥) 𝑥→𝑥0 g(x0 ) + g ′ (x0 )(x − x0 )
f′ (x0 )(x−x0 ) f′ (x )
= lim = g′ (x0 )
𝑥→𝑥0 g′ (x0 )(x−x0 ) 0
This from (4) can be expressed as
𝑓(𝑥) f′ (x)
lim = lim …………………………………………………………………… (5)
𝑥→𝑥0 𝑔(𝑥) 𝑥→𝑥0 g′ (x)
This result called L’Hopital’s rule, converts an indeterminate form of type 0/0 in to a new limit
involving derivatives that in many situations can be evaluated by inspection or by algebraic
methods. For example,
𝑑
1 − 𝑐𝑜𝑠𝑥 [1 − 𝑐𝑜𝑠𝑥] 𝑠𝑖𝑛𝑥
lim = lim 𝑑𝑥 = lim = sin 0 = 0
𝑥→0 𝑥 𝑥→0 𝑑 𝑥→0 1
[𝑥]
𝑑𝑥
THEOREM (L’Hopital’s rule for form 0/0). Let 𝑙𝑖𝑚 stand for one of the limits
lim , lim+ , lim− , lim , 𝑜𝑟 lim , 𝑎𝑛𝑑 𝑠𝑢𝑝𝑝𝑜𝑠𝑒 𝑡ℎ𝑎𝑡 lim 𝑓(𝑥) = 0 𝑎𝑛𝑑 lim 𝑔(𝑥) =
𝑥→𝑎 𝑥→𝑎 𝑥→𝑎 𝑥→+∞ 𝑥→−∞
𝑓 ′ (𝑥)
0. 𝑖𝑓 lim [𝑔′ (𝑥)] ℎ𝑎𝑠 𝑎 𝑓𝑖𝑛𝑖𝑡𝑒 𝑣𝑎𝑙𝑢𝑒 𝐿, 𝑜𝑟 𝑖𝑓 𝑡ℎ𝑖𝑠 𝑙𝑖𝑚𝑖𝑡 𝑖𝑠 + ∞ 𝑜𝑟 − ∞, 𝑡ℎ𝑒𝑛
𝑓(𝑥) 𝑓 ′ (𝑥)
lim = 𝑙𝑖𝑚 ′
𝑔(𝑥) 𝑔 (𝑥)
REMARK. Note that in L’Hopital’s rule the numerator and denominator are differentiated
separately, which is not the same as differentiating f(x)/g(x).
In the following examples we will apply L’Hopital’s rule using the following three-step
process:
Step 1. Check that 𝑙𝑖𝑚 𝑓(𝑥)/𝑔(𝑥) is an indeterminate form. If it is not, then
L’Hopital’s rule cannot be used.
Step 2. Differentiate 𝑓 𝑎𝑛𝑑 𝑔 separately.
Step 3. Find 𝑙𝑖𝑚 𝑓’(𝑥)/𝑔(𝑥). If this limit is finite, +∞ , or -∞ , then it is equal to lim 𝑓(𝑥)/𝑔(𝑥).

Tesfaye G and Wanaw B Page 79


CALCULUS II

Tesfaye G and Wanaw B Page 80


CALCULUS II

Tesfaye G and Wanaw B Page 81


CALCULUS II

Indeterminate forms of type ∞/∞

Tesfaye G and Wanaw B Page 82


CALCULUS II

Indeterminate forms of type 0.∞

Tesfaye G and Wanaw B Page 83


CALCULUS II

Tesfaye G and Wanaw B Page 84


CALCULUS II

Indeterminate forms of type ∞ − ∞

Indeterminate forms of type 𝟎𝟎 , ∞𝟎 , 𝟏∞


Tesfaye G and Wanaw B Page 85
CALCULUS II

Tesfaye G and Wanaw B Page 86


CALCULUS II

Exercise
In exercise 1 and 2, evaluate the given limit without using L’Hopital’s rule, and then check that
your answer is correct using L’Hopital’s rule.
𝑥 2 −4 2𝑥−5
1. (a) 𝑙𝑖𝑚𝑥→2 𝑥 2 +2𝑥−8 (b) 𝑙𝑖𝑚𝑥→+∞ 3𝑥+7
𝑠𝑖𝑛𝑥 𝑥 2 −1
2. (a) 𝑙𝑖𝑚𝑥→0 𝑡𝑎𝑛𝑥 (b) 𝑙𝑖𝑚𝑥→2 𝑥 3 −1

Tesfaye G and Wanaw B Page 87


CALCULUS II

3.3- Improper Integrals

Objectives:
By the end of this section, students will be able to:
 Define Improper Integral
 Evaluate Improper integral

Over view: In this section we will formally define the improper integral and evaluate improper
integrals.

we define the integral of f (not necessarily a positive function) over an infinite interval as the
limit of integrals over finite intervals.

Tesfaye G and Wanaw B Page 88


CALCULUS II

Tesfaye G and Wanaw B Page 89


CALCULUS II

Therefore,

Tesfaye G and Wanaw B Page 90


CALCULUS II

Tesfaye G and Wanaw B Page 91


CALCULUS II

Tesfaye G and Wanaw B Page 92


CALCULUS II

Tesfaye G and Wanaw B Page 93


CALCULUS II

Exercise: Evaluate the following Integrals

Tesfaye G and Wanaw B Page 94


CALCULUS II

3.4 Taylor polynomial and its application


Overview
In this section we will discuss methods for finding Taylor polynomial and use this to
approximate Taylor polynomial function

Section objective:

After you complete the study of this subtopic, you should be able to:
 define Taylor polynomial;
 find Taylor polynomial;
 use Taylor Polynomial to approximate a function.
Taylor polynomials are applicable in approximating functions because polynomials are the
simplest of functions.
Definition : Suppose that f(x) is equal to the sum of its Taylor series at a, that is,
fn (a)
f(x) = ∑∞
n=0 . (x − a)n , then its nth - partial sum of the Taylor series denoted by Pn (x)
n!

which is called the nth degree Taylor polynomial of f at a is given by


fi (a) f′′ (a) fn (a)
pn (x) = ∑ni=0 . (x − a)i = f(a) + f ′ (a)(x − a) + (x − a)2 + ⋯ + . (x − a)n
i! 2! n!

Thus pn (x) can be used as an approximation to f i.e. f(x) ≅ pn (x) .


Remark:
fi (a)
1. pn (x) = ∑ni=0 . (x − a)i
i!

When n = 1 , p1 (x) = f(a) + f ′ (a). (x − a)


f′′ (a)
When n = 2 , p2 (x) = f(a) + f ′ (a). (x − a) + (x − a)2
2!

2. When using a Taylor polynomial pn to approximate a function f , we have to ask the


question how good an approximation is it . To answer this we need to look at the absolute
value of the remainder:
|R n (x)| = |f(x) − Tn (x)|
As n → ∞ , |R n (x)| → ∞ and pn (x) → f(x) .

Example 1:

Tesfaye G and Wanaw B Page 95


CALCULUS II

3
1. Approximate the function f(x) = √x by a Taylor polynomial of degree 2 at a = 8
3 1⁄
Solution: f(x) = √x = x 3

Step1: Compute the derivative f n (a) until n = 2


1⁄ 1⁄
f(x) = x 3 ⟹ f(8) = 8 3 =2
1 1 1
f ′ (x) = 2 ⟹ f ′ (8) = 2 =
3x ⁄3 3.(8 ⁄3 ) 12

−2 −2 −1
f ′′ (x) = 5 ⟹ f ′′ (8) = 5 =
9x ⁄3 9. (8 ⁄3 ) 144

Thus the second degree Taylor polynomial is


fi (8) f′′ (8)
p2 (x) = ∑2i=0 . (x − 8)i = f(8) + f ′ (8)(x − 8) + (x − 8)2
i! 2!
1 1 1
= 2 + 12 (x − 8) − 144 . 2! (x − 8)2
1 1
= 2 + 12 (x − 8) − 288 (x − 8)2
3 1 1
Thus the desired approximation is √x ≅ p2 (x) = 2 + 12 (x − 8) − 288 (x − 8)2

2. Find the Taylor polynomial pn (x) for the function f at the number a .
π
a. f(x) = sin x at a = , n=3
6

Solution:
Compute all the derivative f n (0), n = 1 , 2 , 3
π π 1
f(x) = sin x ⟹ f ( 6 ) = sin (6 ) = 2
π π √3
f ′ (x) = cos x ⟹ f ′ ( 6 ) = cos( 6 ) = 2
π π 1
f ′′ (x) = − sin x ⟹ f ′′ ( 6 ) = − sin ( 6 ) = − 2
π π √3
f ′′′ (x) = − cos x ⟹ f ′′′ ( 6 ) = − cos( 6 ) = − 2

Thus the third Taylor polynomial is


π π π
fi ( ) π i π π π f′′ ( ) π 2 f′′′ ( ) π
p3 (x) = ∑3i=0 6 . (x − ) = f (6) + f ′ ( 6 ) (x − 6) + 6
(x − ) + 6
(x − 6 )3
i! 6 2! 6 3!
π
2 π 3
1 √3 π 1 (x− 6 ) √3 (x− 6 )
= + (x − 6) − 2 −
2 2 2! 2 3!

Hence the desired approximation is


1 √3 π 1 π √3 π
sin x ≅ p3 (x) = 2 + 2
(x − 6 ) − 4 (x − 6 )2 − 12 (x − 6 )3

a. f(x) = ex , a = 2 and n = 3

Tesfaye G and Wanaw B Page 96


CALCULUS II

Solution: We have f(x) = ex and by definition the Taylor series at a = 2 is f(x) =


fn (a)
∑∞
n=0 . (x − a)n .
n!

Step1: Compute all the derivatives f n (a), n = 0 , 1 , 2 , 3


⟹ f ′ (x) = f ′′ (x) = ⋯ = f n (x) = ex for all x .
⟹ f n (a) = f n (2) = e2 for all n.
Step2: Write down the Taylor polynomial of f(x) at a = 2
fn (a)
⟹ f(x) = ∑3n=0 . (x − a)n
n!
f′′ (a) f′′′ (a)
= f(a) + f ′ (a). (x − a) + . (x − a)2 + . (x − a)3
2! 3!
f′′ (2) f′′′ (2)
= f(2) + f ′ (2). (x − 2) + . (x − 2)2 + . (x − 2)3
2! 3!

Thus the third Taylor polynomial is


(x−2)2 (x−2)3
p3 (x) = e2 + e2 (x − 2) + e2 + e2
2! 3!
(x−2)2 (x−2)3
⟹ p3 (x) = e2 (1 + (x − 2) + + )
2 6
(x−2)2 (x−2)3
⟹ ex ≅ p3 (x) = e2 (1 + (x − 2) + + )
2 6

b. f(x) = √3 + x 2 ; a = 1 and n = 2
Solution: f(x) = √3 + x 2 ⟹ f(1) = 2
x 1
f ′ (x) = √3+ ⟹ f ′ (1) = 2
x2
3 3
f ′′ (x) = (3+ ⟹ f ′′ (1) = 8
x2 ).√3+ x2

1 3 (x−1)2
Thus p2 (x) = 2 + . (x − 1) + .
2 8 2!
1 3
=2+ . (x − 1) + . (x − 1)2
2 16
1 3
⟹ √3 + x 2 ≅ p2 (x) = 2 + . (x − 1) + . (x − 1)2
2 16

c. Express the polynomial f(x) = 2. x 3 − 9 . x 2 + 11 . x − 1 as a polynomial in


(x − 2).
Solution: we have a = 2 and f(x) = 2. x 3 − 9 . x 2 + 11 . x − 1
⟹ f(2) = 1 and f ′ (x) = 6. x 2 − 18 . x + 11
⟹ f ′ (2) = −1
f ′′ (x) = 12. x − 18

Tesfaye G and Wanaw B Page 97


CALCULUS II

⟹ f ′′ (2) = 6
f ′′′ (x) = 12
⟹ f ′′′ (2) = 12
In general f n (x) = 0 for all n ≥ 4 ⟹ f n (2) = 0 for all n ≥ 4
6 12
Therefore f(x) = 1 − (x − 2) + . (x − 2)2 + (x − 2)3
2! 3!

= 1 − (x − 2) + 3 . (x − 2)2 + 2 (x − 2)3.
Example 2:
1. Find the Taylor series of f about a
a. f(x) = 4x 2 − 2x + 1 , a = 0 , 3
Solution: f(x) = 4x 2 − 2x + 1 ⟹ f(0) = 1
f ′ (x) = 8x − 2 ⟹ f ′ (0) = −2
f ′′ (x) = 8 ⟹ f ′′ (0) = 8
In general f n (x) = 0 for all n ≥ 3 ⟹ f n (0) = 0 for all n ≥ 3
8x2
f(x) = 1 − 2x + = 1 − 2x + 4x 2
2!

Similarly by the same method we obtain f(3) = 31 , f ′ (3) = 22 , f ′′ (3) = 8


8(x−3)2
⟹ f(x) = 31 + 22(x − 3) + = 31 + 22(x − 3) + 4(x − 3)2
2!
1
b. f(x) = x , a = −1
1
Solution: f(x) = x ⟹ f(−1) = −1
−1
f ′ (x) = ⟹ f ′ (−1) = −1
x2
2
f ′′ (x) = x3 ⟹ f ′′ (−1) = −2
−6
f ′′′ (x) = ⟹ f ′′′ (−1) = −6
x4
24
f 4 (x) = ⟹ f 4 (−1) = −24
x5
−120
f 5 (x) = ⟹ f 5 (−1) = −120
x6

In general f n (−1) = −n!


Thus,
2(x+1)2 6(x+1)3 24(x+1)4
f(x) = −1 − (x + 1) − − − +⋯
2! 3! 4!

= −1 − (x + 1) − (x + 1)2 − (x + 1)3 − (x + 1)4 + ⋯

Tesfaye G and Wanaw B Page 98


CALCULUS II

c. f(x) = sin 2x , a=0


Solution: f(x) = sin 2x ⟹ f(0) = sin 0 = 0
f ′ (x) = 2 cos 2x ⟹ f ′ (0) = 2 . cos 0 = 2
f ′′ (x) = −4 sin 2x ⟹ f ′′ (0) = −4 . sin 0 = 0
f 3 (x) = −8 cos 2x ⟹ f 3 (0) = −8 . cos 0 = −8
f 4 (x) = 16 sin 2x ⟹ f 4 (0) = 16 . sin 0 = 0
f 5 (x) = 32 cos 2x ⟹ f 5 (0) = 32. cos 0 = 32
f 6 (x) = −64 sin 2x ⟹ f 6 (0) = −64 . sin 0 = 0
f 7 (x) = −128 cos 2x ⟹ f 7 (0) = −128 . cos 0 = −128
Thus,
8x3 x5 x7 (2x)2n+1
f(x) = 2x − + 32 5! − 128 + ⋯ = ∑∞ n
n=0(−1) . (2n+1)!
3! 7!

d. f(x) = √x , a = 1
1⁄
Solution: f(x) = √x = x 2 ⟹ f(1) = 1
1 1
f ′ (x) = 2 ⟹ f ′ (1) = 2
√x
−1 −3⁄ −1
f ′′ (x) = .x 2 ⟹ f ′′ (1) =
4 4
3 −5⁄ 3
f 3 (x) = . x 2 ⟹ f 3 (1) = 8
8
−15 −7⁄ −15
f 4 (x) = .x 2 ⟹ f 4 (1) =
16 16
105 −9 105
f 5 (x) = . x ⁄2 ⟹ f 5 (1) =
32 32

Thus,
1 1 (𝑥−1)2 3 (𝑥−1)3 15 (𝑥−1)4 105 (𝑥−1)5
f(x) = 1 + (𝑥 − 1) − . + . − 16 . + . + …
2 4 2! 8 3! 4! 32 5!

2. If f(x) = sin(x 3 ) , then find f 15 (0) ?


Solution: so far we have the Maclaurin series for
x3 x5 x7
f(x) = sin x = x − 3! + 5! − +⋯
7!
x9 x15 x21
Now sin(x 3 ) = x 3 − 3! + − +⋯
5! 7!
f15 (0).x15 x15 15!
= from which we obtain f 15 (0) = .
15! 5! 5!
1
3. Find the sixth derivative of f(x) = 1+x2 at x = 0 ?

Tesfaye G and Wanaw B Page 99


CALCULUS II

Solution: If we try to differentiate directly, we will be hopelessly bogged down at about the third
derivative; consequently we need another method by which we can get the sixth derivative of the
function and for this we have the Maclaurin series.
1
Since the Maclaurin series for = 1 − x + x2 − x3 + x4 − x5 + …
1+x
1
Then we have = 1 − x 2 + x 4 − x 6 + x 8 − x10 + …
1+x2
f6 (0).x6
Now = −x 6
6!

f 6 (0) = −6! = −720


4. If f n (0) = (n + 1)! for n = 1, 2, 3, … , then find the Maclaurin series for f.
Solution: f(x) = ∑∞
n=0 cn . (x − a)
n
but a = 0
fn (0) (n+1)!
⟹ f(x) = ∑∞
n=0 cn . x
n
where cn = = =n+1
n! n!

⟹ f(x) = ∑∞
n=0(n + 1) . x
n

Exercise 1

1. Find the Taylor polynomial up to degree 5 for


i. 𝑓(𝑥) = 𝑐𝑜𝑠 𝑥 center at 𝑎 = 0
1
ii. 𝑓(𝑥) = 𝑥 center at 𝑎 = 1
2. Let 𝑓(𝑥) = 𝑥 6 − 3𝑥 4 + 2𝑥 − 1
a. Find the fifth Taylor polynomial of 𝑓 about 0 ?
b. Find the fourth Taylor polynomial of 𝑓 about -1 ?
c. Find the Taylor series of 𝑓 about -1 ?

Chapter 4
Tesfaye G and Wanaw B Page 100
CALCULUS II

4.1Real Sequence
Chapter objective: At the end of this chapter students will be able to:
 Define a real sequence.
 Define bounded and monotonic sequence.
 Determine whether a given sequence is bounded or not.
 Determine whether a given sequence is monotonic or not.
 Determine a convergence of a sequence.
 State limit laws of a convergent sequence.

4.1.1 Definition and Notation


At the end of this section students will be able to:
 Define a real sequence.

A sequence can be thought as a list of numbers written a defined order


𝑎1 , 𝑎2 , 𝑎3 , … , 𝑎𝑛 , …
The number 𝑎1 is the first term, 𝑎2 is the second term and in general 𝑛 is the 𝑛th term.
Definition 4.1.1: A function that takes on real values and is defined on a set of positive integers
is called a sequence of real numbers or real sequence.
That is, a function 𝑎: ℕ ⟶ ℝ define by 𝑎(𝑛) = 𝑎𝑛 and then the sequence 𝑎1 , 𝑎2 , 𝑎3 , … , 𝑎𝑛 , … is
a real sequence, where 𝑎1 , 𝑎2 , 𝑎3 , … , 𝑎𝑛 are real numbers associated 1,2,3, … , 𝑛 mapping. That
means a real sequence is a sequence whose terms are all real numbers.
Example 4.1.1: A function defined on a set of positive integers by setting
a. 𝑎(𝑛) = 𝑛
𝑛+1
b. 𝑎(𝑛) = 𝑛
1
c. 𝑎(𝑛) = 𝑒 𝑛

are all sequence of real numbers.

Notation: The sequence 𝑎1 , 𝑎2 , 𝑎3 , … , 𝑎𝑛 , … or {𝑎1 , 𝑎2 , 𝑎3 , … , 𝑎𝑛 , … } in the notation form is


written as {𝑎𝑛 }∞
𝑛=1 or {𝑎𝑛 }.

Example 4.1.2:

Tesfaye G and Wanaw B Page 101


CALCULUS II

1 1 1 1 1
1. The sequence {1, 2 , 3 , 4 , 5 , 6 , … } is the sequence whose nth term is obtained by taking the
1 1 ∞ 1
reciprocal of n, that is 𝑎𝑛 = 𝑛. Hence, this sequence is denoted by {𝑛} or {𝑛}.
𝑛=1
−2 3 −4 5 −6 (−1)𝑛 (𝑛+1)
2. { 3 , 9 , 27 , 81 , 243 , … } is the sequence whose 𝑛th term is 𝑎𝑛 = . Hence, this
3𝑛

(−1)𝑛 (𝑛+1) (−1)𝑛−1 𝑛 ∞


sequence is denoted by { }, or we can also denote it as { } .
3𝑛 3𝑛−1 𝑛=2

3. {0,1, √2, √3, 2, √5, … } is the sequence whose 𝑛th term is 𝑎𝑛 = √𝑛 − 1. Hence, this

sequence is denoted by {√𝑛 − 1}, or we can also denote it as {√𝑛 − 3}𝑛=3 .

Exercise 4.1.1: Find the formula for the general term 𝑎𝑛 of the sequence and write its notation.
1 2 3 4 5
a. {2 , 3 , 4 , 5 , 6 , … }

b. {−1,1, −1,1, −1,1, … }


−1 4 −9 16 −25
c. { 2 , 3 , , , ,…}
4 5 6

4.1.1 Convergence of a Sequence


At the end of this section students will be able to:
 Determine a convergence of a sequence.

Definition 4.1.4: The sequence {𝑎𝑛 } convergence to 𝑙 (has a limit 𝑙) and we write
lim 𝑎𝑛 = 𝑙 𝑜𝑟 𝑎𝑛 → 𝑙 𝑎𝑠 𝑛 → ∞
𝑛→∞

if it is possible to make the terms 𝑎𝑛 as close as 𝑙 we like by taking 𝑛 sufficiently large.


If the sequence {𝑎𝑛 } has a finite limit (lim𝑛→∞ 𝑎𝑛 < ∞) we say the sequence converges (or
convergent). Otherwise we say the sequence {𝑎𝑛 } diverges (divergent).
1
Example 4.1.5: Consider the sequence {2𝑛} .
1
lim =0
𝑛→∞ 2𝑛
1
Hence, the sequence {2𝑛 } is convergent sequence and it converges to 0.
𝜋
Example 4.1.6: Consider the sequence {𝑠𝑖𝑛 (2𝑛)} .

𝜋
lim (𝑠𝑖𝑛 ( )) = 0
𝑛→∞ 2𝑛

Tesfaye G and Wanaw B Page 102


CALCULUS II

𝜋
Hence, the sequence {𝑠𝑖𝑛 (2𝑛)} is convergent sequence and it converges to 0.
4𝑛+3
Example 4.1.7: Consider the sequence {6𝑛+5} .
4𝑛 + 3 2
lim =
𝑛→∞ 6𝑛 + 5 3
4𝑛+3 2
Hence, the sequence {6𝑛+5} is convergent sequence and it converges to 3.
𝑛2 −1
Example 4.1.8: Consider the sequence { 𝑛+1 } .

𝑛2 − 1
lim = lim (𝑛 − 1) = ∞
𝑛→∞ 𝑛 + 1 𝑛→∞
4𝑛+3
Hence, the sequence {6𝑛+5} is not convergent sequence that is, it is divergent sequence.

Remark:
1. If lim𝑥→∞ 𝑓(𝑥) = 𝑙 and 𝑓(𝑛) = 𝑎𝑛 , where 𝑛 is an integer, then lim𝑛→∞ 𝑎𝑛 = 𝑙.
2. If {𝑎𝑛 } is a sequence whose denominator and numerator approach infinity as 𝑛 → ∞ we
can’t apply L’Hopitels rule directly because it applies not to the sequence but to the
function of real variable. However, we can apply L’Hopitels rule to the related real
variable function 𝑓(𝑥) where 𝑓(𝑛) = 𝑎𝑛 .

𝑙𝑛(𝑛)
Example 4.1.9: Determine whether the sequence { } converges or not.
2𝑛
𝑙𝑛(𝑛)
Solution: The denominator and numerator of the sequence { } approach infinity as 𝑛 → ∞.
2𝑛
𝑙𝑛(𝑥)
Thus, define a real variable function (𝑥) = . Then, using L’Hopitels rule we have
2𝑥
1
𝑙𝑛(𝑥) 1 1
lim 𝑓(𝑥) = lim 𝑥
= lim 𝑥𝑥 = lim 𝑥 = 0
𝑥→∞ 𝑥→∞ 2 𝑥→∞ 2 𝑙𝑛2 𝑙𝑛2 𝑥→∞ 𝑥2
Thus, by remark 1 above, we have
𝑙𝑛(𝑛)
lim =0
𝑛→∞ 2𝑛
𝑙𝑛(𝑛)
Hence, the sequence { } is convergent and it converges to 0.
2𝑛

To give the precise shape to the above definition we state:


Definition 4.1.5: A sequence {𝑎𝑛 } is said to converge to the number 𝑙, if given any 𝜀 > 0, there
exists a positive integer 𝑚 such that
|𝑎𝑛 − 𝑙| < 𝜀 , for all 𝑛 ≥ 𝑚.

Tesfaye G and Wanaw B Page 103


CALCULUS II

This number 𝑙 is called the limit of the sequence.


 It can be easily shown that the sequence can’t converge to more than one limit.

1
Example 4.1.10: Using the above definition (𝜀 − 𝑚 definition) show that the sequence {𝑛}

converges to zero.
Solution: Given 𝜀 > 0, we want to show there exists 𝑚 ∈ ℤ+ such that
1
|𝑛 − 0| < 𝜀 , for all 𝑛 ≥ 𝑚.

Thus,
1 1 1 1 1
|𝑛 − 0| < 𝜀 ⟹ |𝑛| = 𝑛 < 𝜀 ⟹ 𝑛 < 𝜀 ⟹ 𝑛>𝜀
1
So we can choose 𝑚 ∈ ℤ+ such that 𝑚 > 𝜀 .
10 1 98
In particular, let us take = 98 . Then, we choose 𝑚 to be > 𝜀 = 10 = 9.8 , that is, we can take

𝑚 = 10.
Thus,
1 10
|𝑛 − 0| < 98 , for all 𝑛 ≥ 10.

So we find that the values of 𝑚 depends upon the value of 𝜀. The smaller the value of 𝜀 the
larger the value of 𝑚.
Exercise 4.1.3:Using 𝜀 − 𝑚 definition show that
𝑛 1
a. lim𝑛→∞ 4𝑛+1 = 4
1
b. lim𝑛→∞ 𝑛−1 = 0

Remark:
1. Every bounded and monotonic sequence is convergent.
2. Every convergent sequence is bounded.
3. Every unbounded sequence is divergent.

Let us prove remark 2 above.


Assume the sequence {𝑎𝑛 } is convergent and 𝑎𝑛 → 𝑙 (lim𝑛→∞ 𝑎𝑛 = 𝑙 < ∞).
Thus, for 𝜀 = 1, there exists a positive integer m such that
|𝑎𝑛 − 𝑙| < 1 , for all 𝑛 ≥ 𝑚

Tesfaye G and Wanaw B Page 104


CALCULUS II

⟹ |𝑎𝑛 | − |𝑙| ≤ |𝑎𝑛 − 𝑙| < 1 , for all 𝑛 ≥ 𝑚


⟹ |𝑎𝑛 | < 1 + |𝑙| , for all 𝑛 ≥ 𝑚
Thus, choose = 𝑚𝑎𝑥{|𝑎1 |, |𝑎2 |, … , |𝑎𝑚−1 |, 1 + |𝑙|} , and thus we have
|𝑎𝑛 | ≤ 𝑀 , for all 𝑛
⟹ −𝑀 ≤ 𝑎𝑛 ≤ 𝑀 , for all 𝑛
Therefore, this proves that the sequence {𝑎𝑛 } is bounded.
4.1.2 Limit laws for convergent sequence
At the end of this chapter students will be able to:
 State limit laws of a convergent sequence

Let {𝑎𝑛 } and {𝑏𝑛 } are convergent sequences and let 𝑐 be constant, then
𝑖. lim (𝑎𝑛 + 𝑏𝑛 ) = lim 𝑎𝑛 + lim 𝑏𝑛
𝑛→∞ 𝑛→∞ 𝑛→∞

𝑖𝑖. lim (𝑎𝑛 − 𝑏𝑛 ) = lim 𝑎𝑛 − lim 𝑏𝑛


𝑛→∞ 𝑛→∞ 𝑛→∞

𝑖𝑖𝑖. lim 𝑐𝑎𝑛 = 𝑐 lim 𝑎𝑛


𝑛→∞ 𝑛→∞

𝑖𝑣. lim 𝑎𝑛 𝑏𝑛 = lim 𝑎𝑛 lim 𝑏𝑛


𝑛→∞ 𝑛→∞ 𝑛→∞

lim 𝑎𝑛
𝑎𝑛 𝑛→∞
𝑣. If lim 𝑏𝑛 ≠ 0, then lim =
𝑛→∞ 𝑛→∞ 𝑏𝑛 lim 𝑏𝑛
𝑛→∞
𝑝
𝑣𝑖. lim 𝑎𝑛 𝑝 = ( lim 𝑎𝑛 ) , if 𝑝 > 0 and 𝑎𝑛 > 0
𝑛→∞ 𝑛→∞

Theorem 4.1: Squeezing Theorem


Let {𝑎𝑛 }, {𝑏𝑛 } and {𝑐𝑛 } be real sequences such that
𝑎𝑛 ≤ 𝑏𝑛 ≤ 𝑐𝑛 , for all 𝑛 sufficiently large (for 𝑛 ≥ 𝑛0 )
If 𝑎𝑛 ⟶ 𝑙 and 𝑐𝑛 ⟶ 𝑙 (lim𝑛→∞ 𝑎𝑛 = 𝑙 = lim𝑛→∞ 𝑐𝑛 ), then 𝑏𝑛 ⟶ 𝑙 (lim𝑛→∞ 𝑏𝑛 = 𝑙).
𝑠𝑖𝑛(𝜋𝑛)
Example 4.1.11: Consider the sequence { }.
𝑛

Since −1 ≤ 𝑠𝑖𝑛(𝜋𝑛) ≤ 1 , for all , we have


−1 𝑠𝑖𝑛(𝜋𝑛) 1
≤ ≤
𝑛 𝑛 𝑛
Thus,
−1 1
⟶0 and ⟶0
𝑛 𝑛
Hence,

Tesfaye G and Wanaw B Page 105


CALCULUS II

𝑠𝑖𝑛(𝜋𝑛) 𝑠𝑖𝑛(𝜋𝑛)
⟶0 or lim =0
𝑛 𝑛→∞ 𝑛
𝑠𝑖𝑛(𝜋𝑛)
That is, the sequence { } converges to 0.
𝑛
𝑛!
Example 4.1.12: Determine whether the sequence {𝑛𝑛} converges or diverges.
𝑛!
Solution: Observe that the nth term of the sequence is 𝑎𝑛 = 𝑛𝑛.

Then,
1! 2! 2 1
𝑎1 = , 𝑎2 = = .
11 22 2 2
and
𝑛! 𝑛 𝑛−1 𝑛−2 2 1
𝑎𝑛 = 𝑛
=( ) ( )( )…( ) ( )
𝑛 𝑛 𝑛 𝑛 𝑛 𝑛
Notice that
𝑛 𝑛−1 𝑛−2 2 (𝑛 − 1)(𝑛 − 2) … 2
( )( )( )…( ) =
𝑛 𝑛 𝑛 𝑛 𝑛. 𝑛. 𝑛 … 𝑛
This is at most 1.
𝑛 𝑛−1 𝑛−2 2
That is, (𝑛) ( )( ) … (𝑛) ≤ 1, because the numerator is less than or equal to the
𝑛 𝑛

denominator.
So,
𝑛 𝑛−1 𝑛−2 2
0≤( ) ( )( )…( ) ≤ 1
𝑛 𝑛 𝑛 𝑛
𝑛 𝑛−1 𝑛−2 2 1 1
⟹ 0 ≤ ( )( )( )…( )( ) ≤
𝑛 𝑛 𝑛 𝑛 𝑛 𝑛
𝑛! 1
⟹ 0≤ 𝑛≤ , for all 𝑛
𝑛 𝑛
But,
1
lim 0 = 0 = lim
𝑛→∞ 𝑛→∞ 𝑛

Therefore, by squeezing Theorem we have


𝑛!
lim =0
𝑛→∞ 𝑛𝑛
𝑛!
Hence, the sequence {𝑛𝑛 } converges to 0.

Exercise 4.1.4: State whether or not the sequence converges and, if it does, find the limit

Tesfaye G and Wanaw B Page 106


CALCULUS II

2𝑛2 +𝑠𝑖𝑛(𝜋𝑛) 2𝑛
a. { } g. {𝑙𝑛 𝑛=1}
𝑛3 +1

3+5𝑛2 1 2𝑛
b. { 𝑛2 +𝑛 } h. {(1 + 𝑛) }

√𝑛 1 𝑛
c. {1+ 𝑛} i. {(1 + 𝑛) }

4.1.3 Bounded and monotonic Sequence


At the end of this section students will be able to:
 Define bounded and monotonic sequence.
 Determine whether a given sequence is bounded or not.
 Determine whether a given sequence is monotonic or not.

Definition 4.6:
i. A sequence {𝑎𝑛 } is bounded above if there exists a number 𝑀 such that
𝑎𝑛 ≤ 𝑀 , for all 𝑛.
ii. A sequence {𝑎𝑛 } is bounded below if there exists a number 𝑚 such that
𝑚 ≤ 𝑎𝑛 , for all 𝑛.
iii.A sequence {𝑎𝑛 } is said to be bounded if it is bounded above and below.

A sequence {𝑎𝑛 } is bounded if there exists a numbers 𝑀 and 𝑚 such that


𝑚 ≤ 𝑎𝑛 ≤ 𝑀 , for all 𝑛.
Example 4.1.13:
1
1. The sequence {𝑛} is bounded below by 0 and above by 1. That is
1
0 ≤ 𝑛 ≤ 1 , for all 𝑛.
1
Therefore, the sequence {𝑛} is bounded sequence.
𝑛
2. The sequence { } is bounded below by 0 and above by 1. That is
𝑛+1
𝑛
0 ≤ 𝑛+1 ≤ 1 , for all 𝑛.
𝑛
Therefore, the sequence {𝑛+1} is bounded sequence.
3𝑛+2
3. The sequence { 𝑛+1 } is bounded sequence because
3𝑛+2
2≤ ≤ 3 , for all 𝑛.
𝑛+1

Tesfaye G and Wanaw B Page 107


CALCULUS II

4. The sequence {𝑛} is bounded below by 1 but it is not bounded above. Therefore, {𝑛} is
not bounded sequence.

Definition 4.7: A sequence {𝑎𝑛 } is said to be


i. Increasing sequence iff 𝑎𝑛 < 𝑎𝑛+1 , for all 𝑛.
ii. Non decreasing sequence iff 𝑎𝑛 ≤ 𝑎𝑛+1 , for all 𝑛.
iii. Decreasing sequence iff 𝑎𝑛 > 𝑎𝑛+1 , for all 𝑛.
iv. Non increasing sequence iff 𝑎𝑛 ≥ 𝑎𝑛+1 , for all 𝑛.

A monotonic sequence is a sequence is a sequence which satisfies any of those four properties.
That is the sequence {𝑎𝑛 } is said to be monotonic sequence if it is either increasing or non
decreasing or decreasing or decreasing or non increasing sequence.
Example 4.1.14:
1
1. The sequence {𝑛} is a decreasing sequence. Hence, it is monotonic sequence.

2. The sequence {2−𝑛 } is a decreasing sequence. Hence, it is monotonic sequence.


3. The sequence {1,1,3,3,5,5,7,7,9,9,11,11, … } in non decreasing sequence. Hence it is
monotonic sequence.
4. The sequence {(−1)𝑛 } is a sequence which is not increasing, not non decreasing, not
decreasing, not non increasing. Hence, it is not monotonic sequence.
1 ∞
5. The sequence {3−𝑛} is an increasing sequence. Hence, it is monotonic sequence.
𝑛=4

Exercise 4.1.5: Determine whether the following sequences are monotonic or not and bounded
or not.
4 (−1)𝑛
a. {𝑛+2} f. { 𝑛+1 }

𝑛 1
b. {𝑛+3} g. {√4 − 𝑛}

2𝑛 −1 6𝑛+10
c. { } h. { 2𝑛+4 }
2𝑛
2𝑛2 +𝑛−1 1 1
d. {𝑛 − } i. {𝑛 − 𝑛+1}
𝑛+1
1 ∞
e. {𝑛−3} j. {𝑠𝑖𝑛(𝑛𝜋)}
𝑛=3

Tesfaye G and Wanaw B Page 108


CALCULUS II

Chapter summary
 A sequence can be thought as a list of numbers written a defined order

𝑎1 , 𝑎2 , 𝑎3 , … , 𝑎𝑛 , …

Tesfaye G and Wanaw B Page 109


CALCULUS II

The number 𝑎1 is the first term, 𝑎2 is the second term and in general 𝑛 is the 𝑛th term.
 A sequence {𝑎𝑛 } is bounded above if there exists a number 𝑀 such that
𝑎𝑛 ≤ 𝑀 , for all 𝑛.
 A sequence {𝑎𝑛 } is bounded below if there exists a number 𝑚 such that
𝑚 ≤ 𝑎𝑛 , for all 𝑛.
 A sequence {𝑎𝑛 } is said to be bounded if it is bounded above and below. That is, sequence
{𝑎𝑛 } is bounded if there exists a numbers 𝑀 and 𝑚 such that
𝑚 ≤ 𝑎𝑛 ≤ 𝑀 , for all 𝑛.
 A sequence {𝑎𝑛 } is said to be
v. Increasing sequence iff 𝑎𝑛 < 𝑎𝑛+1 , for all 𝑛.
vi. Non decreasing sequence iff 𝑎𝑛 ≤ 𝑎𝑛+1 , for all 𝑛.
vii. Decreasing sequence iff 𝑎𝑛 > 𝑎𝑛+1 , for all 𝑛.
viii. Non increasing
sequence iff 𝑎𝑛 ≥ 𝑎𝑛+1 , for all 𝑛.
A monotonic sequence is a sequence is a sequence which satisfies any of those
four properties.
 The sequence {𝑎𝑛 } convergence to 𝑙 (has a limit 𝑙) and we write

lim 𝑎𝑛 = 𝑙 𝑜𝑟 𝑎𝑛 → 𝑙 𝑎𝑠 𝑛 → ∞
𝑛→∞

if it is possible to make the terms 𝑎𝑛 as close as 𝑙 we like by taking 𝑛 sufficiently large.


If the sequence {𝑎𝑛 } has a finite limit (lim𝑛→∞ 𝑎𝑛 < ∞) we say the sequence
converges (or convergent). Otherwise we say the sequence {𝑎𝑛 } diverges
(divergent).
 A sequence {𝑎𝑛 } is said to converge to the number 𝑙, if given any 𝜀 > 0, there exists a
positive integer 𝑚 such that
|𝑎𝑛 − 𝑙| < 𝜀 , for all 𝑛 ≥ 𝑚.
This number 𝑙 is called the limit of the sequence.
 Every bounded and monotonic sequence is convergent.
 Every convergent sequence is bounded.
 Every unbounded sequence is divergent.
 Let {𝑎𝑛 } and {𝑏𝑛 } are convergent sequences and let c be constant, then

Tesfaye G and Wanaw B Page 110


CALCULUS II

𝑖. lim (𝑎𝑛 + 𝑏𝑛 ) = lim 𝑎𝑛 + lim 𝑏𝑛


𝑛→∞ 𝑛→∞ 𝑛→∞

𝑖𝑖. lim (𝑎𝑛 − 𝑏𝑛 ) = lim 𝑎𝑛 − lim 𝑏𝑛


𝑛→∞ 𝑛→∞ 𝑛→∞

𝑖𝑖𝑖. lim 𝑐𝑎𝑛 = 𝑐 lim 𝑎𝑛


𝑛→∞ 𝑛→∞

𝑖𝑣. lim 𝑎𝑛 𝑏𝑛 = lim 𝑎𝑛 lim 𝑏𝑛


𝑛→∞ 𝑛→∞ 𝑛→∞

lim 𝑎𝑛
𝑎𝑛 𝑛→∞
𝑣. If lim 𝑏𝑛 ≠ 0, then lim =
𝑛→∞ 𝑛→∞ 𝑏𝑛 lim 𝑏𝑛
𝑛→∞
𝑝
𝑣𝑖. lim 𝑎𝑛 𝑝 = ( lim 𝑎𝑛 ) , if 𝑝 > 0 and 𝑎𝑛 > 0
𝑛→∞ 𝑛→∞

Review Exercise
1. Find the formula for the general term 𝑎𝑛 of the sequence and write its notation.
1 4 9 16 25
d. {4 , 9 , 16 , 25 , 36 , … }

e. {2, −2,2, −2,2, −2,2, … }


Tesfaye G and Wanaw B Page 111
CALCULUS II

−1 3 −5 7 −9 11
f. { 5 , 10 , 17 , 26 , 37 , 50 , … }

2. Determine whether the following sequences are monotonic or not and bounded or not.
2𝑛2 +𝑛−1 1 1
a. {𝑛 − } c. {𝑛 − 𝑛+1}
𝑛+1
1 ∞
b. {𝑛−3} d. {𝑠𝑖𝑛(𝑛𝜋)}
𝑛=3
2𝑛+5
3. Using 𝜀 − 𝑚 definition convergence of a sequence show that the sequence { }
𝑛
2𝑛+5
converges to 2. That is, lim𝑛→∞ =2
𝑛

4. State whether or not the sequence converges and, if it does, find the limit
𝑛 𝑐𝑜𝑠2 𝑛
a. { 𝑛 } d. { }
2 2𝑛

√𝑛+1 1 2𝑛
b. { 2 } e. {𝑛+1 − 𝑛+4}
√𝑛
2𝑛 −1
c. { }
2𝑛

5. Show that

2𝑛
⟶0
𝑛!
2𝑛
That is sequence { 𝑛! } converges to 0.

4.2 Infinite Series


Chapter objective: At the end of this chapter students will be able to:
 Define real series.
 Discuss the concept of convergent series, divergent series and oscilatory series using
𝑛th partial sum.

Tesfaye G and Wanaw B Page 112


CALCULUS II

 Define and distinguish positive term series.


 State Integral Test, Comparison Test, Limit comparison Test, Ratio Test and Root Test
of a positive term series.
 Determine a convergence of a positive term series.
 Define alternating series.
 Define absolute convergence of a series.
 State generalized convergence tests.

4.2.1 Definition and examples of Series


At the end of this section students will be able to:
 Define real series.
 Discuss the concept of convergent series, divergent series and oscilatory series using
𝑛th partial sum.

Definition 4.2.1: If {𝑎𝑛 } is sequence of real numbers, then the expression that is the sum of the
sequence which is an infinite in number
𝑎1 + 𝑎2 + ⋯ + 𝑎𝑛 + ⋯
is called an infinite series or simply series.
Notation: An infinite series (series)
𝑎1 + 𝑎2 + ⋯ + 𝑎𝑛 + ⋯
Is denoted by the symbol ∑∞
𝑛=1 𝑎𝑛 or ∑ 𝑎𝑛 .

Example 4.2.1:
i. 1 + 2 + 3 + 4 + 5 + ⋯ is a series and 1 + 2 + 3 + 4 + 5 … = ∑∞
𝑛=1 𝑛 = ∑ 𝑛.
1 4 9 16 25 36 1 4 9 16 25 36
ii. +3+4+ + + + ⋯ is a series and +3+4+ + + +⋯=
2 5 6 7 2 5 6 7
𝑛2 𝑛2
∑∞
𝑛=1 = ∑ 𝑛+1.
𝑛+1

Suppose we are given a series ∑∞


𝑛=1 𝑎𝑛 = 𝑎1 + 𝑎2 + ⋯ + 𝑎𝑛 + ⋯ Let us denote the sum the first

𝑛 terms of the sequence {𝑎𝑛 } by 𝑆𝑛 .


That is
𝑆1 = 𝑎1
𝑆2 = 𝑎1 + 𝑎2
𝑆3 = 𝑎1 + 𝑎2 + 𝑎3

𝑆𝑛 = 𝑎1 + 𝑎2 + ⋯ + 𝑎𝑛

Tesfaye G and Wanaw B Page 113


CALCULUS II

 For the series ∑∞


𝑛=1 𝑎𝑛 , 𝑆𝑛 (the sum of the first 𝑛 terms of the sequence {𝑎𝑛 }) which is

𝑆𝑛 = ∑ 𝑎𝑛 = 𝑎1 + 𝑎2 + ⋯ + 𝑎𝑛
𝑖=1

is called the 𝑛th partial sum of the series ∑∞


𝑛=1 𝑎𝑛 .

 Let 𝑆𝑛 be the nth partial sum of the series ∑∞


𝑛=1 𝑎𝑛 . then, the arrangement or list of

numbers
𝑆1 , 𝑆2 , 𝑆3 , … , 𝑆𝑛 , …
is a real sequence which is called the sequence of partial sums. That is, the 𝑛th partial
sum of the series form a sequence ({𝑆𝑛 } is a sequence).
Definition 4.2.2: Given a series ∑∞
𝑛=1 𝑎𝑛 = 𝑎1 + 𝑎2 + ⋯ + 𝑎𝑛 + ⋯ and let 𝑆𝑛 denotes the

partial sum of the given series:


𝑛

𝑆𝑛 = ∑ 𝑎𝑛 = 𝑎1 + 𝑎2 + ⋯ + 𝑎𝑛
𝑖=1

If the sequence {𝑆𝑛 } is convergent and lim𝑛→∞ 𝑆𝑛 = 𝑙 , then the series ∑∞


𝑛=1 𝑎𝑛 is called

convergent series and we write


∑ 𝑎𝑛 = 𝑎1 + 𝑎2 + ⋯ + 𝑎𝑛 + ⋯ = 𝑙
𝑛=1

The number 𝑠 is called the sum of the series.


Moreover, if the sequence {𝑆𝑛 } is divergent, then the series ∑∞
𝑛=1 𝑎𝑛 is divergent.

Note that:
∞ 𝑛

∑ 𝑎𝑛 = lim ∑ 𝑎𝑛 = lim 𝑆𝑛
𝑛→∞ 𝑛→∞
𝑛=1 𝑖=1

Convergence, divergence and oscillation of an infinite series:


Let 𝑆𝑛 be the 𝑛th partial sum of the series ∑∞
𝑛=1 𝑎𝑛 and let lim𝑛→∞ 𝑆𝑛 = 𝑙. Then,

i. If 𝑙 is finite real number, then the series ∑∞


𝑛=1 𝑎𝑛 is convergent.

ii. If 𝑙 = ±∞, then the series ∑∞


𝑛=1 𝑎𝑛 is divergent.

iii. If 𝑙 is not unique, then the series ∑∞


𝑛=1 𝑎𝑛 is non-convergent (oscillatory).

Example 4.2.2: Test the convergence of the series

Tesfaye G and Wanaw B Page 114


CALCULUS II


1

2𝑛
𝑛=1
1
Solution: The first 𝑛 terms of the sequence ∑∞
𝑛=1 2𝑛 is

1 1 1
𝑆𝑛 = + +⋯+ 𝑛
2 4 2
Since the series is the geometric series thus,
1 1 𝑛
(1 − (
2 2) ) 1
𝑆𝑛 = =1− 𝑛
1 2
1−
2
Thus,
1
lim 𝑆𝑛 = lim (1 − )=1
𝑛→∞ 𝑛→∞ 2𝑛
Therefore, the series is convergent.
Example 4.2.3: Test the convergence of the series

∑𝑛
𝑛=1

Solution: The first 𝑛 terms of the sequence ∑∞


𝑛=1 𝑛 is

𝑆𝑛 = 1 + 2 + ⋯ + 𝑛
Since the series is the arithmetic series thus,
𝑛 𝑛2 + 𝑛
𝑆𝑛 = (1 + 𝑛) =
2 2
Thus,
𝑛2 + 𝑛
lim 𝑆𝑛 = lim ( )=∞
𝑛→∞ 𝑛→∞ 2
Therefore, the series is divergent.
Exercise 4.2.1: Show that geometric series

∑ 𝑎𝑟 𝑛−1 , 𝑎≠0
𝑛=1

is convergent, if −1 < 𝑟 < 1


divergent, if 𝑟 ≥ 1
oscillatory, if 𝑟 ≤ −1.

Tesfaye G and Wanaw B Page 115


CALCULUS II

Note: A finite number of terms does not affect the convergence or the divergence of a sequence.
For instance, suppose that we were able to show that the series

𝑛

𝑛3 + 1
𝑛=4

is convergent. Since
∞ ∞
𝑛 1 2 3 𝑛
∑ 3 = + + +∑ 3
𝑛 + 1 2 9 28 𝑛 +1
𝑛=1 𝑛=4
𝑛
It follows that the entire series ∑∞
𝑛=1 𝑛3 +1 is convergent.

Similarly, if it is known that the series


∑ 𝑎𝑛
𝑛=𝑁+1

converges, then the full series


∞ 𝑁 ∞

∑ 𝑎𝑛 = ∑ 𝑎𝑛 + ∑ 𝑎𝑛
𝑛=1 𝑛=1 𝑛=𝑁+1

is convergent.
Positive term Series
At the end of this section students will be able to:
 Define and distinguish positive term series.

A series ∑∞
𝑛=1 𝑎𝑛 in which all terms (𝑎𝑖 ′𝑠) are positive is called positive term series.

Example 4.2.4:
2𝑛2 +4
i. The series ∑∞
𝑛=1 is a positive term series.
𝑛+5
(−1)𝑛
ii. The series ∑∞
𝑛=1 is not a positive term series.
𝑛

We have defined the convergence of a series in terms of its sequence of partial sum. In general
practice it is found that it may not be always easy to write the sequence of partial sums. To avoid
this we have a few terms which can be applied directly to the positive term series to see whether
it converges or diverges. Before giving the list of thus tests, we give below a few important
limits which will be useful in application of those tests.
Some important limits

Tesfaye G and Wanaw B Page 116


CALCULUS II

1 𝑛
𝑎. lim (1 + ) = 𝑒
𝑛→∞ 𝑛
1
𝑏. lim 𝑛𝑛 = 1
𝑛→∞

𝑐. lim 𝑙𝑛(𝑛) = ∞
𝑛→∞

𝑑. lim 𝑙𝑛(𝑛) = −∞
𝑛→0

4.2.2 Convergence of a positive term series


At the end of this chapter students will be able to:
 State Integral Test, Comparison Test, Limit comparison Test, Ratio Test and Root Test
of a positive term series.
 Determine a convergence of a positive term series.

Suppose we have a positive term series ∑∞


𝑛=1 𝑎𝑛 then the following are some of the preliminary

tests that can be applied to determine the nature of the series.


Test 1: If lim𝑛→∞ 𝑎𝑛 ≠ 0 or does not exist, then the series ∑∞
𝑛=1 𝑎𝑛 diverges.

Test 2: The series ∑∞


𝑛=1 𝑎𝑛 is convergent if and only if there exists a number 𝐾 such that

𝑆𝑛 = 𝑎1 + 𝑎2 + ⋯ + 𝑎𝑛 < 𝐾 , for all 𝑛.


Test 3: The series ∑∞
𝑛=1 𝑎𝑛 diverges, if each term after a fixed stage is greater than some fixed

positive number.
Example 4.2.5: Show that the series

𝑛

𝑛+1
𝑛=1

is divergent.
𝑛
Solution: The 𝑛th term of the series is 𝑎𝑛 = 𝑛+1. Then,
𝑛
lim 𝑎𝑛 = lim =1
𝑛→∞ 𝑛→∞ 𝑛 + 1
𝑛
Therefore, by test 1 the series ∑∞
𝑛=1 𝑛+1 diverges.

Example 4.2.6: Test for the convergence of the series



1

3𝑛
𝑛=1

Tesfaye G and Wanaw B Page 117


CALCULUS II

1 1
Solution: The series ∑∞
𝑛=1 3𝑛 is the geometric series with common ratio = 3 . Thus, the partial

sum 𝑆𝑛 is given by
1 1 𝑛 1 1 𝑛+1 1
(1 − ( ) ) − (
1 1 1
𝑆𝑛 = + + ⋯ + 𝑛 =
3 3
=3 3) ⁄
< 3 =
1
3 9 3 1 2 2⁄ 2
1−3 3
3
1
Thus, for 𝐾 = 2
1 1 1
𝑆𝑛 = + + ⋯+ 𝑛 < 𝐾 , for all 𝑛
3 9 3
1
Therefore, by test 2 the series ∑∞
𝑛=1 3𝑛 is convergent.

Exercise 4.2.2:
1. Test for the convergence of the series

∞ ∞ ∞
𝑛 𝑛
4𝑛2 + 2
𝑎. ∑(1) 𝑏. ∑2 𝑐. ∑
3𝑛5 + 𝑛4 + 7𝑛 + 2
𝑛=1 𝑛=1 𝑛=1

2. Let 𝑟 > 0. For what value of 𝑟 the geometric series



1

𝑟 𝑛−1
𝑛=1

is convergent.
4.2.4 Convergence tests for positive series
Integral Test and Comparison Test
Integral Test
Suppose {𝑎𝑛 } be a sequence and 𝑓 is a continuous positive, decreasing function on [1, ∞) and let
𝑓(𝑛) = 𝑎𝑛 . Then,
∞ 𝑥
i. If ∫1 𝑓(𝑥)𝑑𝑥 = lim𝑥→∞ ∫1 𝑓(𝑥)𝑑𝑥 converges, then the series ∑∞
𝑛=1 𝑎𝑛 is convergent.
∞ 𝑥
ii. If ∫1 𝑓(𝑥)𝑑𝑥 = lim𝑥→∞ ∫1 𝑓(𝑥)𝑑𝑥 diverges, then the series ∑∞
𝑛=1 𝑎𝑛 is divergent.

Example 4.2.7: Determine whether the series



4

𝑛+5
𝑛=1

converges or diverges.

Tesfaye G and Wanaw B Page 118


CALCULUS II

4
Solution: Define a real variable function 𝑓(𝑥) = 𝑥+5.

Thus, this function 𝑓 is continuous on [1, ∞) and 𝑓(𝑥) > 0, for all 𝑥 ∈ [1, ∞), that is 𝑓 is
positive.
Checking 𝑓 is decreasing,
−4
𝑓 ′ (𝑥) =
𝑥+5
Since 𝑓 ′ (𝑥) < 0, for all 𝑥 ∈ [1, ∞), 𝑓 is decreasing function on [1, ∞).
Thus, 𝑓 is continuous, positive and decreasing function on [1, ∞). So, we can apply integral test
∞ 𝑥 𝑥
−4
∫ 𝑓(𝑥)𝑑𝑥 = lim ∫ 𝑓(𝑥)𝑑𝑥 = lim ∫ 𝑑𝑥
1 𝑥→∞ 1 𝑥→∞ 1 𝑥+5
= 4 lim [𝑙𝑛(𝑥 + 5) + 𝑙𝑛6]
𝑥→∞

= 4𝑙𝑛6 + 4 lim 𝑙𝑛(𝑥 + 5) = ∞


𝑥→∞
∞ 4
Since the improper integral ∫1 𝑓(𝑥)𝑑𝑥 is divergent, the series ∑∞
𝑛=1 𝑛+5 diverges by integral test.

Note:
1. When we use an integral test it is not necessary to start the series or the integral at 𝑛 = 1.
That is, if {𝑎𝑛 }∞
𝑛=𝑁 be a sequence and 𝑓 is a continuous positive, decreasing function on

[𝑁, ∞) and let 𝑓(𝑛) = 𝑎𝑛 . Then, for the series


∞ 𝑥
∑∞
𝑛=1 𝑎𝑛 we use ∫𝑁 𝑓(𝑥)𝑑𝑥 = lim𝑥→∞ ∫𝑁 𝑓(𝑥)𝑑𝑥

2. When we use integral test it is not necessary that 𝑓 be always decreasing. What is
important is that 𝑓 be ultimately decreasing, that is, decreasing for 𝑥 larger than some
number 𝑁. Then is ∑∞ ∞ ∞
𝑛=𝑁 𝑎𝑛 convergent, so is ∑𝑛=1 𝑎𝑛 convergent or ∑𝑛=𝑁 𝑎𝑛 divergent,

so is ∑∞
𝑛=1 𝑎𝑛 divergent.

Example 4.2.8: Determine whether the series



1

𝑛𝑙𝑛𝑛
𝑛=2

converges or diverges.
1
Solution: Define a real variable function 𝑓(𝑥) = . The function 𝑓 is continuous and positive
𝑥𝑙𝑛𝑥

for 𝑥 > 1(logarithmic function is continuous for 𝑥 > 1).


To check f is decreasing

Tesfaye G and Wanaw B Page 119


CALCULUS II

−(1 + 𝑙𝑛𝑥)
𝑓 ′ (𝑥) =
𝑥 2 (𝑙𝑛𝑥)2
1 1
Thus, 𝑓 ′ (𝑥) < 0 when 1 + 𝑙𝑛𝑥 > 0, that is for 𝑥 > 𝑒 . It follows that 𝑓 is decreasing for 𝑥 > 𝑒 .

Thus, 𝑓 is continuous, positive and decreasing for 𝑥 ≥ 2. So, we can apply integral test
∞ 𝑥 𝑥
1
∫ 𝑓(𝑥)𝑑𝑥 = lim ∫ 𝑓(𝑥)𝑑𝑥 = lim ∫ 𝑑𝑥
2 𝑥→∞ 2 𝑥→∞ 2 𝑥𝑙𝑛𝑥
= lim [𝑙𝑛(𝑙𝑛𝑥) − 𝑙𝑛(𝑙𝑛2)]
𝑥→∞

= −𝑙𝑛(𝑙𝑛2) + lim 𝑙𝑛(𝑙𝑛𝑥) = ∞


𝑥→∞
∞ 1
Since the improper integral ∫2 𝑓(𝑥)𝑑𝑥 diverges, therefore by integral test the series ∑∞
𝑛=2 𝑛𝑙𝑛𝑛 is

divergent.
Example 4.2.9: For what value of 𝑝 the series

1

𝑛𝑝
𝑛=1

is convergent? This series is called 𝑝-series.


1
Solution: Define a real variable function 𝑓(𝑥) = 𝑥 𝑝 .

Here 𝑓 is continuous and positive on [1, ∞). Then, checking f is decreasing function,
−𝑝𝑥 𝑝−1 −𝑝
𝑓 ′ (𝑥) = =
𝑥 2𝑝 𝑥 𝑝+1
Thus, 𝑓 ′ (𝑥) < 0, for all 𝑥 ∈ [1, ∞) if 𝑝 > 0. So we will see two cases.
Case 1: 𝑝 > 0
1
The function 𝑓(𝑥) = 𝑥 𝑝 is continuous, positive and decreasing function on [1, ∞). Thus,
∞ 𝑥 𝑥
1
∫ 𝑓(𝑥)𝑑𝑥 = lim ∫ 𝑓(𝑥)𝑑𝑥 = lim ∫ 𝑑𝑥
1 𝑥→∞ 1 𝑥→∞ 1 𝑥𝑝
But,
𝑥 𝑙𝑛𝑥 , 𝑖𝑓 𝑝 = 1
1
∫ 𝑝 𝑑𝑥 = { 1 1
1 𝑥 𝑝−1
+ , 𝑖𝑓 𝑝 > 1
𝑝𝑥 𝑝−1
Thus,
∞ ∞ 𝑥 , 𝑖𝑓 0 < 𝑝 ≤ 1
∫ 𝑓(𝑥)𝑑𝑥 = lim ∫ 𝑓(𝑥)𝑑𝑥 = { 1
1 𝑥→∞ 1 , 𝑖𝑓 𝑝 > 1
𝑝−1

Tesfaye G and Wanaw B Page 120


CALCULUS II

Implying that, the improper integral diverges for 0 < 𝑝 ≤ 1 and converges for 𝑝 > 1. Therefore,
1
by integral test, the series ∑∞
𝑛=1 𝑛𝑝 diverges for 0 < 𝑝 ≤ 1 and converges for 𝑝 > 1.

Case 2: 𝑝 ≤ 0
1
Thus, the nth term of the series is 𝑎𝑛 = 𝑛𝑝 .

Thus,
1 ∞ , 𝑖𝑓 𝑝 < 0
lim 𝑎𝑛 = lim ={
𝑛→∞ 𝑛→∞ 𝑛𝑝 1 , 𝑖𝑓 𝑝 = 0
1
Hence, by test 1 the series ∑∞
𝑛=1 𝑛𝑝 diverges for 𝑝 ≤ 0.

1
Therefore, the 𝑝-series ∑∞
𝑛=1 𝑛𝑝 converges for 𝑝 > 1 and diverges for 𝑝 ≤ 1.

Exercise 4.2.3: Determine whether the following series is convergent or divergent.



𝑛 1 1 1 1 1
𝑎. ∑ 𝑓. 2 + 9 + 6 + + + + + +⋯
𝑛2 + 1 16 25 36 49 64
𝑛=1
∞ ∞
1 𝑛
𝑏. ∑4 𝑔. ∑
√𝑛 2𝑛
𝑛=1 𝑛=1
∞ ∞
𝑛3 𝑙𝑛𝑛
𝑐. ∑ 𝑛 ℎ. ∑
𝑒 𝑛𝑝
𝑛=1 𝑛=1

𝑛+2
𝑑. ∑
𝑛2 + 𝑛
𝑛=1
3 4 5 6 7
𝑒. −10 + 8 + 17 + 11 + (5)(6) + (6)(7) + (7)(8)
+ (8)(9) + (9)(10)
+⋯

Comparison Test
Suppose that ∑∞ ∞
𝑛=1 𝑎𝑛 and ∑𝑛=1 𝑏𝑛 are two positive term series such that

𝑎𝑛 ≤ 𝑏𝑛 , for all 𝑛
Then,
i. If ∑∞ ∞
𝑛=1 𝑏𝑛 is convergent, then ∑𝑛=1 𝑎𝑛 is alse convergent.

ii. If ∑∞ ∞
𝑛=1 𝑎𝑛 is divergent, then ∑𝑛=1 𝑏𝑛 is alse divergent.

In using comparison test we must, of course, have some known series for thew purpose of
comparison. Most of the time we use one of these series.
 A 𝑝-series

Tesfaye G and Wanaw B Page 121


CALCULUS II


1

𝑛𝑝
𝑛=1

converges if 𝑝 > 1 and diverges for 𝑝 ≤ 1.


 A geometric series

∑ 𝑎𝑟 𝑛−1
𝑛=1

converges if |𝑟| < 1 and diverges if |𝑟| ≥ 1.


Example 4.2.10: Determine whether the series

𝑛

3𝑛5 + 2𝑛 + 1
𝑛=1

converges or diverges.
Solution: Observe that
𝑛 𝑛 1
< 5= 4
3𝑛5 + 2𝑛 + 1 3𝑛 3𝑛
because the left hand side has bigger denominator than the right side.
𝑛 1
So taking 𝑎𝑛 = 3𝑛5 +2𝑛+1 and 𝑏𝑛 = 3𝑛4 we have

𝑎𝑛 ≤ 𝑏𝑛 , for all 𝑛
1
Burt, the series ∑∞
𝑛=1 𝑛4 is a 𝑝-series with 𝑝 = 4 > 1 and is con vergent, hence the series

1
∑∞
𝑛=1 is convergent.
3𝑛4
𝑛
Therefore, by comparison test(i), the series ∑∞
𝑛=1 3𝑛5 +2𝑛+1 is convergent.

Example 4.2.11: Determine whether the series



𝑙𝑛𝑛

𝑛
𝑛=1

converges or diverges.

Solution: Observe that since 𝑙𝑛𝑛 > 1 for 𝑛 ≥ 3, we have


1 𝑙𝑛𝑛
< , for all 𝑛 ≥ 3
𝑛 𝑛
because the right hand side has bigger numerator than the right side for 𝑛 ≥ 3.

Tesfaye G and Wanaw B Page 122


CALCULUS II

1 1
Burt, the series ∑∞ ∞
𝑛=1 𝑛 is a 𝑝-series with 𝑝 = 1 ≤ 1 and hence the series ∑𝑛=1 𝑛 is divergent.
𝑛
Therefore, by comparison test(ii), the series ∑∞
𝑛=1 3𝑛5 +2𝑛+1 is divergent.

Exercise 4.2.4: Determine whether the series converges or diverges.



4𝑛 − 3
𝑎. ∑
10𝑛8 + 7
𝑛=1

𝑛2 + 2
𝑏. ∑
7𝑛5 + 14𝑛3 + 4
𝑛=1

Note: The terms of the series being tested must be smaller that those of the convergent series or
larger than the divergent series. If the terms are larger than the convergent series or smaller than
the divergent series, then the comparison test does not apply. Consider for instance, the series

1

5𝑛 − 2
𝑛=1

The inequality
1 1
𝑛
< 𝑛 , for all 𝑛
5 5 −2
1
is useless as far as the comparison test is concerned, because ∑∞
𝑛=1 is convergent and
5𝑛

1 1
𝑛
< 𝑛 , for all 𝑛
5 5 −2
1
But, we have a feeling that ∑∞
𝑛=1 5𝑛 −2 ought to be convergen because it is very similar to the
1 𝑛
convergent geometric series ∑∞
𝑛=1 (5) . In each case the following test can be used.
Limit comparison Test, Ratio Test and root Test
The Limit comparison Test
Suppose that ∑∞ ∞
𝑛=1 𝑎𝑛 and ∑𝑛=1 𝑏𝑛 are two positive term series. If
𝑎𝑛
lim =𝑐
𝑛→∞ 𝑏𝑛
where 𝑐 is a positive real number, then ∑∞ ∞
𝑛=1 𝑎𝑛 and ∑𝑛=1 𝑏𝑛 converges or diverges together.

Example 4.2.12: Test the convergence of the series



1

𝑛(𝑛 + 2)(𝑛 + 4)
𝑛=1
1
Solution: The 𝑛th term of the series is 𝑎𝑛 = 𝑛(𝑛+2)(𝑛+4) and

Tesfaye G and Wanaw B Page 123


CALCULUS II

1 1
𝑎𝑛 = =
𝑛(𝑛 + 2)(𝑛 + 4) 𝑛3 (1 + 2) (1 + 4)
𝑛 𝑛
1 ∞ ∞
Take 𝑏𝑛 = 𝑛3 , and thus both ∑𝑛=1 𝑎𝑛 and ∑𝑛=1 𝑏𝑛 are positive term series.
Then,
𝑎𝑛 1 1
= . 𝑛3 =
𝑏𝑛 𝑛3 (1 + 2) (1 + 4) 2 4
(1 + 𝑛) (1 + 𝑛)
𝑛 𝑛
Thus,
𝑎𝑛 1
lim = lim =1
𝑛→∞ 𝑏𝑛 𝑛→∞ 2 4
(1 + 𝑛) (1 + 𝑛)
1 1
But, ∑∞ ∞ ∞ ∞
𝑛=1 𝑏𝑛 = ∑𝑛=1 𝑛3 is a 𝑝-series with 𝑝 = 3 > 1, hence the series ∑𝑛=1 𝑏𝑛 = ∑𝑛=1 𝑛3 is
convergent series.
1
Therefore, by limit comparison test, the series ∑∞
𝑛=1 𝑛(𝑛+2)(𝑛+4) is also convergent series.
Example 4.2.13: Test the convergence of the series

1
∑ 2
𝑛=1
(1 + 5𝑛)3 (4 + 𝑛) ⁄3
Solution: The 𝑛th term of the series is
1
𝑎𝑛 = 2
(1 + 5𝑛)3 (4 + 𝑛) ⁄3
And
1 1 1
𝑎𝑛 = 2 = 2⁄ = 2⁄
3 3
(1 + 5𝑛)3 (4 + 𝑛) ⁄3 1 2 4 3 11 1 4 3
𝑛3 (𝑛 + 5) 𝑛 ⁄3 (𝑛 + 1) 𝑛 ⁄3 (𝑛 + 5) (𝑛 + 1)
1
Taking 𝑏𝑛 = 11⁄ , and thus both ∑∞ ∞
𝑛=1 𝑎𝑛 and ∑𝑛=1 𝑏𝑛 are positive term series.
𝑛 3
Then,
𝑎𝑛 1 11⁄ 1
= 2⁄ .𝑛 3 = 2⁄
𝑏𝑛 11⁄ 1 3
4 3 1 3
4 3
𝑛 3( + 5) (𝑛 + 1) (𝑛 + 5) (𝑛 + 1)
𝑛
Thus,
𝑎𝑛 1 1
lim
= lim 2 =
𝑛→∞ 𝑏𝑛 𝑛→∞ 3 ⁄3 5
1 4
( + 5) ( + 1)
𝑛 𝑛
1 11 1
But, ∑𝑛=1 𝑏𝑛 = ∑𝑛=1 11⁄ is a 𝑝-series with 𝑝 = 3 > 1, hence the series ∑∞
∞ ∞ ∞
𝑛=1 𝑏𝑛 = ∑𝑛=1 11
𝑛 3 𝑛 ⁄3
is convergent series.
1
Therefore, by limit comparison test, the series ∑∞
𝑛=1 2 is also convergent series.
(1+5𝑛)3 (4+𝑛) ⁄3
Example 4.2.14: Test the convergence of the series

3⁄ 3⁄
∑ √𝑛 2 +7−𝑛 4
𝑛=1
Solution: The 𝑛th term of the series is

Tesfaye G and Wanaw B Page 124


CALCULUS II

3⁄ 3⁄
𝑎𝑛 = √𝑛 2 +7−𝑛 4

And
3⁄ 3⁄ 3⁄ 3⁄
√𝑛3⁄2 + 7 + 𝑛3⁄4
√ 2 4 √
𝑎𝑛 = 𝑛 + 7 − 𝑛 = 𝑛 + 7 − 𝑛 .
2 4
√𝑛3⁄2 + 7 + 𝑛3⁄4
3 3
𝑛 ⁄2 + 7 − 𝑛 ⁄2
=
√𝑛3⁄2 + 7 + 𝑛3⁄4
7
=
√𝑛3⁄2 + 7 + 𝑛3⁄4
7
=
3 7
𝑛 ⁄4 (√1 + 3⁄ + 1)
𝑛 2
1
Taking 𝑏𝑛 = 3 , and thus both ∑∞ ∞
𝑛=1 𝑎𝑛 and ∑𝑛=1 𝑏𝑛 are positive term series.
𝑛 ⁄4
Then,
𝑎𝑛 7 3⁄ 7
= .𝑛 4 =
𝑏𝑛
3⁄ 7 7
𝑛 4 (√1 + 3 + 1) √1 + 3⁄ +1
𝑛 ⁄2 𝑛 2

Thus,
𝑎𝑛 7 7
lim = lim =
𝑛→∞ 𝑏𝑛 𝑛→∞ 2
7
√1 + 3⁄ +1
𝑛 2

3⁄ 3⁄ 1
Thus, the series ∑∞ √
𝑛=1 𝑛 2 +7−𝑛 4 and ∑∞
𝑛=1 3 converge and diverge together by limit
𝑛 ⁄4
comparison test.
∞ 1 3 ∞ 1
But, ∑∞
𝑛=1 𝑏𝑛 = ∑𝑛=1 3 is a 𝑝-series with 𝑝 = 4 ≤ 1, hence the series ∑∞
𝑛=1 𝑏𝑛 = ∑𝑛=1 3 is
𝑛 ⁄4 𝑛 ⁄4
divergent series.
Therefore, the series ∑∞ √ 3⁄ 3⁄
𝑛=1 𝑛 2 + 7 − 𝑛 4 is also divergent series.
Exercise 4.2.5: Determine whether the series converges or diverges.
∞ ∞
1 1
𝑎. ∑ 𝑑. ∑ 2⁄
(𝑛 + 1)(𝑛 + 2) (1 + 5𝑛)3 (4 + 𝑛) 3
𝑛=1 𝑛=1
∞ ∞
1 √𝑛
𝑏. ∑ 2 𝑒. ∑
𝑛 +𝑛+1 𝑛 + √𝑛
𝑛=1 𝑛=1
∞ ∞
1
𝑐. ∑ 𝑓. ∑ √𝑛2 + 3 − 𝑛
𝑛=1
√𝑛3 + 1 𝑛=1

Tesfaye G and Wanaw B Page 125


CALCULUS II

Ratio test
Suppose ∑∞ 𝑛=1 𝑎𝑛 is a positive term series with
𝑎𝑛+1
lim =𝑙
𝑛→∞ 𝑎𝑛
Then,
i. If 𝑙 < 1, then the series ∑∞ 𝑛=1 𝑎𝑛 is convergent.
ii. If 𝑙 > 1 or 𝑙 = ∞, then the series ∑∞ 𝑛=1 𝑎𝑛 is divergent.
iii. If 𝑙 = 1, then the test fails (no conclusion can be drawn using this test).

Example 4.2.15: Determine whether the series



𝑛
∑ 𝑛
2
𝑛=1
is convergent or divergent.
Solution: Here
𝑛 𝑛+1
𝑎𝑛 = and 𝑎𝑛+1 =
2𝑛 2𝑛+1
Then,
𝑎𝑛+1 𝑛 + 1 2𝑛 𝑛 + 1
= 𝑛+1 . =
𝑎𝑛 2 𝑛 2𝑛
Thus,
𝑎𝑛+1 𝑛+1 1
lim = lim = <1
𝑛→∞ 𝑎𝑛 𝑛→∞ 2𝑛 2
2𝑛
Therefore, by ratio test, the series ∑∞
𝑛=1 is convergent.
𝑛

Example 4.2.16: Determine whether the series



𝑛𝑛

𝑛!
𝑛=1
is convergent or divergent.
Solution: Here
𝑛𝑛 (𝑛 + 1)𝑛+1
𝑎𝑛 = and 𝑎𝑛+1 =
𝑛! (𝑛 + 1)!
Then,

Tesfaye G and Wanaw B Page 126


CALCULUS II

𝑎𝑛+1 (𝑛 + 1)𝑛+1 𝑛! (𝑛 + 1)(𝑛 + 1)𝑛 𝑛! (𝑛 + 1)𝑛


= . = . 𝑛=
𝑎𝑛 (𝑛 + 1)! 𝑛𝑛 (𝑛 + 1)𝑛! 𝑛 𝑛𝑛
Thus,
𝑎𝑛+1 (𝑛 + 1)𝑛 1 𝑛
lim = lim = lim (1 + ) =𝑒>1
𝑛→∞ 𝑎𝑛 𝑛→∞ 𝑛𝑛 𝑛→∞ 𝑛
2𝑛
Therefore, by ratio test, the series ∑∞
𝑛=1 is divergent.
𝑛

Example 4.2.17: For what value of 𝑥 the following series is convergent and divergent?

(𝑛 + 2)𝑥 2𝑛

(𝑛 + 3)(𝑛 + 4)
𝑛=1

Solution: For 𝑥 = 0 clearly the series is convergent. And for 𝑥 ≠ 0 the series is a positive term
series. And

(𝑛 + 2)𝑥 2𝑛 (𝑛 + 3)𝑥 2(𝑛+1)


𝑎𝑛 = and 𝑎𝑛+1 =
(𝑛 + 3)(𝑛 + 4) (𝑛 + 4)(𝑛 + 5)
Then,
𝑎𝑛+1 (𝑛 + 3)𝑥 2(𝑛+1) (𝑛 + 3)(𝑛 + 4) 𝑥 2 (𝑛2 + 6𝑛 + 9)
= . =
𝑎𝑛 (𝑛 + 4)(𝑛 + 5) (𝑛 + 2)𝑥 2𝑛 𝑛2 + 7𝑛 + 10
Thus,
𝑎𝑛+1 𝑥 2 (𝑛2 + 6𝑛 + 9)
lim = lim = 𝑥2
𝑛→∞ 𝑎𝑛 𝑛→∞ 𝑛2 + 7𝑛 + 10

So using ratio test,


the series is convergent, if 𝑥 2 < 1 , that is, −1 < 𝑥 < 1
the series is divergent, if 𝑥 2 > 1, that is, 𝑥 < −1 or 𝑥 > 1.
𝑎𝑛+1
But, if 𝑥 = ±1, the ratio test fails, because lim𝑛→∞ = 1. And, if 𝑥 = ±1, the series becomes
𝑎𝑛

(𝑛 + 2)

(𝑛 + 3)(𝑛 + 4)
𝑛=1

So we can use limit comparison test.


(𝑛+2)
And thus, the 𝑛th term of the above series ∑∞
𝑛=1 (𝑛+3)(𝑛+4) is

(𝑛 + 2) 1 (1 + 2⁄𝑛)
𝑎𝑛 = = ( )
(𝑛 + 3)(𝑛 + 4) 𝑛 (1 + 3⁄𝑛)(1 + 4⁄𝑛)

Tesfaye G and Wanaw B Page 127


CALCULUS II

1 (𝑛+2) 1
Take, 𝑏𝑛 = 𝑛 , and thus ∑∞ ∞ ∞ ∞
𝑛=1 𝑎𝑛 = ∑𝑛=1 (𝑛+3)(𝑛+4) and ∑𝑛=1 𝑏𝑛 = ∑𝑛=1 𝑛 are both positive term

series. Then,

1 (1 + 2⁄𝑛)
𝑛 (1 + 3⁄ )(1 + 4⁄ ))
(
𝑎𝑛 𝑛 𝑛 (1 + 2⁄𝑛)
lim = lim = lim =1
𝑛→∞ 𝑏𝑛 𝑛→∞ 1 𝑛→∞ (1 + 3⁄ )(1 + 4⁄ )
𝑛 𝑛
𝑛
(𝑛+2) 1
Thus, the series ∑∞ ∞ ∞ ∞
𝑛=1 𝑎𝑛 = ∑𝑛=1 (𝑛+3)(𝑛+4) and ∑𝑛=1 𝑏𝑛 = ∑𝑛=1 𝑛 converge and diverge
together by limit comparison test.
∞ 1 ∞ 1
But, ∑∞ ∞
𝑛=1 𝑏𝑛 = ∑𝑛=1 𝑛 is a 𝑝-series with 𝑝 = 1 ≤ 1, hence the series ∑𝑛=1 𝑏𝑛 = ∑𝑛=1 𝑛 is
divergent series.
(𝑛+2)
Therefore, the series ∑∞
𝑛=1 (𝑛+3)(𝑛+4) is also divergent series.
That is, the series

(𝑛 + 2)𝑥 2𝑛

(𝑛 + 3)(𝑛 + 4)
𝑛=1

is divergent for 𝑥 = ±1.


Therefore, the series

(𝑛 + 2)𝑥 2𝑛

(𝑛 + 3)(𝑛 + 4)
𝑛=1

is convergent, if −1 < 𝑥 < 1 and is divergent, if 𝑥 ≤ −1 or 𝑥 ≥ 1.


Exercise 4.2.6:
1. Determine whether the series converges or diverges
∞ ∞
𝑛3 1
𝑎. ∑ 𝑛+1 𝑒. ∑
5 𝑛!
𝑛=1 𝑛=1
∞ ∞
4 𝑛
𝑛2 + 3
𝑏. ∑ 𝑛 3 𝑓. ∑
3𝑛 + 1
𝑛=1 𝑛=1

7𝑛
𝑐. ∑
(𝑛 + 1)√𝑛
𝑛=1

Root Test
Suppose ∑∞
𝑛=1 𝑎𝑛 is a positive term series with
1
lim (𝑎𝑛 ) ⁄𝑛 = 𝑙
𝑛→∞
Then,

Tesfaye G and Wanaw B Page 128


CALCULUS II

i. If 𝑙 < 1, then the series ∑∞ 𝑛=1 𝑎𝑛 is convergent.


ii. If 𝑙 > 1 or 𝑙 = ∞, then the series ∑∞ 𝑛=1 𝑎𝑛 is divergent.
iii. If 𝑙 = 1, then the test fails (no conclusion can be drawn using this test).

Example 4.2.18: Determine whether the series


∞ 2
1 𝑛
∑ (1 + )
𝑛
𝑛=1
is convergent or divergent
2
1 𝑛
Solution: The 𝑛th term of series is 𝑎𝑛 = (1 + 𝑛) . Then,

1⁄ 1 𝑛
(𝑎𝑛 ) 𝑛 = (1 + )
𝑛
Thus,
1⁄ 1 𝑛
lim (𝑎𝑛 ) 𝑛 = lim (1 + ) = 𝑒 > 1
𝑛→∞ 𝑛→∞ 𝑛
2
1 𝑛
Therefore, by ratio test, the series ∑∞
𝑛=1 (1 + 𝑛) is divergent.

Example 4.2.19: Determine whether the series



3𝑛 + 1 𝑛
∑( 2 )
4𝑛 + 5
𝑛=1
is convergent or divergent
3𝑛+1 𝑛
Solution: The 𝑛th term of series is 𝑎𝑛 = (4𝑛2 +5) . Then,

1⁄ 3𝑛 + 1
(𝑎𝑛 ) 𝑛 =
4𝑛2 + 5
Thus,
1⁄ 3𝑛 + 1 3
lim (𝑎𝑛 ) 𝑛 = lim 2
= <1
𝑛→∞ 𝑛→∞ 4𝑛 + 5 4
3𝑛+1 𝑛
Therefore, by ratio test, the series ∑∞
𝑛=1 (4𝑛2 +5) is convergent.

Exercise 4.2.7:
1. Determine whether the series converges or diverges
∞ ∞ 2
𝑛 2𝑛 𝑛𝑛
𝑎. ∑ 𝑛 𝑒. ∑
2 (1 + 𝑛)𝑛2
𝑛=1 𝑛=1
∞ 2 ∞
𝑛−1 𝑛 1 + 2𝑛 𝑛
𝑏. ∑ ( ) 𝑓. ∑( )
𝑛 𝑛
𝑛=1 𝑛=1

Tesfaye G and Wanaw B Page 129


CALCULUS II

2. Determine for what value of 𝑥 the following series converges and diverges.
∞ 𝑛
𝑛𝑥 2
∑( )
1+𝑛
𝑛=1

4.2.5 Alternating series


At the end of this section students will be able to:
 Define alternating series.
 Determine the convergence of an alternating series using Leibnitz’s Test.

The series (infinite series) of the form


𝑎1 − 𝑎2 + 𝑎3 − 𝑎4 + 𝑎5 − 𝑎6 + ⋯
that is,

∑(−1)𝑛−1 𝑎𝑛
𝑛=1

where 𝑎𝑛 > 0, for all 𝑛 is called an alternating series.


Example 4.2.20: The following are alternating series
𝑎. 2 − 1 + 6 − 8 + 12 − 4 + 7 − 15 + ⋯
∞ ∞
𝑛+1
(−1)𝑛
𝑏. ∑(−1) 𝑛 𝑐. ∑
2𝑛
𝑛=1 𝑛=1

Leibnitz’s Test : The alternating series


∑(−1)𝑛−1 𝑎𝑛 = 𝑎1 − 𝑎2 + 𝑎3 − 𝑎4 + 𝑎5 − 𝑎6 + ⋯
𝑛=1

is convergent if
𝑖. 𝑎𝑛 ≥ 𝑎𝑛+1 , for all 𝑛
𝑖𝑖. lim 𝑎𝑛 = 0
𝑛→∞

Example 4.2.21: Test for the convergence of the series



(−1)𝑛−1

𝑛
𝑛=1

Solution: The given series is an alternating series

Tesfaye G and Wanaw B Page 130


CALCULUS II


(−1)𝑛−1 1 1 1 1 1
∑ =1− + − + − +⋯
𝑛 2 3 4 5 6
𝑛=1

And,
1 1
𝑎𝑛 = , 𝑎𝑛+1 =
𝑛 𝑛+1
1 1
Clearly, 𝑛 ≥ 𝑛+1 and this implies 𝑎𝑛 ≥ 𝑎𝑛+1 , for all 𝑛.

Moreover,
1
lim 𝑎𝑛 = lim =0
𝑛→∞ 𝑛→∞ 𝑛
(−1)𝑛−1
Hence, by Leibnitz’s test, the series ∑∞
𝑛=1 is convergent.
𝑛

Example 4.2.22: Test for the convergence of the series



(−1)𝑛+1

𝑛=1
𝑛 √𝑛

Solution: The given series is an alternating series


And,
1 1
𝑎𝑛 = , 𝑎𝑛+1 =
𝑛 √𝑛 (𝑛 + 1)√𝑛 + 1
1 1 (𝑛+1)√𝑛+1−𝑛√𝑛
Clearly, 𝑛 − (𝑛+1) = ≥ 0 because (𝑛 + 1)√𝑛 + 1 > 𝑛√𝑛 , for all 𝑛 and
√ 𝑛 √𝑛+1 𝑛√𝑛(𝑛+1)√𝑛+1
1 1
this implies ≥ (𝑛+1) , for all 𝑛, that is 𝑎𝑛 ≥ 𝑎𝑛+1 , for all 𝑛
𝑛 √𝑛 √𝑛+1

Moreover,
1
lim 𝑎𝑛 = lim =0
𝑛→∞ 𝑛→∞ 𝑛√𝑛

(−1)𝑛+1
Hence, by Leibnitz’s test, the series ∑∞
𝑛=1 is convergent.
𝑛 √𝑛

Example4. 2.23: Test for the convergence of the series



𝑛
∑(−1)𝑛−1
𝑛2 +4
𝑛=1

Solution: The given series is an alternating series.


And,
𝑛
𝑎𝑛 =
𝑛2 +4

Tesfaye G and Wanaw B Page 131


CALCULUS II

𝑥 𝑛
Consider the related real variable function (𝑥) = 𝑥 2 +4 , where 𝑓(𝑛) = 𝑛2 +4 = 𝑎𝑛 , for all

𝑛 = 0,1,2,3, ….
Then,

′ (𝑥)
4 − 𝑥2
𝑓 = 2
(𝑥 + 4)2
Since we are considering only positive 𝑥, we see that 𝑓 ′ (𝑥) < 0, if 4 − 𝑥 2 < 0, that is if 𝑥 > 2.
Thus, 𝑓 is decreasing on an interval (2, ∞). This means that 𝑓(𝑛 + 1) ≤ 𝑓(𝑛) and therefore
𝑎𝑛 ≥ 𝑎𝑛+1 , for all 𝑛 ≥ 3. Hence, (i) of Leibnitz’s test is satisfied for 𝑛 ≥ 3
Moreover,
𝑛
lim 𝑎𝑛 = lim =0
𝑛→∞ 𝑛→∞ 𝑛2 +4
𝑛−1 𝑛
Hence, by Leibnitz’s test, the series ∑∞
𝑛=1(−1) is convergent.
𝑛2 +4

Therefore,
∞ ∞
1 2 𝑛 𝑛
− + ∑(−1)𝑛−1 2 = ∑(−1)𝑛−1 2
5 8 𝑛 +4 𝑛 +4
𝑛=3 𝑛=1

is convergent.
Exercise 4.2.8: Determine whether the series converges or diverges.
∞ ∞
𝑛−1
𝑛
𝑎. ∑(−1) 𝑐. ∑(−1)𝑛−1 (√𝑛 + 1 − √𝑛)
5𝑛 + 4
𝑛=1 𝑛=1
∞ 𝜋 ∞
𝑠𝑖𝑛 ((2𝑛 − 1) 2 ) 𝑛2
𝑛−1
𝑏. ∑ 𝑑. ∑(−1)
𝑛2 6𝑛3 − 5
𝑛=1 𝑛=1

4.2.6 Absolute convergence


At the end of this section students will be able to:
 Define absolute convergence of a series.
 Determine the absolute convergence of a series.

Tesfaye G and Wanaw B Page 132


CALCULUS II

Given any series ∑∞


𝑛=1 𝑎𝑛 . We can have a series whose terms are absolute values of terms of the

series ∑∞
𝑛=1 𝑎𝑛 , that is given by

∑|𝑎𝑛 | = |𝑎1 | + |𝑎2 | + |𝑎3 | + ⋯


𝑛=1

Definition 4.2.3: The series ∑∞


𝑛=1 𝑎𝑛 is called absolutly convergent, if the series whose terms are

absolute values of terms of the series ∑∞


𝑛=1 𝑎𝑛 is convergent.

That is, The series ∑∞ ∞


𝑛=1 𝑎𝑛 is called absolutly convergent, if the series ∑𝑛=1|𝑎𝑛 | is convergent.

Example 4.2.24: Consider the series



(−1)𝑛

𝑛3
𝑛=1
(−1)𝑛
The series whose terms are absolute values of terms of the series ∑∞
𝑛=1 is
𝑛3
∞ ∞
(−1)𝑛 1 1 1 1
∑| 3 | = 1+ + + +⋯= ∑ 3
𝑛 8 27 64 𝑛
𝑛=1 𝑛=1

which is a 𝑝-series with 𝑝 = 3 > 1, hence it is convergent.


(−1)𝑛
Hence, the series ∑∞
𝑛=1 is absolutly convergent.
𝑛3

Exerise 4.2.9: Show that the series



(−1)𝑛+1

𝑛2 + 2𝑛 + 3
𝑛=1

is absolutly convergent.
Theorem 4.2.1: An absolutly convergent series is convergent.
Definition 4.2.4: A series ∑∞
𝑛=1 𝑎𝑛 which is convergent but not absolutly convergent (absolutly

divergent) is called conditionaly convergent.

Example 4.2.25: Consider the series



(−1)𝑛−1

𝑛
𝑛=1
1 1
Here, the series is an alternating series with 𝑎𝑛 = 𝑛 and 𝑎𝑛+1 = 𝑛+1.

Tesfaye G and Wanaw B Page 133


CALCULUS II

Here,
1 1
≥ , for all 𝑛
𝑛 𝑛+1
That is
𝑎𝑛 ≥ 𝑎𝑛+1 , for all 𝑛
And,
1
lim 𝑎𝑛 = lim =0
𝑛→∞ 𝑛→∞ 𝑛
(−1)𝑛−1
Hence, by Liebnitz’s test the series ∑∞
𝑛=1 is convergent.
𝑛
(−1)𝑛−1
The series whose terms are the absolute value of terms of the series ∑∞
𝑛=1 is a series
𝑛

1

𝑛
𝑛=1
1
which is a 𝑝-series with 𝑝 = 1 ≤ 1, and hence is not convergent, that is, ∑∞
𝑛=1 𝑛 is a divergent

(−1)𝑛−1
series. That is, the series ∑∞
𝑛=1 is not absolutly convergent series.
𝑛
(−1)𝑛−1
Thus, the series ∑∞
𝑛=1 is a convergent but not absolutly convergent.
𝑛
(−1)𝑛−1
Therfore, the series ∑∞
𝑛=1 is conditionaly convergent series.
𝑛

Exercise 4.2.10:
1. Prove Theorem 4.2.1.
2. Show that the series


(−1)𝑛

𝑛=1
√𝑛
is conditionaly converges.

4.2.7 Generalized convergence tests


At the end of this section students will be able to:
 State generalized convergence tests.

 Generalized comparison test

Tesfaye G and Wanaw B Page 134


CALCULUS II

Let ∑∞ ∞
𝑛=1 𝑎𝑛 be an series and ∑𝑛=1 𝑏𝑛 be a piositive term series such that

|𝑎𝑛 | ≤ 𝑏𝑛 , for all 𝑛


Then, if ∑∞ ∞
𝑛=1 𝑏𝑛 is convergent, the series ∑𝑛=1 𝑎𝑛 is absolutly convergent hence it is convergent.

 Generalized limit comparison test

Let ∑∞ ∞
𝑛=1 𝑎𝑛 be an series and ∑𝑛=1 𝑏𝑛 be a piositive term series such that

|𝑎𝑛 |
lim =𝑙
𝑛→∞ 𝑏𝑛

Then, if 𝑙 > 0 and ∑∞ ∞


𝑛=1 𝑏𝑛 is convergent, the series ∑𝑛=1 𝑎𝑛 is absolutly convergent hence it is

convergent.
 Generalized Ratio test

Let ∑∞
𝑛=1 𝑎𝑛 be an series such that
𝑎𝑛+1
lim | |=𝑙 , where 𝑎𝑛 ≠ 0
𝑛→∞ 𝑎𝑛

Then, if 𝑙 < 1, then the series ∑∞


𝑛=1 𝑎𝑛 is absolutly convergent hence it is convergent.

 Generalized Root test

Let ∑∞
𝑛=1 𝑎𝑛 be an series such that
1⁄
lim (|𝑎𝑛 |) 𝑛 =𝑙 , where 𝑎𝑛 ≠ 0
𝑛→∞

Then, if 𝑙 < 1, then the series ∑∞


𝑛=1 𝑎𝑛 is absolutly convergent hence it is convergent

Exercise: Show that the series


𝑥𝑛

𝑛=1
√𝑛
converges for |𝑥| < 1, and conditionaly converges for 𝑥 = −1.

Chapter Summary
 If {𝑎𝑛 } is sequence of real numbers, then the expression that is the sum of the sequence
which is an infinite in number

𝑎1 + 𝑎2 + ⋯ + 𝑎𝑛 + ⋯
Tesfaye G and Wanaw B Page 135
CALCULUS II

is called an infinite series or simply series and is denoted by the symbol ∑∞


𝑛=1 𝑎𝑛 or ∑ 𝑎𝑛 .

 For the series ∑∞


𝑛=1 𝑎𝑛 , 𝑆𝑛 (the sum of the first 𝑛 terms of the sequence {𝑎𝑛 }) which is

𝑆𝑛 = ∑ 𝑎𝑛 = 𝑎1 + 𝑎2 + ⋯ + 𝑎𝑛
𝑖=1

is called the 𝑛th partial sum of the series ∑∞


𝑛=1 𝑎𝑛 .

 Let 𝑆𝑛 be the 𝑛th partial sum of the series ∑∞


𝑛=1 𝑎𝑛 and let lim𝑛→∞ 𝑆𝑛 = 𝑙. Then,

iv. If 𝑙 is finite real number, then the series ∑∞


𝑛=1 𝑎𝑛 is convergent.

v. If 𝑙 = ±∞, then the series ∑∞


𝑛=1 𝑎𝑛 is divergent.

vi. If 𝑙 is not unique, then the series ∑∞


𝑛=1 𝑎𝑛 is non-convergent (oscillatory).

 A series ∑∞
𝑛=1 𝑎𝑛 in which all terms (𝑎𝑖 ′𝑠) are positive is called positive term series.

 Suppose we have a positive term series ∑∞


𝑛=1 𝑎𝑛 then

Test 1: If lim𝑛→∞ 𝑎𝑛 ≠ 0 or does not exist, then the series ∑∞


𝑛=1 𝑎𝑛 diverges.

Test 2: The series ∑∞


𝑛=1 𝑎𝑛 is convergent if and only if there exists a number 𝐾 such that

𝑆𝑛 = 𝑎1 + 𝑎2 + ⋯ + 𝑎𝑛 < 𝐾 , for all 𝑛.


Test 3: The series ∑∞
𝑛=1 𝑎𝑛 diverges, if each term after a fixed stage is greater than some

fixed positive number.


 Integral Test: Suppose {𝑎𝑛 } be a sequence and 𝑓 is a continuous positive, decreasing
function on [1, ∞) and let 𝑓(𝑛) = 𝑎𝑛 . Then,
∞ 𝑥
iii. If ∫1 𝑓(𝑥)𝑑𝑥 = lim𝑥→∞ ∫1 𝑓(𝑥)𝑑𝑥 converges, then the series ∑∞
𝑛=1 𝑎𝑛 is convergent.
∞ 𝑥
iv. If ∫1 𝑓(𝑥)𝑑𝑥 = lim𝑥→∞ ∫1 𝑓(𝑥)𝑑𝑥 diverges, then the series ∑∞
𝑛=1 𝑎𝑛 is divergent.

 Comparison Test: Suppose that ∑∞ ∞


𝑛=1 𝑎𝑛 and ∑𝑛=1 𝑏𝑛 are two positive term series such that

𝑎𝑛 ≤ 𝑏𝑛 , for all 𝑛
Then,
iii. If ∑∞ ∞
𝑛=1 𝑏𝑛 is convergent, then ∑𝑛=1 𝑎𝑛 is alse convergent.

iv. If ∑∞ ∞
𝑛=1 𝑎𝑛 is divergent, then ∑𝑛=1 𝑏𝑛 is alse divergent.

 Limit Comparison Test: Suppose that ∑∞ ∞


𝑛=1 𝑎𝑛 and ∑𝑛=1 𝑏𝑛 are two positive term series. If
𝑎𝑛
lim =𝑐
𝑛→∞ 𝑏𝑛

where 𝑐 is a positive real number, then ∑∞ ∞


𝑛=1 𝑎𝑛 and ∑𝑛=1 𝑏𝑛 converges or diverges together.

Tesfaye G and Wanaw B Page 136


CALCULUS II

 Ratio Test: Suppose ∑∞


𝑛=1 𝑎𝑛 is a positive term series with

𝑎𝑛+1
lim =𝑙
𝑛→∞ 𝑎𝑛

Then,
iv. If 𝑙 < 1, then the series ∑∞
𝑛=1 𝑎𝑛 is convergent.

v. If 𝑙 > 1 or 𝑙 = ∞, then the series ∑∞


𝑛=1 𝑎𝑛 is divergent.

vi. If 𝑙 = 1, then the test fails (no conclusion can be drawn using this test).
 Root Test: Suppose ∑∞
𝑛=1 𝑎𝑛 is a positive term series with

1⁄
lim (𝑎𝑛 ) 𝑛 =𝑙
𝑛→∞

Then,
iv. If 𝑙 < 1, then the series ∑∞
𝑛=1 𝑎𝑛 is convergent.

v. If 𝑙 > 1 or 𝑙 = ∞, then the series ∑∞


𝑛=1 𝑎𝑛 is divergent.

vi. If 𝑙 = 1, then the test fails (no conclusion can be drawn using this test).
 The series (infinite series) of the form
𝑎1 − 𝑎2 + 𝑎3 − 𝑎4 + 𝑎5 − 𝑎6 + ⋯
that is,

∑(−1)𝑛−1 𝑎𝑛
𝑛=1

where 𝑎𝑛 > 0, for all 𝑛 is called an alternating series.


 Leibnitz’s Test: The alternating series

∑(−1)𝑛−1 𝑎𝑛 = 𝑎1 − 𝑎2 + 𝑎3 − 𝑎4 + 𝑎5 − 𝑎6 + ⋯
𝑛=1

is convergent if
𝑖. 𝑎𝑛 ≥ 𝑎𝑛+1 , for all 𝑛
𝑖𝑖. lim 𝑎𝑛 = 0
𝑛→∞

 The series ∑∞
𝑛=1 𝑎𝑛 is called absolutly convergent, if the series whose terms are absolute

values of terms of the series ∑∞ ∞


𝑛=1 𝑎𝑛 is convergent. That is, The series ∑𝑛=1 𝑎𝑛 is called

absolutly convergent, if the series ∑∞


𝑛=1|𝑎𝑛 | is convergent.

Tesfaye G and Wanaw B Page 137


CALCULUS II

 A series ∑∞
𝑛=1 𝑎𝑛 which is convergent but not absolutly convergent is called conditionaly

convergent.

Review Exercise
1. Using 𝑛th partial sum determine whether the following series is convergent or divergent.

2 4
∑ ( 𝑛 + 𝑛)
5 3
𝑛=1

Tesfaye G and Wanaw B Page 138


CALCULUS II

2. Determine whether the following series is convergent or divergent.

∞ ∞
𝑛
𝑎. ∑ 𝑛 𝑓. ∑ √𝑛 + 1 − √𝑛 − 1
2
𝑛=1 𝑛=1
∞ ∞
𝑙𝑛𝑛 4𝑛 𝑛𝑛
𝑏. ∑ 𝑔. ∑
𝑛𝑝 𝑛
𝑛=1 𝑛=1
∞ ∞
3 (𝑛2 − 1)2𝑛
𝑐. ∑ 𝑛 ℎ. ∑
3 +1 𝑛2 + 1
𝑛=1 𝑛=1
∞ ∞ 2
log 5 𝑛 𝑛𝑛
𝑑. ∑ 𝑖. ∑
𝑛+2 (1 + 𝑛)𝑛2
𝑛=1 𝑛=1

𝑛
𝑒. ∑
𝑛=1
1 + 𝑛 √𝑛 + 1
3. Determine for what value of 𝑥 the following series converges and diverges.

(𝑥 − 1)4𝑛
𝑎. ∑
𝑛2
𝑛=1

𝑥 2𝑛
𝑏. ∑
𝑛
𝑛=1

4. Determine for what value of 𝑥 the following series converges and diverges.
∞ 𝑛
𝑛𝑥 2
∑( )
1+𝑛
𝑛=1

5. Determine whether the series converges or diverges.



𝑐𝑜𝑠((𝑛 − 1)𝜋)
𝑎. ∑
𝑛2
𝑛=1

𝑛
𝑏. ∑(−1)𝑛−1
6𝑛3 − 5
𝑛=1

6. Show that the series

Tesfaye G and Wanaw B Page 139


CALCULUS II


(−2)𝑛 𝑛
∑ 𝑥
2𝑛 + 1
𝑛=1
1
converges for |𝑥| ≤ 2, and conditionaly converges for 𝑥 = −1.

Tesfaye G and Wanaw B Page 140


CALCULUS II

4.3 Power series


Chapter objective: At the end of this chapter students will be able to:
 Define power series.
 Determine a convergence of a power series.
 Differentaite and integrate a power series in its interval of convergence.
 Find power series expansion of certain functions by manipulating geometric series
or by differentiating or integrating such a series.
 Explain algebraic operations on converging power series.
 Find Taylor series expansion and Maclaurine series expansion of a function.

4.3.1 Definition of power series


At the end of this section students will be able to:
 Define power series.
 Determine a convergence of a power series.

Definition 4.3.1: A series of the form


∑ 𝑐𝑛 𝑥 𝑛 = 𝑐0 + 𝑐1 𝑥 + 𝑐2 𝑥 2 + 𝑐3 𝑥 3 + 𝑐4 𝑥 4 + ⋯
𝑛=0

where 𝑥 is a variable and 𝑐𝑛 ’s are constants is called a power series (a power series in 𝑥).
Generally, a series of the form

∑ 𝑐𝑛 (𝑥 − 𝑎)𝑛 = 𝑐0 + 𝑐1 (𝑥 − 𝑎) + 𝑐2 (𝑥 − 𝑎)2 + 𝑐3 (𝑥 − 𝑎)3 + 𝑐4 (𝑥 − 𝑎)4 + ⋯


𝑛=0

where 𝑥 is a variable and 𝑐𝑛 ’s are constants is called a power series in 𝑥 − 𝑎 or a power series
about 𝑎 or a power serie centered at 𝑎.
𝑛
For each 𝑥, the sereis ∑∞
𝑛=0 𝑐𝑛 (𝑥 − 𝑎) is a sseries of constants that we can test for
convergence or divergence. A power series may converge for some values of 𝑥 and diverge for
othe values of 𝑥.
For instance, if we take 𝑐𝑛 = 1, for all 𝑛 ≥ 0 and 𝑎 = 0, we have a series

∑ 𝑥𝑛 = 1 + 𝑥 + 𝑥2 + 𝑥3 + 𝑥4 + ⋯
𝑛=0

Tesfaye G and Wanaw B Page 141


CALCULUS II

which is a geometric series (power series centered at 0) and is convergent when −1 < 𝑥 < 1 and
is divergent when 𝑥 ≤ −1 or 𝑥 ≥ 1. Here, this series has radius of convergence 𝑅 = 1 and
converges in the interval (−1,1) which we call it interval of convergence.
Define

𝑓(𝑥) = ∑ 𝑐𝑛 (𝑥 − 𝑎)𝑛
𝑛=0

which is power series centered at 𝑎.


Problem: What is the domain of 𝑓?
The domain of 𝑓 is the collection of 𝑥 for which the series converges. We note that a is in the
domain of 𝑓.
Therefore, the domain of a power series

∑ 𝑐𝑛 (𝑥 − 𝑎)𝑛
𝑛=0

is the collection of all values of 𝑥 for which the series converges.


Theorem 4.3.1: For a given series

∑ 𝑐𝑛 (𝑥 − 𝑎)𝑛
𝑛=0

there are only three possibilities.


i. The series converges only when 𝑥 = 𝑎.
ii. The series converges for all 𝑥.
iii. There is a positive number 𝑅 such that the series converges if |𝑥 − 𝑎| < 𝑅 and
diverges |𝑥 − 𝑎| > 𝑅.

The number 𝑅 in case (iii) is called radius of convergence of a power series.


By convention, if 𝑅 = 0, the series converges only when 𝑥 = 𝑎 (case i) and if 𝑅 = ∞, the series
converges for all 𝑥 (case ii).
Definition 4.3.2: The interval of convergence of a power series is the interval that consists of all
values of 𝑥 for which the series converges.
Thus,
a. If 𝑅 = 0, the series converges only when 𝑥 = 𝑎 and the interval of convergence is the
interval consists of just a single point 𝑎.

Tesfaye G and Wanaw B Page 142


CALCULUS II

b. If 𝑅 = ∞, the series converges form all 𝑥 and the interval of convergence is the interval is
(−∞, ∞).
c. If 𝑅 ≠ 0 and is finite real number, there are four possibilities
I. The series converges for 𝑥 such that |𝑥 − 𝑎| < 𝑅 and the interval of convergence
is (𝑎 − 𝑅, 𝑎 + 𝑅).
II. The series converges for 𝑥 such that |𝑥 − 𝑎| ≤ 𝑅 and the interval of convergence
is [𝑎 − 𝑅, 𝑎 + 𝑅].
III. The series converges for 𝑥 such that 𝑎 − 𝑅 ≤ 𝑥 < 𝑎 + 𝑅 and the interval of
convergence is [𝑎 − 𝑅, 𝑎 + 𝑅).
IV. The series converges for 𝑥 such that 𝑎 − 𝑅 < 𝑥 ≤ 𝑎 + 𝑅 and the interval of
convergence is (𝑎 − 𝑅, 𝑎 + 𝑅].

In most cases Ratio test can be used ton determine the radius of convergence (hence we can
𝑛
determine the interval of convergence) of the power series ∑∞
𝑛=0 𝑐𝑛 (𝑥 − 𝑎) . The Ratio test

always fails when 𝑥 is in an end point of the interval of convergence, so the end points of the
interval of convergence must be checked with some other test.
Given a power series

∑ 𝑐𝑛 (𝑥 − 𝑎)𝑛
𝑛=0

Then, 𝑎𝑛 = 𝑐𝑛 (𝑥 − 𝑎) and 𝑎𝑛+1 = 𝑐𝑛+1 (𝑥 − 𝑎)𝑛+1 .


𝑛

Then, by Generalized Ratio test the series converges if


𝑎𝑛+1
lim | |<1
𝑛→∞ 𝑎𝑛

And diverges for


𝑎𝑛+1
lim | |>1
𝑛→∞ 𝑎𝑛
Thus,
𝑎𝑛+1 𝑐𝑛+1 (𝑥 − 𝑎)𝑛+1 𝑐𝑛+1 (𝑥 − 𝑎)
lim | | = lim | | = lim | |
𝑛→∞ 𝑎𝑛 𝑛→∞ 𝑐𝑛 (𝑥 − 𝑎)𝑛 𝑛→∞ 𝑐𝑛
𝑐𝑛+1
= |𝑥 − 𝑎| lim | |
𝑛→∞ 𝑐𝑛

Implying that

Tesfaye G and Wanaw B Page 143


CALCULUS II

𝑎𝑛+1 1 𝑐𝑛
lim | |<1 if |𝑥 − 𝑎| < 𝑐𝑛+1 = 𝑛→∞
lim | |
𝑛→∞ 𝑎𝑛 𝑐𝑛+1
lim | 𝑐 |
𝑛→∞ 𝑛

Thus, the power series converges if


𝑐𝑛
|𝑥 − 𝑎| < lim | |
𝑛→∞ 𝑐𝑛+1

And diverges
𝑐𝑛
|𝑥 − 𝑎| > lim | |
𝑛→∞ 𝑐𝑛+1

So, how can we determine the radius of convergence of a power series using Generalized Ratio
Test?
Therefore, suppose

∑ 𝑐𝑛 (𝑥 − 𝑎)𝑛
𝑛=0

is a given power series, then the value


𝑐𝑛
lim | |=𝑅
𝑛→∞ 𝑐𝑛+1

is the radius of convergence.


Example 4.3.1: Find the radius of convergence and interval of convergence of the geometric
series

∑ 𝑥𝑛
𝑛=0

∑∞ 𝑛
Solution: 𝑛=0 𝑥 is a power serties about 0 where 𝑐𝑛 = 1, for all 𝑛 ≥ 0. Thus,
𝑐𝑛 = 1 and 𝑐𝑛+1 = 1
Then,
𝑐𝑛 1
lim | | = lim | | = 1
𝑛→∞ 𝑐𝑛+1 𝑛→∞ 1
𝑛
Therefore, the radius of convergence of the power series ∑∞
𝑛=0 𝑥 is 𝑅 = 1.

Hence, the series converges if |𝑥| < 1 and diverges |𝑥| > 1.
But, we need to check at the end points of the interval (−1,1), thast is at 𝑥 = ±1.
If 𝑥 = 1, the series becomes

Tesfaye G and Wanaw B Page 144


CALCULUS II

∑ 1𝑛
𝑛=0

which is a divergent series.


If 𝑥 = −1, the series becomes

∑(−1)𝑛
𝑛=0

which is a divergent series.


𝑛
Therefore, the radius of convergence of the power series ∑∞
𝑛=0 𝑥 is 𝑅 = 1 and its interval of

convergence is (−1,1).
Example 4.3.2: Find the radius of convergence and interval of convergence of the series

∑ 𝑛! 𝑥 𝑛
𝑛=0
𝑛
Solution: ∑∞
𝑛=0 𝑛! 𝑥 is a power serties about 0 and,

𝑐𝑛 = 𝑛! and 𝑐𝑛+1 = (𝑛 + 1)!


Then,
𝑐𝑛 𝑛! 1
lim | | = lim | | = lim =0
𝑛→∞ 𝑐𝑛+1 𝑛→∞ (𝑛 + 1)! 𝑛→∞ 𝑛 + 1
𝑛
Therefore, the radius of convergence of the power series ∑∞
𝑛=0 𝑛! 𝑥 is 𝑅 = 0.
𝑛
Therefore, the radius of convergence of the power series ∑∞
𝑛=0 𝑛! 𝑥 is 𝑅 = 0 and its interval of

convergence is {0}.
Example 4.3.3: Find the radius of convergence and interval of convergence of the series

(𝑥 − 5)𝑛

𝑛+1
𝑛=0
(𝑥−5)𝑛
Solution: ∑∞
𝑛=0 is a power serties about 5 and,
𝑛+1

1 1
𝑐𝑛 = and 𝑐𝑛+1 =
𝑛+1 𝑛+2
Then,
𝑐𝑛 𝑛+2 𝑛+2
lim | | = lim | | = lim =1
𝑛→∞ 𝑐𝑛+1 𝑛→∞ 𝑛 + 1 𝑛→∞ 𝑛 + 1

(𝑥−5)𝑛
Therefore, the radius of convergence of the power series ∑∞
𝑛=0 is 𝑅 = 1.
𝑛+1

Tesfaye G and Wanaw B Page 145


CALCULUS II

Hence, the series converges if |𝑥 − 5| < 1 and diverges |𝑥 − 5| > 1.


But, we need to check at the convergence of the series at the end points of the interval (4,6),
thast is at 𝑥 = 4 and at 𝑥 = 6.
If 𝑥 = 4, the serioes becomes
∞ ∞
(−1)𝑛 (−1)𝑛+1
∑ = ∑
𝑛+1 𝑛
𝑛=0 𝑛=1

which is an alternating series and it is convergent by Leibnitz’s Test.


If 𝑥 = 6, the series becomes

1

𝑛
𝑛=0

which is 𝑝-series with 𝑝 = 1 ≤ 1 and hence it is divergent series.


(𝑥−5)𝑛
Therefore, the radius of convergence of the power series ∑∞
𝑛=0 is 𝑅 = 1 and its interval of
𝑛+1

convergence is [4,6).
Example 4.3.4: Find the radius of convergence and interval of convergence of the series

(𝑥 + 2)𝑛
𝑛
∑(−1)
𝑛2𝑛
𝑛=1
(𝑥−5)𝑛 (𝑥−5)𝑛
Solution: ∑∞
𝑛=1 is a power serties about −2 and and it has a form ∑∞
𝑛=0 , where
𝑛 𝑛
(−1)𝑛
𝑐0 = 0 and 𝑐𝑛 = , for all 𝑛 ≥ 1. Then,
𝑛2𝑛

(−1)𝑛 (−1)𝑛+1
𝑐𝑛 = and 𝑐𝑛+1 =
𝑛2𝑛 (𝑛 + 1)2𝑛+1
Then,
𝑐𝑛 (−1)𝑛 (𝑛 + 1)2𝑛+1 −(2𝑛 + 2)
lim | | = lim | . | = lim | |=2
𝑛→∞ 𝑐𝑛+1 𝑛→∞ 𝑛2𝑛 (−1)𝑛+1 𝑛→∞ 𝑛
𝑛 (𝑥+2)𝑛
Therefore, the radius of convergence of the power series ∑∞
𝑛=1(−1) is 𝑅 = 2.
𝑛2𝑛

Hence, the series converges if |𝑥 + 2| < 2 and diverges |𝑥 + 2| > 2.


But, we need to check at the convergence of the series at the end points of the interval (−2 −
2, −2 + 2) = (−4,0), thast is at 𝑥 = −4 and at 𝑥 = 0.
If 𝑥 = −4, the serioes becomes

Tesfaye G and Wanaw B Page 146


CALCULUS II

∞ ∞ ∞
(−2)𝑛 𝑛 2𝑛
2𝑛 1
∑(−1) 𝑛
= ∑(−1) 𝑛
=∑
𝑛2 𝑛2 𝑛
𝑛=1 𝑛=1 𝑛=1

which is a 𝑝-series with 𝑝 = 1 ≤ 1, hence it is divergent series.


If 𝑥 = 0, the series becomes
∞ ∞ ∞
(2)𝑛 (−1)𝑛
𝑛
(−1)𝑛
∑(−1) = ∑ = −1 + ∑
𝑛2𝑛 𝑛 𝑛
𝑛=1 𝑛=1 𝑛=2

which is an alternating series and hence it is converges by Leibnitz’s test.


𝑛 (𝑥+2)𝑛
Therefore, the radius of convergence of the power series ∑∞
𝑛=1(−1) is 𝑅 = 2 and its
𝑛2𝑛

interval of convergence is (−4,0].


Example 4.3.5: Find the radius of convergence and interval of convergence of the series

𝑥𝑛

𝑛!
𝑛=0
𝑥𝑛
Solution: observe that the series ∑∞
𝑛=0 𝑛! is a power serties about 0 and,

1 1
𝑐𝑛 = and 𝑐𝑛+1 =
𝑛! (𝑛 + 1)!
Then,
𝑐𝑛 (𝑛 + 1)!
lim | | = lim | | = lim (𝑛 + 1) = ∞
𝑛→∞ 𝑐𝑛+1 𝑛→∞ 𝑛! 𝑛→∞

𝑥𝑛
Therefore, the radius of convergence of the power series ∑∞
𝑛=0 𝑛! is 𝑅 = ∞.

𝑥𝑛
Therefore, the radius of convergence of the power series ∑∞
𝑛=0 𝑛! is 𝑅 = ∞ and its interval of

convergence is (−∞, ∞).


Exercise 4.3.1: Find the radius of convergence and interval of convergence of the following
power series.
∞ ∞
𝑥𝑛
𝑎. ∑ ℎ. ∑ 𝑛! (2𝑥 − 1)𝑛
𝑛=1
√𝑛 𝑛=0
∞ ∞
𝑥𝑛 𝑛
𝑏. ∑(−1) 𝑖. ∑ 3𝑛 (9𝑥 − 3)𝑛
𝑛+1
𝑛=0 𝑛=0
∞ ∞
𝑥𝑛 2𝑛 (8𝑥 + 24)𝑛
𝑐. ∑ 𝑛 𝑗. ∑
𝑛3 𝑛!
𝑛=1 𝑛=0

Tesfaye G and Wanaw B Page 147


CALCULUS II

4.3.2 Power series Representation of functions


At the end of this section students will be able to:
 Find power series expansion of certain functions by manipulating geometric series.
In this section we will see how to represent certain types of functions as a sum of power series by
manipulating a geometric series or by differentiating or integrating such a series.
We start with an equation that we have seen before. The power series representation of a
1
function 1−𝑥 , for |𝑥| < 1 is

1
= 1 + 𝑥 + 𝑥2 + 𝑥3 + ⋯ = ∑ 𝑥𝑛
1−𝑥
𝑛=0

That is,

1
= ∑ 𝑥𝑛
1−𝑥
𝑛=0

Example 4.3.6: Find the power series representation of the function


𝑥2
1+𝑥
Solution: We can rewrite it as
∞ ∞ ∞
𝑥2 1
= 𝑥2 ( ) = 𝑥 2 (∑(−𝑥)𝑛 ) = ∑(−1)𝑛 𝑥 𝑛+2 = ∑(−1)𝑛−2 𝑥 𝑛
1+𝑥 1 − (−𝑥)
𝑛=0 𝑛=0 𝑛=2

𝑛−2 𝑛 𝑥2
Hence, ∑∞
𝑛=2(−1) 𝑥 is the power series representation of the function 1+𝑥 . And observe

that, the series converges when |−𝑥| < 1, that is when |𝑥| < 1 and diverges when |−𝑥| > 1,
that is when |𝑥| > 1 and its interval of convergence is (−1,1).
Example 4.3.7: Find the power series representation of the function
1
𝑥−2
And determine the interval of convergence of the power series.
Solution: We can rewrite it as

1 −1 1
= ( )
𝑥−2 2 1−𝑥
2
But,

Tesfaye G and Wanaw B Page 148


CALCULUS II


1 𝑥 𝑛
𝑥 = ∑ ( )
1 − 2 𝑛=0 2
∞ ∞ ∞
1 −1 1 −1 𝑥 𝑛 −𝑥 𝑛 2𝑛+1
𝑥𝑛
= ( ) = (∑ ( ) ) = ∑ = ∑(−1)
𝑥−2 2 1−𝑥 2 2 2𝑛+1 2𝑛+1
2 𝑛=0 𝑛=0 𝑛=0

2𝑛+1 𝑥𝑛 1
Hence, ∑∞
𝑛=0(−1) is the power series representation of the function . And this
2𝑛+1 𝑥−2
𝑥 𝑥
series converges when |2| < 1, that is when |𝑥| < 2 and diverges when |2| > 1, that is when

|𝑥| > 2 and the series is divergent at the two end points of the interval (−2,2), hence its interval
of convergence is (−2,2).
Exercise: Find the power series representation of each of the following functions and determine
the interval of convergence of the power series.
𝑥 1
a. d.
1−𝑥 1+9𝑥 2
1 𝑥2
b. e.
4+𝑥 2 𝑥+5
𝑥
c. 4𝑥+1

4.3.3 Algebraic operations on converging power series


At the end of this section students will be able to:
 Explain algebraic operations on converging power series.

Two converging power series centered at the same point can be added, multiplied and divided
very much like polynomials.
Suppose
∞ ∞

∑ 𝑝𝑛 (𝑥 − 𝑎)𝑛 and ∑ 𝑞𝑛 (𝑥 − 𝑎)𝑛


𝑛=0 𝑛=0

be two power series having interval of convergence 𝐼1 and 𝐼2 respectively. Then,


∞ ∞ ∞

𝑖. ∑ 𝑝𝑛 (𝑥 − 𝑎) ± ∑ 𝑞𝑛 (𝑥 − 𝑎) = ∑ 𝑐𝑛 (𝑥 − 𝑎)𝑛
𝑛 𝑛

𝑛=0 𝑛=0 𝑛=0

where 𝑐𝑛 = 𝑝𝑛 ± 𝑞𝑛 , for 𝑛 ≥ 0.
Moreover, the series

Tesfaye G and Wanaw B Page 149


CALCULUS II

∑ 𝑐𝑛 (𝑥 − 𝑎)𝑛
𝑛=0

converges in the interior of the common interval of convergence.


∞ ∞ ∞

𝑖𝑖. (∑ 𝑝𝑛 (𝑥 − 𝑎) ) (∑ 𝑞𝑛 (𝑥 − 𝑎) ) = ∑ 𝑐𝑛 (𝑥 − 𝑎)𝑛
𝑛 𝑛

𝑛=0 𝑛=0 𝑛=0

where 𝑐𝑛 = 𝑝𝑛 𝑞0 + 𝑝𝑛−1 𝑞1 + 𝑝𝑛−2 𝑞2 + ⋯ + 𝑝0 𝑞𝑛 .


Moreover, the series

∑ 𝑐𝑛 (𝑥 − 𝑎)𝑛
𝑛=0

converges in the interior of the common interval of convergence.


𝑛 ∞
∑∞
𝑛=0 𝑝𝑛 (𝑥 − 𝑎)
𝑖𝑖𝑖. = ∑ 𝑐𝑛 (𝑥 − 𝑎)𝑛
∑∞
𝑛=0 𝑞𝑛 (𝑥 − 𝑎) 𝑛
𝑛=0

where 𝑞𝑛 = 𝑐𝑛 𝑞0 + 𝑐𝑛−1 𝑞1 + 𝑐𝑛−2 𝑞2 + ⋯ + 𝑐0 𝑞𝑛 .


Example 4.3.8: Find the power series representation of
𝑥2 + 𝑥
1−𝑥
1
Solution: The power series representation of is
1−𝑥

1
= ∑ 𝑥𝑛 , for |𝑥| < 1
1−𝑥
𝑛=0

Thus,

𝑥2 + 𝑥
= (𝑥 2 + 𝑥) (∑ 𝑥 𝑛 )
1−𝑥
𝑛=0
∞ ∞
𝑛+2
= ∑𝑥 + ∑ 𝑥 𝑛+1
𝑛=0 𝑛=0
∞ ∞

= ∑ 𝑥𝑛 + ∑ 𝑥𝑛
𝑛=2 𝑛=1
∞ ∞
𝑛
= ∑𝑥 + ∑ 𝑥𝑛 +
𝑛=2 𝑛=2

Tesfaye G and Wanaw B Page 150


CALCULUS II

= ∑(1 + 1)𝑥 𝑛 + 𝑥
𝑛=2

= ∑ 2𝑥 𝑛 + 𝑥
𝑛=2
𝑥 2 +𝑥
Therefore, the power series representation of is
1−𝑥

𝑥2 + 𝑥
= ∑ 𝑐𝑛 𝑥 𝑛
1−𝑥
𝑛=0

where 𝑐0 = 0 , 𝑐1 = 1 and 𝑐𝑛 = 2, for 𝑛 ≥ 2.


Exercise 4.3.2: Find the power series representation of
𝑥3 + 𝑥2 + 𝑥 𝑥3 + 𝑥2
𝑎. 𝑏.
1−𝑥 1 + 𝑥2
4.3.4 Differentiation and integration of power series
At the end of this section students will be able to:
 Differentaite and integrate a power series in its interval of convergence.
 Find power series expansion of certain functions by manipulating geometric series
and then by differentiating or integrating such a series.

The sum of power series is a function


𝑓(𝑥) = ∑ 𝑐𝑛 (𝑥 − 𝑎)𝑛
𝑛=0

whose domain is the interval of convergence of the power series. We would like to be able to
differentiate and integrate such functions, and the following theorem says that we can do so by
differentiating or integrating each individual term in a series in its interval of convergence, just
as we would for a polynomial. This is called term by term differentiation and integration.
Theorem 4.3.2: Let the power series

∑ 𝑐𝑛 (𝑥 − 𝑎)𝑛
𝑛=0

has a radius of convergence 𝑅 > 0. Then, the function 𝑓 defined by


𝑓(𝑥) = ∑ 𝑐𝑛 (𝑥 − 𝑎)𝑛 = 𝑐0 + 𝑐1 (𝑥 − 𝑎) + 𝑐2 (𝑥 − 𝑎)2 + 𝑐3 (𝑥 − 𝑎)3 + 𝑐4 (𝑥 − 𝑎)4 + ⋯


𝑛=0

Tesfaye G and Wanaw B Page 151


CALCULUS II

is differentiable (therefore continuous) on an interval of convergence (𝑎 − 𝑅, 𝑎 + 𝑅) and


∞ ∞ ∞
𝑑 𝑑
𝑖. 𝑓 ′ (𝑥)
= (∑ 𝑐𝑛 (𝑥 − 𝑎)𝑛 ) = ∑ (𝑐𝑛 (𝑥 − 𝑎)𝑛 ) = ∑ 𝑛𝑐𝑛 (𝑥 − 𝑎)𝑛−1
𝑑𝑥 𝑑𝑥
𝑛=0 𝑛=0 𝑛=1
∞ ∞ ∞
𝑛 𝑛 )𝑑𝑥
𝑐𝑛 (𝑥 − 𝑎)𝑛+1
𝑖𝑖. ∫ 𝑓(𝑥)𝑑𝑥 = ∫ (∑ 𝑐𝑛 (𝑥 − 𝑎) ) 𝑑𝑥 = ∑ ∫(𝑐𝑛 (𝑥 − 𝑎) =∑ +𝑐
𝑛+1
𝑛=0 𝑛=0 𝑛=1

Moreover, both power series in (𝑖) and (𝑖𝑖) above has a radius of convergenc 𝑅.
Note: Although the Theorem says that the radius of convergence remains the same when the
power series is differentiated or integrated, this does not mean that the interval of convergence
remains the same. It may happen that the original series converges at an end point, where as the
differentiated or integrated series diverges their.
Example 4.3.9: Given the series

𝑥𝑛
∑ 2
𝑛
𝑛=1

i. Find the radius of convergence and interval of convergence of the given power series.
ii. Find the radius of convergence and interval of convergence of the series


𝑑 𝑥𝑛
(∑ 2 )
𝑑𝑥 𝑛
𝑛=1

iii. Find the radius of convergence and interval of convergence of the series


𝑥𝑛
∫ (∑ ) 𝑑𝑥
𝑛2
𝑛=1

Solution:
𝑥𝑛
i. Observe that the power series ∑∞
𝑛=1 𝑛2 is a power series about 0.

And,
1 1
𝑐𝑛 = and 𝑐𝑛+1 =
𝑛2 (𝑛 + 1)2
Then,
𝑐𝑛 𝑛2 + 2𝑛 + 1 𝑛2 + 2𝑛 + 1
lim | | = lim | | = lim =1
𝑛→∞ 𝑐𝑛+1 𝑛→∞ 𝑛2 𝑛→∞ 𝑛2

Tesfaye G and Wanaw B Page 152


CALCULUS II

𝑥𝑛
Therefore, the radius of convergence of the power series ∑∞
𝑛=1 𝑛2 is 𝑅 = 1 and the series

converges if |𝑥| < 1 and diverges |𝑥| > 1.


We need to check at the convergence of the series at the end points of the interval (−1,1), thast
is at 𝑥 = −1 and at 𝑥 = 1.
If 𝑥 = 1, the series becomes

1

𝑛2
𝑛=1

which is a 𝑝-series with 𝑝 = 2 > 1, hence it is convergent series.


If 𝑥 = −1, the series becomes

(−1)𝑛

𝑛2
𝑛=1

which is an alternating series and hence it is converges by Leibnitz’s test.


𝑥𝑛
Therefore, the radius of convergence of the power series ∑∞
𝑛=1 𝑛2 is 𝑅 = 1 and its interval of

convergence is [−1,1].
ii.

∞ ∞ ∞ ∞ ∞
𝑑 𝑥𝑛 𝑑 𝑥𝑛 𝑛𝑥 𝑛−1 𝑥 𝑛−1 𝑥𝑛
(∑ 2 ) = ∑ ( 2) = ∑ = ∑ = ∑
𝑑𝑥 𝑛 𝑑𝑥 𝑛 𝑛2 𝑛 𝑛+1
𝑛=1 𝑛=1 𝑛=1 𝑛=1 𝑛=0

Thus, for the power series


∞ ∞
𝑑 𝑥𝑛 𝑥𝑛
(∑ 2 ) = ∑
𝑑𝑥 𝑛 𝑛+1
𝑛=1 𝑛=0

which is a power series about 0, we have


1 1
𝑐𝑛 = and 𝑐𝑛+1 =
𝑛+1 𝑛+2
Then,
𝑐𝑛 𝑛+2 𝑛+2
lim | | = lim | | = lim =1
𝑛→∞ 𝑐𝑛+1 𝑛→∞ 𝑛 + 1 𝑛→∞ 𝑛 + 1

𝑑 𝑥𝑛 𝑥𝑛
Therefore, the radius of convergence of the power series (∑∞
𝑛=1 2) =
∑∞
𝑛=0 is 𝑅 = 1 that
𝑑𝑥 𝑛 𝑛+1
𝑥𝑛 𝑑 𝑥𝑛 𝑥𝑛
is the same as the radius of convergenc of ∑∞
𝑛=1 𝑛2 . And the series 𝑑𝑥
(∑∞ ∞
𝑛=1 𝑛2 ) = ∑𝑛=0 𝑛+1

converges if |𝑥| < 1 and diverges |𝑥| > 1.

Tesfaye G and Wanaw B Page 153


CALCULUS II

𝑑 𝑥𝑛 𝑥𝑛
Checking the convergence or divergence of the series (∑∞ ∞
𝑛=1 𝑛2 ) = ∑𝑛=0 𝑛+1 at the two end
𝑑𝑥

points of the interval (−1,1), thast is at 𝑥 = −1 and at 𝑥 = 1.


If 𝑥 = 1, the series becomes
∞ ∞
1 1
∑ = 1+ ∑
𝑛+1 𝑛+1
𝑛=0 𝑛=1

which is divergent series by Limit comparison test.


If 𝑥 = −1, the series becomes

(−1)𝑛

𝑛+1
𝑛=0

which is an alternating series and hence it is converges by Leibnitz’s test.


𝑑 𝑥𝑛 𝑥𝑛
Therefore, the radius of convergence of the power series (∑∞ ∞
𝑛=1 𝑛2 ) = ∑𝑛=0 𝑛+1 is 𝑅 = 1 and
𝑑𝑥

its interval of convergence is [−1,1).


iii.

∞ ∞ ∞ ∞
𝑥𝑛 𝑥𝑛 𝑥 𝑛+1 𝑥𝑛
∫ (∑ 2 ) 𝑑𝑥 = ∑ ∫ 2 𝑑𝑥 = ∑ + 𝑐 = ∑ +𝑐
𝑛 𝑛 (𝑛 + 1)𝑛2 𝑛(𝑛 − 1)2
𝑛=1 𝑛=0 𝑛=1 𝑛=2
𝑥𝑛
The convergence or the divergence of the power series ∑∞
𝑛=2 𝑛(𝑛−1)2 + 𝑐 depoendes ohn the

𝑥𝑛
convergence or divergence of the power series ∑∞
𝑛=2 𝑛(𝑛−1)2 .

Thus, for the power series



𝑥𝑛

𝑛(𝑛 − 1)2
𝑛=2

which is a power series about 0, we have


1 1
𝑐𝑛 = and 𝑐𝑛+1 =
𝑛(𝑛 − 1)2 (𝑛 + 1)𝑛2
Then,
𝑐𝑛 (𝑛 + 1)𝑛2
lim | | = lim =1
𝑛→∞ 𝑐𝑛+1 𝑛→∞ 𝑛(𝑛 − 1)2

Tesfaye G and Wanaw B Page 154


CALCULUS II

𝑥𝑛 𝑥𝑛
Therefore, the radius of convergence of the power series ∫ (∑∞ ∞
𝑛=1 𝑛2 ) 𝑑𝑥 = ∑𝑛=2 𝑛(𝑛−1)2 + 𝑐 is

𝑥𝑛
𝑅 = 1 and that is the same as the radius of convergence of ∑∞
𝑛=1 𝑛2 . And the series

𝑥𝑛 𝑥𝑛
∫ (∑∞ ∞
𝑛=1 𝑛2 ) 𝑑𝑥 = ∑𝑛=2 𝑛(𝑛−1)2 + 𝑐 converges if |𝑥| < 1 and diverges |𝑥| > 1.

𝑥𝑛
Checking the convergence or divergence of the series ∑∞
𝑛=2 𝑛(𝑛−1)2 + 𝑐 at the two end points of

the interval (−1,1), thast is at 𝑥 = −1 and at 𝑥 = 1.


If 𝑥 = 1, the series becomes

1

𝑛(𝑛 − 1)2
𝑛=2

which is a convergent series by Limit comparison test ( comparison test).


If 𝑥 = −1, the series becomes

(−1)𝑛

𝑛(𝑛 − 1)2
𝑛=2

which is an alternating series and hence it is converges by Leibnitz’s test.


𝑥𝑛 𝑥𝑛
Therefore, the radius of convergence of the power series ∫ (∑∞ ∞
𝑛=1 𝑛2 ) 𝑑𝑥 = ∑𝑛=2 𝑛(𝑛−1)2 + 𝑐is

𝑅 = 1 and its interval of convergence is [−1,1].


Example 4.3.10: Express
1
(1 − 𝑥)2
as a power series an find the radius of convergence and interval of convergence of its power
series.
Solution: We know that

1
= ∑ 𝑥𝑛
1−𝑥
𝑛=0

having a radius of convergence 𝑅 = 1.


And observe that
𝑑 1 1
( )=
𝑑𝑥 1 − 𝑥 (1 − 𝑥)2

Tesfaye G and Wanaw B Page 155


CALCULUS II

∞ ∞ ∞ ∞
𝑑 1 𝑑 𝑑 𝑛
⟹ ( )= (∑ 𝑥 𝑛 ) = ∑ 𝑥 = ∑ 𝑛𝑥 𝑛−1 = ∑(𝑛 + 1)𝑥 𝑛
𝑑𝑥 1 − 𝑥 𝑑𝑥 𝑑𝑥
𝑛=0 𝑛=0 𝑛=1 𝑛=0

Hence,

1
= ∑(𝑛 + 1)𝑥 𝑛
(1 − 𝑥)2
𝑛=0

is the power series representation.


And by the above Theorem the radius of convergence of the differentiated series is the same as
1
the radius of convergence of the original series. Thus, (1−𝑥)2
has a power series representation

∑∞ 𝑛
𝑛=0(𝑛 + 1)𝑥 having radius of convergence 𝑅 = 1 and this series converges for |𝑥| < 1 and

diverges |𝑥| > 1.


To find the interval of convergence we need to check the two end points of the interval (−1,1).
If 𝑥 = 1, the series becomes

∑(𝑛 + 1)
𝑛=2

which is a divergent series.


If 𝑥 = −1, the series becomes

∑(−1)𝑛 (𝑛 + 1)
𝑛=2

which is an alternating series and hence it is divergent series.


1 𝑛
Therefore, (1−𝑥)2 has a power series representation ∑∞
𝑛=0(𝑛 + 1)𝑥 having radius of convergence

𝑅 = 1 and interval of convergence is (−1,1).


Example 4.3.11:
a. Express

−1
(1 + 𝑥)2
as a power series an find the radius of convergence of its power series.
b. Use (a) to find the power series representation of

−1
(1 + 𝑥)2

Tesfaye G and Wanaw B Page 156


CALCULUS II

and find the radius of convergence of its power series.


Solution:
a. We know that

∞ ∞
1 1
= = ∑(−𝑥)𝑛 = ∑(−1)𝑛 𝑥 𝑛
1 + 𝑥 1 − (−𝑥)
𝑛=0 𝑛=0

having a radius of convergence 𝑅 = 1.


And observe that
𝑑 1 −1
( )=
𝑑𝑥 1 + 𝑥 (1 + 𝑥)2
∞ ∞
𝑑 1 −1 𝑑 𝑑
⟹ ( )= = (∑(−1)𝑛 𝑥 𝑛 ) = ∑ ((−1)𝑛 𝑥 𝑛 )
𝑑𝑥 1 + 𝑥 (1 + 𝑥) 2 𝑑𝑥 𝑑𝑥
𝑛=0 𝑛=0

= ∑(−1)𝑛 𝑛𝑥 𝑛−1
𝑛=1

= ∑(−1)𝑛+1 (𝑛 + 1)𝑥 𝑛
𝑛=0
−1 𝑛+1 (𝑛
Hence, (1+𝑥)2 has a power series representation ∑∞
𝑛=0(−1) + 1)𝑥 𝑛 . That is

−1
= ∑(−1)𝑛+1 (𝑛 + 1)𝑥 𝑛
(1 + 𝑥)2
𝑛=0

And by the above Theorem the radius of convergence of the differentiated series is the same as
the radius of convergence of the original series.
𝑛+1 (𝑛 −1
Therefore, the power series expansion ∑∞
𝑛=0(−1) + 1)𝑥 𝑛 of (1+𝑥)2
have radius of

convergence 𝑅 = 1.
b. Using (a), that is,


−1
= ∑(−1)𝑛+1 (𝑛 + 1)𝑥 𝑛
(1 + 𝑥)2
𝑛=0

We have,

−𝑥
= 𝑥 (∑(−1)𝑛+1 (𝑛 + 1)𝑥 𝑛 )
(1 + 𝑥)2
𝑛=0

Tesfaye G and Wanaw B Page 157


CALCULUS II

= ∑(−1)𝑛+1 (𝑛 + 1)𝑥 𝑛+1


𝑛=0

= ∑(−1)𝑛 𝑛𝑥 𝑛
𝑛=1

having radius of convergence 𝑅 = 1.


Example 4.3.12: Find the power series representation of 𝑙𝑛(1 + 𝑥) and find the radius of
convergence of the power series.
1
Solution: Observe that the derivative of 𝑙𝑛(1 + 𝑥) is 1+𝑥 . Thus,
1
𝑙𝑛(1 + 𝑥) = ∫ 𝑑𝑥
1+𝑥
But,

1
= ∑(−1)𝑛 𝑥 𝑛
1+𝑥
𝑛=0

which has radius of convergence 𝑅 = 1.


Thus,
∞ ∞
1 𝑛 𝑛 𝑛
𝑥 𝑛+1
𝑙𝑛(1 + 𝑥) = ∫ 𝑑𝑥 = ∫ (∑(−1) 𝑥 ) 𝑑𝑥 = ∑(−1) +𝑐
1+𝑥 𝑛+1
𝑛=0 𝑛=0

𝑥𝑛
= ∑(−1)𝑛 +𝑐
𝑛
𝑛=1

𝑛 𝑥𝑛
Therefore, 𝑙𝑛(1 + 𝑥) has a power seriesn representation ∑∞
𝑛=1(−1) + 𝑐 and by the above
𝑛

theorem the radius of convergence of the integrated series is equal to the radius of convergence
of the origional series.
𝑛 𝑥𝑛
Therefore, the radius of convergence of the power series ∑∞
𝑛=1(−1) + 𝑐 is 𝑅 = 1.
𝑛
𝑥3
Example 4.2.13: Find the power series representation of (2−4𝑥)2
and find the radius of

convergence of the power series.


1
Solution: Now, manipulating 2−4𝑥 as a geopmetric series
1 1 1
= ( )
2 − 4𝑥 2 1 − 2𝑥
But,

Tesfaye G and Wanaw B Page 158


CALCULUS II


1
= ∑ 2𝑛 𝑥 𝑛
1 − 2𝑥
𝑛=0
1
which have radius of convergence 𝑅 = 2 .

Thus,

1 1
= 2( ) = ∑ 2𝑛−1 𝑥 𝑛
2 − 4𝑥 1 − 2𝑥
𝑛=0
1
which have a radius of convergence 𝑅 = 2.

But, observe that


𝑑 1 4
( )=
𝑑𝑥 2 − 4𝑥 (2 − 4𝑥)2
∞ ∞
𝑑 1 4 𝑑
⟹ ( )= = (∑ 2𝑛−1 𝑥 𝑛 ) = ∑ 𝑛2𝑛−1 𝑥 𝑛−1
𝑑𝑥 2 − 4𝑥 (2 − 4𝑥) 2 𝑑𝑥
𝑛=0 𝑛=1
∞ ∞
4
⟹ = ∑ 𝑛2𝑛−1 𝑥 𝑛−1 = ∑(𝑛 + 1)2𝑛 𝑥 𝑛
(2 − 4𝑥)2
𝑛=1 𝑛=0
∞ ∞
1 1
⟹ = (∑(𝑛 + 1)2𝑛 𝑥 𝑛 ) = ∑(𝑛 + 1)2𝑛−2 𝑥 𝑛
(2 − 4𝑥) 2 4
𝑛=0 𝑛=0
1
Thus, the power series representation of (2−4𝑥)2 is

∑(𝑛 + 1)2𝑛−2 𝑥 𝑛
𝑛=0
1
and by the above theorem the radius of convergence of the this power series is 𝑅 = 2.

Then,
∞ ∞ ∞
𝑥3
= 𝑥 3 (∑(𝑛 + 1)2𝑛−2 𝑥 𝑛 ) = ∑(𝑛 + 1)2𝑛−2 𝑥 𝑛+3 = ∑(𝑛 − 2)2𝑛−5 𝑥 𝑛
(2 − 4𝑥)2
𝑛=0 𝑛=0 𝑛=3

Therefore,

∑(𝑛 − 2)2𝑛−5 𝑥 𝑛
𝑛=3
𝑥3
is the power series representation of (2−4𝑥)2 having radus of convergence 𝑅 = 1.

Tesfaye G and Wanaw B Page 159


CALCULUS II

𝑥
Example 4.2.14: Find the power series representation of 𝑡𝑎𝑛−1 (2) and find the radius of

convergence of the power series.


Solution: Observe that
𝑥
1 𝑡𝑎𝑛−1 (𝑎)
∫ 2 𝑑𝑥 =
𝑥 + 𝑎2 𝑎
𝑎 𝑥
⟹ ∫ 2 2
𝑑𝑥 = 𝑡𝑎𝑛−1 ( )
𝑥 +𝑎 𝑎
Thus,
2 −1
𝑥
∫ 𝑑𝑥 = 𝑡𝑎𝑛 ( )
𝑥2 + 4 2
But,
∞ 𝑛 ∞
2 1 1 𝑥2 1 𝑛
𝑥 2𝑛
= = (∑ (− ) ) = (∑(−1) 𝑛 )
𝑥2 + 4 𝑥2 2 4 2 4
2 (1 − (− 4 )) 𝑛=0 𝑛=0


1 𝑥 2𝑛
= (∑(−1)𝑛 2𝑛 )
2 2
𝑛=0

𝑥 2𝑛
= ∑(−1)𝑛
22𝑛+1
𝑛=0
2
Hence, 𝑥 2 +4 has a power series representation of

𝑥 2𝑛
∑(−1)𝑛
22𝑛+1
𝑛=0

and this seies has a radius of convergence 𝑅 = 4.


Then,
𝑥 2
𝑡𝑎𝑛−1 ( ) = ∫ 2 𝑑𝑥
2 𝑥 +4

𝑥 2𝑛
= ∫ (∑(−1)𝑛 ) 𝑑𝑥
22𝑛+1
𝑛=0

𝑥 2𝑛+1
𝑛
= ∑(−1) 2𝑛+1
2 (2𝑛 + 1)
𝑛=0

Therefore,

Tesfaye G and Wanaw B Page 160


CALCULUS II


𝑥 2𝑛+1
∑(−1)𝑛
22𝑛+1 (2𝑛 + 1)
𝑛=0
𝑥
is the power series representation of 𝑡𝑎𝑛−1 (2). And this series has a radius of convergence

𝑅 = 4.
Exercise 4.3.3:
1. Find the power series representation of 𝑓 and determine the radius of convergence and
interval of convergence of the power series.
𝑥3
a. 𝑓(𝑥) = (3𝑥+2)2 e. 𝑓(𝑥) = ln(5 − 𝑥)
1
b. 𝑓(𝑥) = (𝑥+1)2 f. 𝑓(𝑥) = 𝑡𝑎𝑛−1 (2𝑥)

2. Evaluate the indefinite integral as a power series. What is the radius of convergence?
𝑥
a. ∫ 1−𝑥 8 𝑑𝑥
ln(1−𝑥)
b. ∫ 𝑑𝑥
𝑥2
𝑥
c. ∫ 𝑡𝑎𝑛−1 (5) 𝑑𝑥

4.3.5Taylor series and Maclaurin Series


At the end of this section students will be able to:
 Find Taylor series expansion and Maclaurine series expansion of a function.

In this section we were able to find the power series representation of certain functions. Here, we
investigate the more general problem.
Taylor Series
If a function 𝑓 has a power series representation at 𝑎, that is if

𝑓(𝑥) = ∑ 𝑐𝑛 (𝑥 − 𝑎)𝑛 , |𝑥 − 𝑎| < 𝑅


𝑛=0

then, the coefficient are given by the foermula


𝑓 (𝑛) (𝑎)
𝑐𝑛 =
𝑛!
where 𝑓 (0) (𝑎) = 𝑓(𝑎) and 𝑓 (𝑛) (𝑎) is the nth drivative of 𝑓 at 𝑎.
Substituting 𝑐𝑛 ’s back in to the series it will give us the following statement.
If a function 𝑓 has a power series representation at 𝑎, then

Tesfaye G and Wanaw B Page 161


CALCULUS II


𝑓 (𝑛) (𝑎)
𝑓(𝑥) = ∑ (𝑥 − 𝑎)𝑛
𝑛!
𝑛=0

𝑓 ′ (𝑎) 𝑓 ′′ (𝑎) 𝑓 ′′′ (𝑎) 𝑓 (4) (𝑎)


= 𝑓(𝑎) + (𝑥 − 𝑎) + (𝑥 − 𝑎)2 + (𝑥 − 𝑎)3 + (𝑥 − 𝑎)4 + ⋯
1! 2! 3! 4!
This power series is called Taylor series expansion of a function at 𝑎.
In particular if we take 𝑎 = 0, we will have the following.
If a function 𝑓 has a power series representation at 0, then

𝑓 (𝑛) (0) 𝑛
𝑓(𝑥) = ∑ 𝑥
𝑛!
𝑛=0

𝑓 ′ (0) 𝑓 ′′ (0) 2 𝑓 ′′′ (0) 3 𝑓 (4) (0) 4


= 𝑓(0) + 𝑥+ 𝑥 + 𝑥 + 𝑥 +⋯
1! 2! 3! 4!
This power series is called Maclaurine series expansion, which is a particular case of Taylor
series expansion of a function.
Exmple 4.3.15: Find the Taylor series expansion of the function 𝑓(𝑥) = 𝑒 𝑥 at 0.
Solution: If 𝑓 has a power series representation at 0,

𝑓(𝑥) = ∑ 𝑐𝑛 𝑥 𝑛
𝑛=0

𝑓 (𝑛) (0)
where 𝑐𝑛 = .
𝑛!

Thus,

𝑒 𝑥 = ∑ 𝑐𝑛 𝑥 𝑛
𝑛=0

𝑓 (𝑛) (0)
where 𝑐𝑛 = and 𝑓(𝑥) = 𝑒 𝑥 .
𝑛!

But,
𝑓 (0) (0) = 𝑓(0) = 1 , 𝑓 ′ (0) = 1 , 𝑓 ′′ (0) = 1 , 𝑓 ′′′ (0) = 1 , … , 𝑓 (𝑛) (0) = 1 , for all 𝑛
≥0
Hence, the Taylor series expanssion of 𝑓(𝑥) = 𝑒 𝑥 at 0, that is the Maclaurine series expansion of
𝑓(𝑥) = 𝑒 𝑥 ) is

𝑥 𝑥2 𝑥3 𝑥4 𝑥5
𝑒𝑥 = 1 + + + + + +⋯
1! 2! 3! 4! 5!

Tesfaye G and Wanaw B Page 162


CALCULUS II

That is,

𝑥
𝑥𝑛
𝑒 =∑
𝑛!
𝑛=0

Here, in the Maclaurine series expansinon of the function 𝑓(𝑥) = 𝑒 𝑥 , that is



𝑥
𝑥𝑛
𝑒 =∑
𝑛!
𝑛=0

we have
1 1
𝑐𝑛 = and 𝑐𝑛+1 =
(𝑛 + 1)! (𝑛 + 1)!
Thus,
𝑐𝑛 (𝑛 + 1)! (𝑛 + 1)𝑛!
lim | | = lim | | = lim | | = lim (𝑛 + 1) = ∞
𝑛→∞ 𝑐𝑛+1 𝑛→∞ 𝑛! 𝑛→∞ 𝑛! 𝑛→∞

Therefore the Taylor seires expansion of 𝑒 𝑥 at 0 (Maclaurin series of 𝑒 𝑥 ) is



𝑥
𝑥𝑛
𝑒 =∑
𝑛!
𝑛=0

having radius of convergence 𝑅 = ∞ and its interval of converghence is (−∞, ∞).


Problem: What functions have a power series representation or under what circumstance is a
function equal to the sum of its power series? If a function 𝑓 has a derivative of all order at 𝑎,
𝑓 (𝑛) (𝑎)
when it is true that 𝑓(𝑥) = ∑∞
𝑛=0 (𝑥 − 𝑎)𝑛 ?
𝑛!

In the case of Taylor series the partial sums of the series



𝑓 (𝑛) (𝑎)
∑ (𝑥 − 𝑎)𝑛
𝑛!
𝑛=0

are
𝑛
𝑓 (𝑖) (𝑎)
𝑇𝑛 (𝑥) = ∑ (𝑥 − 𝑎)𝑖
𝑖!
𝑖=0

𝑓 ′ (𝑎) 𝑓 ′′ (𝑎) 𝑓 (𝑛) (𝑎)


= 𝑓(𝑎) + (𝑥 − 𝑎) + (𝑥 − 𝑎)2 + ⋯ + (𝑥 − 𝑎)4
1! 2! 𝑛!
If we let
𝑅𝑛 (𝑥) = 𝑓(𝑥) − 𝑇𝑛 (𝑥)

Tesfaye G and Wanaw B Page 163


CALCULUS II

Definition 4.3.3: If a function 𝑓 has a derivative of all order at 𝑎, then


𝑛
𝑓 (𝑖) (𝑎)
𝑇𝑛 (𝑥) = ∑ (𝑥 − 𝑎)𝑖
𝑖!
𝑖=0

𝑓 ′ (𝑎) 𝑓 ′′ (𝑎) 2
𝑓 (𝑛) (𝑎)
= 𝑓(𝑎) + (𝑥 − 𝑎) + (𝑥 − 𝑎) + ⋯ + (𝑥 − 𝑎)4
1! 2! 𝑛!
is called the 𝑛th degree Taylor polynomial of 𝑓 at 𝑎. And the expresion 𝑅𝑛 (𝑥) where
𝑅𝑛 (𝑥) = 𝑓(𝑥) − 𝑇𝑛 (𝑥)
is called the remainder of the Taylor series.
Theorem 4.3.3: If
lim 𝑅𝑛 (𝑥) = 0 , for |𝑥 − 𝑎| < 𝑅
𝑛→∞

then, 𝑓 equals to the sum of its Taylor series on the interval |𝑥 − 𝑎| < 𝑅.
Proof: Suppose 𝑇𝑛 (𝑥) be the nth degree Taylor polynomial of 𝑓 at 𝑎 and 𝑅𝑛 (𝑥) be the remainder
of the Taylor series.
By our assumption
𝑙𝑖𝑚 𝑅𝑛 (𝑥) = 0 , 𝑓𝑜𝑟 |𝑥 − 𝑎| < 𝑅
𝑛→∞

But,
𝑅𝑛 (𝑥) = 𝑓(𝑥) − 𝑇𝑛 (𝑥)
⟹ 𝑇𝑛 (𝑥) = 𝑓(𝑥) − 𝑅𝑛 (𝑥)
⟹ 𝑙𝑖𝑚 𝑇𝑛 (𝑥) = 𝑙𝑖𝑚 [𝑓(𝑥) − 𝑅𝑛 (𝑥)] = 𝑓(𝑥)
𝑛→∞ 𝑛→∞

But,
𝑛 ∞
𝑓 (𝑖) (𝑎) 𝑖
𝑓 (𝑛) (𝑎)
𝑙𝑖𝑚 𝑇𝑛 (𝑥) = 𝑙𝑖𝑚 (∑ (𝑥 − 𝑎) ) = ∑ (𝑥 − 𝑎)𝑛
𝑛→∞ 𝑛→∞ 𝑖! 𝑛!
𝑖=0 𝑛=0

𝑓 (𝑛) (𝑎)
⟹ 𝑓(𝑥) = ∑ (𝑥 − 𝑎)𝑛
𝑛!
𝑛=0

Therefore, 𝑓 equals to the sum of its Taylor series on the interval |𝑥 − 𝑎| < 𝑅.
In trying to show that 𝑙𝑖𝑚𝑛→∞ 𝑅𝑛 (𝑥) = 0 for a specific function of 𝑓 we use the followin fact.
Theorem 4.3.4: Taylor’s Theorem: If |𝑓 (𝑛+1) (𝑥)| ≤ 𝑀 , for |𝑥 − 𝑎| < 𝑑, then the remainder
𝑅𝑛 (𝑥) of a Taylor series satisfies the inequality
𝑀
|𝑅𝑛 (𝑥)| ≤ |𝑥 − 𝑎|𝑛+1 , for |𝑥 − 𝑎| < 𝑑
(𝑛 + 1)!

Tesfaye G and Wanaw B Page 164


CALCULUS II

Example 4.3.16: Prove that 𝑒 𝑥 equals to the sum of its Maclaurine series.
Solution: Let 𝑓(𝑥) = 𝑒 𝑥 .
𝑓 (𝑛) (𝑥) = 𝑒 𝑥 , for all 𝑛 ≥ 0
⟹ 𝑓 (𝑛+1) (𝑥) = 𝑒 𝑥 , for all 𝑛 ≥ 0
If 𝑑 is any positive number and |𝑥| < 𝑑, then
|𝑓 (𝑛+1) (𝑥)| = |𝑒 𝑥 | = 𝑒 𝑥 < 𝑒 𝑑
So, the the Taylor inequality, with 𝑎 = 0 and 𝑀 = 𝑒 𝑑 , we have
𝑒𝑑
|𝑅𝑛 (𝑥)| ≤ |𝑥|𝑛+1 , for |𝑥| < 𝑑
(𝑛 + 1)!
−𝑒 𝑑 𝑒𝑑
⟹ |𝑥|𝑛+1 ≤ 𝑅𝑛 (𝑥) ≤ |𝑥|𝑛+1 , for |𝑥| < 𝑑
(𝑛 + 1)! (𝑛 + 1)!
But,
−𝑒 𝑑 𝑒𝑑
𝑙𝑖𝑚 ( |𝑥|𝑛+1 ) = 𝑙𝑖𝑚 ( |𝑥|𝑛+1 )
𝑛→∞ (𝑛 + 1)! 𝑛→∞ (𝑛 + 1)!

Thus, it follows that by Squeezing Theorem


𝑙𝑖𝑚𝑛→∞ 𝑅𝑛 (𝑥) = 0
and therefore,
𝑙𝑖𝑚 𝑅𝑛 (𝑥) = 0
𝑛→∞

for all values of 𝑥.


Therefore, by the above Theorem, 𝑒 𝑥 is equal to the sum of its power series at 0 and

𝑥
𝑥𝑛
𝑒 =∑ , for all 𝑥
𝑛!
𝑛=0

Example 4.3.17: Find the Maclaurin series expansion of 𝑠𝑖𝑛𝑥.


Solution: Let 𝑓(𝑥) = 𝑠𝑖𝑛𝑥.
Thus,
𝑓(0) = 𝑠𝑖𝑛0 = 0
𝑓 ′ (𝑥) = 𝑐𝑜𝑠𝑥 ⟹ 𝑓 ′ (0) = 𝑐𝑜𝑠0 = 1
𝑓 ′′ (𝑥) = −𝑠𝑖𝑛𝑥 ⟹ 𝑓 ′ (0) = −𝑠𝑖𝑛0 = 0
𝑓 ′′′ (𝑥) = −𝑐𝑜𝑠𝑥 ⟹ 𝑓 ′′′ (0) = −𝑐𝑜𝑠0 = −1
𝑓 (4) (𝑥) = 𝑠𝑖𝑛𝑥 ⟹ 𝑓 (4) (0) = 𝑠𝑖𝑛0 = 0

Tesfaye G and Wanaw B Page 165


CALCULUS II

𝑓 (5) (𝑥) = 𝑐𝑜𝑠𝑥 ⟹ 𝑓 (5) (0) = 𝑐𝑜𝑠0 = −1


𝑓 (6) (𝑥) = −𝑠𝑖𝑛𝑥 ⟹ 𝑓 (6) (0) = −𝑠𝑖𝑛0 = 0
𝑓 (7) (𝑥) = −𝑐𝑜𝑠𝑥 ⟹ 𝑓 (7) (0) = −𝑐𝑜𝑠0 = −1 …
Thus,
0 , 𝑖𝑓 𝑛 = 0,2,4,6,8, …
𝑓(𝑥) = { 1 , 𝑖𝑓 𝑛 = 1,5,9,13,17, …
−1 , 𝑖𝑓 𝑛 = 3,7,11,15,19, …
Thus,

𝑓 (𝑛) (0) 𝑛
𝑠𝑖𝑛𝑥 = ∑ 𝑥
𝑛!
𝑛=0

𝑓 ′ (0) 𝑓 ′′ (0) 2 𝑓 ′′′ (0) 3 𝑓 (4) (0) 4


⟹ 𝑠𝑖𝑛𝑥 = 𝑓(0) + 𝑥+ 𝑥 + 𝑥 + 𝑥 +⋯
1! 2! 3! 𝑛!

Thus, the Maclaurine series of 𝑠𝑖𝑛𝑥 is


1 0 (−1) 3 0 4 1 5 0 6 (−1) 7
𝑠𝑖𝑛𝑥 = 0 + 𝑥 + 𝑥2 + 𝑥 + 𝑥 + 𝑥 + 𝑥 + 𝑥 +⋯
1! 2! 3! 4! 5! 6! 7!
1 (−1) 3 1 5 (−1) 7
⟹ 𝑠𝑖𝑛𝑥 = 𝑥 + 𝑥 + 𝑥 + 𝑥 +⋯
1! 3! 5! 7!
𝑥 3 𝑥 5 𝑥 7 𝑥 9 𝑥11
⟹ 𝑠𝑖𝑛𝑥 = 𝑥 − + − + − +⋯
3! 5! 7! 9! 11!
Therefore,

𝑥 2𝑛+1
𝑛
𝑠𝑖𝑛𝑥 = ∑(−1)
(2𝑛 + 1)!
𝑛=0

is the Maclauyrine series rerpresentation of 𝑠𝑖𝑛𝑥, with raduius of convergence 𝑅 = ∞, and hence
its interval of convergence is (−∞, ∞).
Now, it is clear that 𝑓 (𝑛+1) (𝑥) is ±𝑠𝑖𝑛𝑥 or ±𝑐𝑜𝑠𝑥 and thus
|𝑓 (𝑛+1) (𝑥)| ≤ 1 , for all 𝑥
Thus, we can take 𝑀 = 1 and by Taylor inequality
1
|𝑅𝑛 (𝑥)| ≤ |𝑥|𝑛+1 , for |𝑥| < 1
(𝑛 + 1)!
By squeezing thjeorem we will obtain that
𝑙𝑖𝑚 𝑅𝑛 (𝑥) = 0
𝑛→∞

Tesfaye G and Wanaw B Page 166


CALCULUS II

Therefore, 𝑠𝑖𝑛𝑥 is equal to its Taylor series at 0 and



𝑥 2𝑛+1 𝑛
𝑠𝑖𝑛𝑥 = ∑(−1)
(2𝑛 + 1)!
𝑛=0

Exrecise 4.3.4: Show that the Maclaurine series representation for 𝑐𝑜𝑠𝑥 is

𝑥 2𝑛
𝑐𝑜𝑠𝑥 = ∑(−1)𝑛
(2𝑛)!
𝑛=0

 The following are important Taylor expansions that are useful to find the Taylor
expansion of a certain functions.

1
1. = ∑ 𝑥𝑛 , for |𝑥| < 1
1−𝑥
𝑛=0

𝑥
𝑥𝑛
2. 𝑒 =∑ , for all 𝑥
𝑛!
𝑛=0

𝑥 2𝑛+1 𝑛
3. 𝑠𝑖𝑛𝑥 = ∑(−1) , for all 𝑥
(2𝑛 + 1)!
𝑛=0

𝑥 2𝑛
4. 𝑐𝑜𝑠𝑥 = ∑(−1)𝑛 , for all 𝑥
(2𝑛)!
𝑛=0

−1
𝑥 2𝑛+1 𝑛
5. 𝑡𝑎𝑛 𝑥 = ∑(−1) , for |𝑥| < 1
2𝑛 + 1
𝑛=0

Example 4.3.18: Find the Maclaurine series of 𝑒 −2𝑥 .


Solution: Simply replacing 𝑥 by −2𝑥 in the series

𝑥𝑛
𝑒𝑥 = ∑
𝑛!
𝑛=0

we have

−2𝑥
𝑥𝑛 𝑛
𝑒 = ∑(−2) , for all 𝑥
𝑛!
𝑛=0

which is the Maclaurine series expansion of 𝑒 −2𝑥 .


2
Example 4.3.19: Find the Maclaurine series expansion of 𝑒 −𝑥 + 𝑐𝑜𝑠𝑥 and find its interval of
convergence.

Tesfaye G and Wanaw B Page 167


CALCULUS II

2
Solution: The Maclaurine series expansion of 𝑒 −𝑥 is a series obtaind by replacing −𝑥 2 by 𝑥 of
a series

𝑥
𝑥𝑛
𝑒 =∑ , for all 𝑥
𝑛!
𝑛=0

That is

−𝑥 2
(−𝑥 2 )𝑛
𝑒 =∑ , for all 𝑥
𝑛!
𝑛=0

−𝑥 2
(−1)𝑛 2𝑛
⟹ 𝑒 =∑ 𝑥 , for all 𝑥
𝑛!
𝑛=0

And we have

𝑥 2𝑛
𝑐𝑜𝑠𝑥 = ∑(−1)𝑛 , for all 𝑥
(2𝑛)!
𝑛=0

Thus,
∞ ∞
−𝑥 2
(−1)𝑛 2𝑛 𝑥 2𝑛
𝑒 + 𝑐𝑜𝑠𝑥 = ∑ 𝑥 + ∑(−1)𝑛
𝑛! (2𝑛)!
𝑛=0 𝑛=0

(−1)𝑛 (−1)𝑛 2𝑛
= ∑( + )𝑥
𝑛! (2𝑛)!
𝑛=0

1 1
= ∑(−1)𝑛 ( + ) 𝑥 2𝑛
𝑛! (2𝑛)!
𝑛=0

Hence,

1 1
∑(−1)𝑛 ( + ) 𝑥 2𝑛
𝑛! (2𝑛)!
𝑛=0
2
is the Maclaurine series expansion of 𝑒 −𝑥 + 𝑐𝑜𝑠𝑥 having raduis of convergence 𝑅 = ∞.
Therefore,

−𝑥 2
1 1
𝑒 + 𝑐𝑜𝑠𝑥 = ∑(−1)𝑛 ( + ) 𝑥 2𝑛 , for all 𝑥
𝑛! (2𝑛)!
𝑛=0

Exercise 4.3.5: Find the Taylor series expansion of 𝑓 at 0 (Maclaurine series expansion of 𝑓) and
find the radius of convergence of the power series.

Tesfaye G and Wanaw B Page 168


CALCULUS II

1
𝑎. 𝑓(𝑥) = 𝑐. 𝑓(𝑥) = 𝑐𝑜𝑠𝜋𝑥
(1 − 𝑥)2
2
𝑏. 𝑓(𝑥) = 𝑒 −2𝑥 𝑑. 𝑓(𝑥) = 𝑠𝑖𝑛(𝑥 4 )
𝜋
Example 4.3.20: Find the Taylor series of 𝑠𝑖𝑛𝑥 about 4 and show that it converges to 𝑠𝑖𝑛𝑥 for

all 𝑥.
Solution:
𝜋 𝜋 𝜋 𝜋
𝑠𝑖𝑛𝑥 = 𝑠𝑖𝑛 (𝑥 − + ) = 𝑠𝑖𝑛 ((𝑥 − ) + )
4 4 4 4
𝜋 𝜋 𝜋 𝜋
= 𝑠𝑖𝑛 (𝑥 − ) 𝑐𝑜𝑠 + 𝑠𝑖𝑛 𝑐𝑜𝑠 (𝑥 − )
4 4 4 4
But,
∞ 𝜋 2𝑛+1
𝜋 (𝑥 − 4 )
𝑠𝑖𝑛 (𝑥 − ) = 𝑠𝑖𝑛𝑥 = ∑(−1)𝑛 , for all 𝑥
4 (2𝑛 + 1)!
𝑛=0

And
∞ 𝜋 2𝑛
𝜋 (𝑥 − 4)
𝑐𝑜𝑠 (𝑥 − ) = ∑(−1)𝑛 , for all 𝑥
4 (2𝑛)!
𝑛=0

Thus,
∞ 𝜋 2𝑛+1 ∞ 𝜋 2𝑛
√2 (𝑥 − 4) √2 (𝑥 − 4)
𝑠𝑖𝑛𝑥 = (∑(−1)𝑛 )+ (∑(−1)𝑛 )
2 (2𝑛 + 1)! 2 (2𝑛)!
𝑛=0 𝑛=0

∞ 𝜋 2𝑛+1 ∞ 𝜋 2𝑛
√2 (𝑥 − 4 ) (𝑥 − 4)
= (∑(−1)𝑛 + ∑(−1)𝑛 )
2 (2𝑛 + 1)! (2𝑛)!
𝑛=0 𝑛=0

𝜋 𝜋 3 𝜋 5 𝜋 7
√2 (𝑥 − 4) (𝑥 − 4) (𝑥 − 4 ) (𝑥 − 4 )
= ( − + − +⋯)
2 1! 3! 5! 7!
(
𝜋 2 𝜋 4 𝜋 6
(𝑥 − 4 ) (𝑥 − 4) (𝑥 − 4)
+ (1 − + − +⋯)
2! 4! 6!
)

Tesfaye G and Wanaw B Page 169


CALCULUS II

𝜋 𝜋 2 𝜋 3 𝜋 4 𝜋 5 𝜋 6 𝜋 7
√2 (𝑥 − 4) (𝑥 − 4) (𝑥 − 4 ) (𝑥 − 4 ) (𝑥 − 4 ) (𝑥 − 4) (𝑥 − 4)
= (1 + − − + + − −
2 1! 2! 3! 4! 5! 6! 7!

+⋯)

𝜋
Therefore, the Taylor series expansion of 𝑠𝑖𝑛𝑥 about 4 is

𝜋 𝜋 2 𝜋 3 𝜋 4 𝜋 5
√2 √2 (𝑥 − 4 ) √2 (𝑥 − 4) √2 (𝑥 − 4) √2 (𝑥 − 4) √2 (𝑥 − 4 )
𝑠𝑖𝑛𝑥 = + − − + +
2 2(1!) 2(2!) 2(3!) 2(4!) 2(5!)
𝜋 6 𝜋 7
√2 (𝑥 − 4) √2 (𝑥 − 4)
− − +⋯ , for all 𝑥
2(6!) 2(7!)

Example 4.3.21: Find the Taylor series of 𝑒 𝑥 about 3 and show that it converges to 𝑒 𝑥 for all 𝑥.
Solution:
𝑒 𝑥 = 𝑒 (𝑥−3)+3 = 𝑒 𝑥−3 𝑒 3
But,

𝑥−3
(𝑥 − 3)𝑛
𝑒 =∑ , for all 𝑥
𝑛!
𝑛=0

Hence,
∞ ∞
𝑥 3
(𝑥 − 3)𝑛 𝑒 3 (𝑥 − 3)𝑛
𝑒 = 𝑒 (∑ )=∑ , for all 𝑥
𝑛! 𝑛!
𝑛=0 𝑛=0

Therefore, the Taylor series expansion of 𝑒 𝑥 about 3 is



𝑥
𝑒 3 (𝑥 − 3)𝑛
𝑒 =∑ , for all 𝑥
𝑛!
𝑛=0

Tesfaye G and Wanaw B Page 170


CALCULUS II

Example

Tesfaye G and Wanaw B Page 171


CALCULUS II

Tesfaye G and Wanaw B Page 172


CALCULUS II

Chapter Summary
 A series of the form

∑ 𝑐𝑛 𝑥 𝑛 = 𝑐0 + 𝑐1 𝑥 + 𝑐2 𝑥 2 + 𝑐3 𝑥 3 + 𝑐4 𝑥 4 + ⋯
𝑛=0

where 𝑥 is a variable and 𝑐𝑛 ’s are constants is called a power series (a power series in 𝑥).
Generally, a series of the form

∑ 𝑐𝑛 (𝑥 − 𝑎)𝑛 = 𝑐0 + 𝑐1 (𝑥 − 𝑎) + 𝑐2 (𝑥 − 𝑎)2 + 𝑐3 (𝑥 − 𝑎)3 + 𝑐4 (𝑥 − 𝑎)4 + ⋯


𝑛=0

where 𝑥 is a variable and 𝑐𝑛 ’s are constants is called a power series in 𝑥 − 𝑎 or a power series
about 𝑎 or a power serie centered at 𝑎.
 For a given series

∑ 𝑐𝑛 (𝑥 − 𝑎)𝑛
𝑛=0

there are only three possibilities.


iv. The series converges only when 𝑥 = 𝑎.
v. The series converges for all 𝑥.
vi. There is a positive number 𝑅 such that the series converges if |𝑥 − 𝑎| < 𝑅 and
diverges |𝑥 − 𝑎| > 𝑅.

The number 𝑅 in case (iii) is called radius of convergence of a power series.


 The interval of convergence of a power series is the interval that consists of all values of
𝑥 for which the series converges.
 Suppose
∞ ∞

∑ 𝑝𝑛 (𝑥 − 𝑎)𝑛 and ∑ 𝑞𝑛 (𝑥 − 𝑎)𝑛


𝑛=0 𝑛=0

be two power series having interval of convergence 𝐼1 and 𝐼2 respectively. Then,


∞ ∞ ∞

𝑖. ∑ 𝑝𝑛 (𝑥 − 𝑎) ± ∑ 𝑞𝑛 (𝑥 − 𝑎) = ∑ 𝑐𝑛 (𝑥 − 𝑎)𝑛
𝑛 𝑛

𝑛=0 𝑛=0 𝑛=0

where 𝑐𝑛 = 𝑝𝑛 ± 𝑞𝑛 , for 𝑛 ≥ 0.

Tesfaye G and Wanaw B Page 173


CALCULUS II

Moreover, the series


∑ 𝑐𝑛 (𝑥 − 𝑎)𝑛
𝑛=0

converges in the interior of the common interval of convergence.


∞ ∞ ∞

𝑖𝑖. (∑ 𝑝𝑛 (𝑥 − 𝑎) ) (∑ 𝑞𝑛 (𝑥 − 𝑎) ) = ∑ 𝑐𝑛 (𝑥 − 𝑎)𝑛
𝑛 𝑛

𝑛=0 𝑛=0 𝑛=0

where 𝑐𝑛 = 𝑝𝑛 𝑞0 + 𝑝𝑛−1 𝑞1 + 𝑝𝑛−2 𝑞2 + ⋯ + 𝑝0 𝑞𝑛 .


Moreover, the series

∑ 𝑐𝑛 (𝑥 − 𝑎)𝑛
𝑛=0

converges in the interior of the common interval of convergence.


𝑛 ∞
∑∞
𝑛=0 𝑝𝑛 (𝑥 − 𝑎)
𝑖𝑖𝑖. = ∑ 𝑐𝑛 (𝑥 − 𝑎)𝑛
∑∞
𝑛=0 𝑞𝑛 (𝑥 − 𝑎)
𝑛
𝑛=0

where 𝑞𝑛 = 𝑐𝑛 𝑞0 + 𝑐𝑛−1 𝑞1 + 𝑐𝑛−2 𝑞2 + ⋯ + 𝑐0 𝑞𝑛 .


 Let the power series

∑ 𝑐𝑛 (𝑥 − 𝑎)𝑛
𝑛=0

has a radius of convergence 𝑅 > 0. Then, the function 𝑓 defined by


𝑓(𝑥) = ∑ 𝑐𝑛 (𝑥 − 𝑎)𝑛 = 𝑐0 + 𝑐1 (𝑥 − 𝑎) + 𝑐2 (𝑥 − 𝑎)2 + 𝑐3 (𝑥 − 𝑎)3 + 𝑐4 (𝑥 − 𝑎)4 + ⋯


𝑛=0

is differentiable (therefore continuous) on an interval of convergence (𝑎 − 𝑅, 𝑎 + 𝑅) and


∞ ∞ ∞
𝑑 𝑑
𝑖. 𝑓 ′ (𝑥) = (∑ 𝑐𝑛 (𝑥 − 𝑎)𝑛 ) = ∑ (𝑐 (𝑥 − 𝑎)𝑛 ) = ∑ 𝑛𝑐𝑛 (𝑥 − 𝑎)𝑛−1
𝑑𝑥 𝑑𝑥 𝑛
𝑛=0 𝑛=0 𝑛=1
∞ ∞ ∞
𝑛 𝑛 )𝑑𝑥
𝑐𝑛 (𝑥 − 𝑎)𝑛+1
𝑖𝑖. ∫ 𝑓(𝑥)𝑑𝑥 = ∫ (∑ 𝑐𝑛 (𝑥 − 𝑎) ) 𝑑𝑥 = ∑ ∫(𝑐𝑛 (𝑥 − 𝑎) =∑ +𝑐
𝑛+1
𝑛=0 𝑛=0 𝑛=1

Moreover, both power series in (𝑖) and (𝑖𝑖) above has a radius of convergenc 𝑅.
 If a function 𝑓 has a power series representation at 𝑎, then

Tesfaye G and Wanaw B Page 174


CALCULUS II


𝑓 (𝑛) (𝑎)
𝑓(𝑥) = ∑ (𝑥 − 𝑎)𝑛
𝑛!
𝑛=0

𝑓 ′ (𝑎) 𝑓 ′′ (𝑎) 𝑓 ′′′ (𝑎) 𝑓 (4) (𝑎)


= 𝑓(𝑎) + (𝑥 − 𝑎) + (𝑥 − 𝑎)2 + (𝑥 − 𝑎)3 + (𝑥 − 𝑎)4 + ⋯
1! 2! 3! 4!
This power series is called Taylor series expansion of a function at 𝑎.
 If a function 𝑓 has a power series representation at 0, then


𝑓 (𝑛) (0) 𝑛
𝑓(𝑥) = ∑ 𝑥
𝑛!
𝑛=0

𝑓 ′ (0) 𝑓 ′′ (0) 2 𝑓 ′′′ (0) 3 𝑓 (4) (0) 4


= 𝑓(0) + 𝑥+ 𝑥 + 𝑥 + 𝑥 +⋯
1! 2! 3! 4!
This power series is called Maclaurine series expansion, which is a particular case of Taylor
series expansion of a function.

Tesfaye G and Wanaw B Page 175


CALCULUS II

Review Exercise
1. Find the radius of convergence and interval of convergence of the following power series.

∞ ∞
𝑛(𝑥 − 4)𝑛 𝑛3 (𝑥 − 1)𝑛
𝑎. ∑ 𝑒. ∑
𝑛3 + 1 7𝑛+1
𝑛=0 𝑛=0
∞ ∞
𝑥 2𝑛 𝑛
5𝑛 (𝑥 − 2)𝑛
𝑏. ∑(−1) 2𝑛 𝑓. ∑
2 (𝑛!)2 8𝑛7
𝑛=0 𝑛=1
∞ 2 𝑛 ∞
𝑛 𝑥 (𝑥 + 1)2𝑛
𝑐. ∑ 𝑔. ∑
10𝑛 4𝑛−1
𝑛=0 𝑛=0
∞ ∞
𝑛𝑥 3𝑛 𝑛𝑥 3𝑛
𝑑. ∑ 𝑛−1 ℎ. ∑
8 3𝑛
𝑛=0 𝑛=0
2. Find the power series representation of 𝑓 and determine the radius of convergence and
interval of convergence of the power series.
𝑥3
a. 𝑓(𝑥) = (𝑥−2)2 c. 𝑓(𝑥) = ln(𝑥 − 1)
𝑥
b. 𝑓(𝑥) = 𝑡𝑎𝑛−1 ( )
√3

3. Evaluate the indefinite integral as a power series. What is the radius of convergence?
𝑥 𝑥
d. ∫ 1−𝑥 8 𝑑𝑥 c. ∫ 𝑡𝑎𝑛−1 (3) 𝑑𝑥
ln(4−𝑥)
e. ∫ 𝑑𝑥
𝑥2

4. Evaluate the appropriate value of the following integral


0.5 1 0.3 𝑥 2
a. ∫0 𝑑𝑥 b. ∫0 𝑑𝑥
1+𝑥 5 1+𝑥 4

5. Find the Taylor series expansion of 𝑓 at 0 (Maclaurine series expansion of 𝑓) and find the
radius of convergence of the power series..
2 𝑠𝑖𝑛𝑥
𝑎. 𝑓(𝑥) = √1 − 𝑥 𝑐. 𝑓(𝑥) = 𝑒 −𝑥 + 𝑥
𝑏. 𝑓(𝑥) = 2𝑥 𝑑. 𝑓(𝑥) = 𝑥 2 𝑒 −𝑥
𝜋
6. Find the Taylor series expansion of 𝑐𝑜𝑠𝑥 about 3 and show that it converges to it for all 𝑥.
𝜋
7. Find the Taylor series expansion of 𝑠𝑖𝑛𝑥 about − 6 and show that it converges to it for all 𝑥.

8. Find the Taylor series of 𝑒 𝑥 about −1 and show that it converges to 𝑒 𝑥 for all 𝑥.

Tesfaye G and Wanaw B Page 176


CALCULUS II

References:

 Angus E, W. Robert Mann, Advanced Calculus, Third Edition, John-Wiley and Son,
INC., 1995.
 Robert Wrede, Murray R. Spiegel, Theory of advanced calculus, Second Edition.,
McGraw-Hill, 2002.
 Wilfred Kaplan, Advanced Calculus, Fifth Edition
 James Stewart, Calculus early transcendendentals, sixth edition.
 Thomas, Calculus, eleventh edition
 Paul Dawkins, Calculus III, 2007
 Robert Ellis and Gulick, Calculus with analytic geometry, sixth edition.
 Leithold, the calculus with analytic geometry, third edition, Herper and Row, publishers.
 Adams, Calculus: A complete course, Fifth edition, Addison Wesley, 2003.
 E.J.Purcell and D.Varberg, Calculus with analytic geometry, Prentice-Hall INC., 1987.
 Hans Sagan, Advanced Calculus,
 R.T.Smith and R.B.Minton, Calculus concepts and connections, McGraw-Hill book
company, 2006.
 Karl Heinz Dovermann, Applied Calculus (Math 215), July 1999.
 Serge Lang, Calculus of several variables, November 1972.
 R. Tavakol, Mas102 Calculus II, Queen Mary University of London 2001-2003

Tesfaye G and Wanaw B Page 177

You might also like